You are on page 1of 240

OGP 2019 GEOGRAPHY MODULE MCQS

Table of Contents
WORLD PHYSICAL GEOGRAPHY ................................................................................................................................................ 2
UNIVERSE AND THEORIES ..................................................................................................................................................... 2
SOLAR SYSTEM AND OTHER CELESTRIAL BODIES ................................................................................................................. 3
EARTH AND ITS PROPERTIES ................................................................................................................................................. 7
GEOMAGNETISM........................................................................................................................... 14
THE ORIGIN OF EARTH AND EVOLUTION ............................................................................................................................ 16
CONTINENAL DRIFT THEORY ......................................................................................................... 18
PLATE TECTONIC............................................................................................................................ 18
INTERIOR OF THE EARTH ..................................................................................................................................................... 20
DISTRIBUTION OF OCEANS AND CONTNENTS .................................................................................................................... 21
LANDFORMS ........................................................................................................................................................................ 22
MINERALS AND ROCKS .................................................................................................................. 22
GEOMORPHIC PROCESSES ............................................................................................................ 27
LANDFORMS AND THEIR EVOLUTION ........................................................................................... 27
CLIMATE .............................................................................................................................................................................. 32
COMPOSITION AND STRUCTURE OF ATMOSPHERE ..................................................................... 33
SOLAR RADIATION,HEAT BALANCE AND TEMPERATURE.............................................................. 34
ATMOSPHERIC CIRCULATION AND WEATHER SYSTEMS ............................................................... 37
WATER IN THE ATMOSPHERE ....................................................................................................... 43
WORLD CLIMATE AND CLIMATE CHANGE..................................................................................... 45
WATER(OCEANS) ................................................................................................................................................................. 50
WATER ........................................................................................................................................... 50
MOVEMENT OF OCEAN WATER .................................................................................................... 53
INDIAN PHYSICAL GEOGRAPHY ............................................................................................................................................... 55
PHYSIOGRAPHY ................................................................................................................................................................... 55
STRUCTURE AND PHYSIOGRAPHY ................................................................................................. 55
DRAINAGE SYSTEM........................................................................................................................ 70
CLIMATE, VEGETATION AND SOIL ....................................................................................................................................... 76
CLIMATE ........................................................................................................................................ 76
MONSOON .................................................................................................................................... 82
VEGETATION.................................................................................................................................. 89
SOILS .............................................................................................................................................. 96
NATURAL HAZARDS AND DIASASTER:CAUSES CONSEQUENCES AND MANAGEMENT ...................................................... 99
https://telegram.me/UPSC_PDF

OGP 2019 GEOGRAPHY MODULE MCQS

CYCLONE........................................................................................................................................ 99
EARTHQUAKE AND TSUNAMI...................................................................................................... 103
FLOODS........................................................................................................................................ 109
OTHER.......................................................................................................................................... 110
INSTITUTIONS .............................................................................................................................. 113
HUMAN GEOGRAPHY ............................................................................................................................................................ 115
POPULATION ..................................................................................................................................................................... 115
DISTRIBUTION ............................................................................................................................. 115
DENSITY ....................................................................................................................................... 116
GROWTH ..................................................................................................................................... 116
COMPOSITION ............................................................................................................................. 117
URBANIZATION............................................................................................................................ 118
MIGRATION ....................................................................................................................................................................... 120
TYPES ........................................................................................................................................... 120
CAUSES ........................................................................................................................................ 120
CONSEQUENCES .......................................................................................................................... 120
HUMAN DEVELOPMENT.................................................................................................................................................... 120
HUMAN SETTLEMENTS ..................................................................................................................................................... 120
LAND RESOURCES AND AGRICULTURE ............................................................................................................................. 120
WATER RESOURCES........................................................................................................................................................... 136
MINERAL AND ENERGY RESOURCES ................................................................................................................................. 144
MANUFACTURING INDUSTRIES......................................................................................................................................... 153
PLANNING AND SUSTAINABLE DEVELOPMENT IN INDIA.................................................................................................. 153
TRANSPORTATION AND COMMUNICATION ..................................................................................................................... 153
INTERNATIONAL TRADE .................................................................................................................................................... 158
LOCATION BASED QUESTIONS (MAPPING) ........................................................................................................................... 158
INDIA BASED ...................................................................................................................................................................... 158
WORLD BASED................................................................................................................................................................... 177

WORLD PHYSICAL GEOGRAPHY


UNIVERSE AND THEORIES

1. Which of the following best describes the shape of our Milky Way galaxy?

Direct PDF - https://telegram.me/UPSC_PrelimsTest


https://telegram.me/UPSC_PDF

OGP 2019 GEOGRAPHY MODULE MCQS

a) Geoid
b) Sphere
c) Cylinder
d) Barred spiral

Solution: D
Justification: A barred spiral galaxy is a spiral galaxy with a central bar-shaped structure composed of stars.
Unlike a regular spiral, a barred spiral contains a bar across its center region, and has two major arms. The
Milky Way also contains two significant minor arms, as well as two smaller spurs. One of the spurs, known as
the Orion Arm, contains the sun and the solar system. The Orion arm is located between two major arms,
Perseus and Sagittarius.

SOLAR SYSTEM AND OTHER CELESTRIAL BODIES

2. Consider the following about Solar flares and coronal mass ejections (CMEs):
1) A CME can jostle Earth’s magnetic fields creating currents that drive particles down toward Earth's poles.
2) Both eruptions are created when the motion of the sun’s interior contorts its own magnetic fields.
3) While solar flares involve huge explosions of energy, CMEs involve formation of energy suctions in space
that consume energy from its surroundings.
4) CMEs travel at the speed of light, Solar flares do not.

Select the correct answer using the codes below.


a) 1 and 2 only
b) 3 and 4 only
c) 1, 2 and 3 only
d) 1, 2 and 4 only

Solution: A
Justification: Statement 1: Flares and CMEs have different effects at Earth as well. The energy from a flare
can disrupt the area of the atmosphere through which radio waves travel. This can lead to degradation and,
at worst, temporary blackouts in navigation and communications signals. On the other hand, CMEs can funnel
particles into near-Earth space. A CME can jostle Earth’s magnetic fields creating currents that drive particles
down toward Earth's poles. When these react with oxygen and nitrogen, they help create the aurora, also
known as the Northern and Southern Lights. Additionally, the magnetic changes can affect a variety of human
technologies.
Statement 3: Like the sudden release of a twisted rubber band, the magnetic fields explosively realign, driving
vast amounts of energy into space. This phenomenon can create a sudden flash of light -- a solar flare. The
magnetic contortions can also create a different kind of explosion that hurls solar matter into space.
Statement 4: Flares can last minutes to hours and they contain tremendous amounts of energy. Traveling at
the speed of light, it takes eight minutes for the light from a solar flare to reach Earth. Some of the energy
released in the flare also accelerates very high energy particles that can reach Earth in tens of minutes. The
CME is an immense cloud of magnetized particles hurled into space. Traveling over a million miles per hour,
the hot material called plasma takes up to three days to reach Earth. The differences between the two types
of explosions can be seen through solar telescopes, with flares appearing as a bright light and CMEs
appearing as enormous fans of gas swelling into space.

3. Earth’s orbital velocity around the Sun varies with


a) Shape of Earth’s orbit
b) It remains constant throughout the orbit.
c) Earth’s distance from the Sun
d) Both (a) and (c)

Solution: D

Direct PDF - https://telegram.me/UPSC_PrelimsTest


https://telegram.me/UPSC_PDF

OGP 2019 GEOGRAPHY MODULE MCQS

Justification: As the earth moves close to the Sun, its orbital velocity increases, and decreases as it goes
farther from the Sun. Based on Kepler’s three laws of motion (9th standard physics), you will know that in an
elliptical orbit, a body will trace equal orbital area in equal intervals of time. So, depending on the shape of
the orbit, the velocity of earth will vary

4. When the earth crosses the tail of a comet, it indicates


a) an impending solar spot cycle
b) possible sight of a meteor shower
c) an impending high tide event in oceans
d) None of the above

Solution: B
Justification: A meteor shower is a celestial event in which a number of meteors are observed to radiate, or
originate, from one point in the night sky. A meteor shower is the result of an interaction between a planet,
such as Earth, and streams of debris from a comet. A comet is an icy small Solar System body that, when
passing close to the Sun, heats up and begins to outgas, displaying a visible atmosphere or coma, and
sometimes also a tail. These phenomena are due to the effects of solar radiation and the solar wind upon the
nucleus of the comet. Some people think that comets are messengers of disasters, such as wars, epidemics
and floods. But these are all myths and superstitions. Appearance of a comet is a natural phenomenon

Direct PDF - https://telegram.me/UPSC_PrelimsTest


https://telegram.me/UPSC_PDF

OGP 2019 GEOGRAPHY MODULE MCQS

5. Consider the following statements.


1) Sunspots are temporary phenomena on the photosphere of the Sun.
2) They appear visibly as bright spots compared to surrounding regions.
3) They may affect the climate on earth by changing the solar output received by earth.

Select the correct answer using the codes below.


a) 1 only
b) 2 and 3 only
c) 1 and 3 only
d) 1, 2 and 3

Solution: C
Justification: They appear as darker spots. They reduce solar output if they increase in number. Moreover,
they reflect intense magnetic activity on the Sun at those spots. If the solar output is changed, it would affect
the temperature and wind patterns on earth causing either a short-term weather change or if persistent may
also cause a long-term climate change.

6. Consider the following about ‘fast radio burst’, recently seen in news:
1. They are found in parts of the sky outside the Milky Way galaxy.
2. They last for several years but felt on earth for a very short duration of time.

Which of the above is/are correct?


a) 1 only
b) 2 only
c) Both 1 and 2
d) None

Solution: A
Justification: In radio astronomy, a fast radio burst (FRB) is a high-energy astrophysical phenomenon of
unknown origin manifested as a transient radio pulse lasting only a few milliseconds. Fast radio bursts are
bright, unresolved (point source-like), broadband (spanning a large range of radio frequencies), millisecond
flashes found in parts of the sky outside the Milky Way. Unlike many radio sources the signal from a burst is
detected in a short period of time with enough strength to stand out from the noise floor.

7. With reference to coronal explosions, consider the following:


1. It produces large radiations and particles which may significantly influence the atmosphere of the star.
2. It has been observed only in the Sun.
3. If the flare from such an explosion reaches earth, it can damage electrical equipment and power grids.

Select the correct answer using the codes below.


a) 1 and 2 only
b) 1 and 3 only
c) 2 only
d) 1, 2 and 3

Solution: B
Justification: Coronal explosion is an unusually large release of plasma and magnetic field from the solar
corona. They often follow solar flares and are normally present during a solar prominence eruption. The
plasma is released into the solar wind.
Statement 2: Recently, India’s first dedicated space observatory AstroSat along with other space and earth-
based observatories have detected a massive coronal explosion on Proxima Centauri, sun’s closest star
neighbour.

Direct PDF - https://telegram.me/UPSC_PrelimsTest


https://telegram.me/UPSC_PDF

OGP 2019 GEOGRAPHY MODULE MCQS

Statement 3: The massive coronal explosion has sent out powerful solar flare having energy about 100 times
a typical solar flare. If such a massive flare occurs in our Sun, it might have a devastating effect on power
grids, interrupt broadcasts and electricity, affect electronic instruments, and cause excess UV radiation in
space.
Statement 1: Such powerful flaring may have produced large radiations and particles which may significantly
influence the atmosphere of the Proxima Centauri b and affect its habitability.

8. Consider the following statements.


1. Ursa Major constellation is most clearly visible from Southern Hemisphere as against a partial obstruction
from Northern Hemisphere.
2. Pole Star is a rough indicator of the direction toward the geographic North Pole.

Which of the above is/are correct?


a) 1 only
b) 2 only
c) Both 1 and 2
d) None

Solution: B
Justification: Statement 1: Ursa Major is visible throughout the year from most of the northern hemisphere,
and appears circumpolar above the mid-northern latitudes. From southern temperate latitudes, the main
asterism is invisible, but the northern parts of the constellation can still be viewed.
Statement 2: North Star has the property of remaining in a fixed position throughout the course of the night
and its use in celestial navigation. It is virtually fixed, and its angle of elevation can also be used to determine
latitude.

9. With reference to planets of the solar system and their orbital characteristics, consider the following
matches:
1. Uranus: Made entirely of rocks
2. Saturn: Longest spin time on own axis
3. Mars: Highest number of Moons
4. Mercury: Shortest orbital period around Sun

Select the correct answer using the codes below.


a) 1, 3 and 4 only
b) 4 only
c) 2 only
d) 1 and 3 only

Solution: B
Justification: Statement 1: Planets beyond Mars are gaseous planets, not terrestrial. They are made of gases.
Statement 2: It is Venus, with 243 days.
Statement 3: Saturn has the highest number of moons – about 18.
Statement 4: It takes only 88 days.

10. Consider the following statements.


1. In the Solar system planets, only Jupiter and Saturn have rings around them.
2. The rings surrounding the solar system planet are large enough to be visible from naked eyes.

Which of the above is/are correct?


a) 1 only
b) 2 only
c) Both 1 and 2

Direct PDF - https://telegram.me/UPSC_PrelimsTest


https://telegram.me/UPSC_PDF

OGP 2019 GEOGRAPHY MODULE MCQS

d) None

Solution: D
Justification: Statement 1: Jupiter, Saturn and Uranus have rings around them. These are belts of small
debris.
The rings appear to be created by dust thrown off by impacts on small moons.
Statement 2: These rings may be seen from the earth with the help of powerful telescopes, not with naked
eyes.

11. As meteoroids come near the earth and tend to drop upon it, we often see a flash of light. This is caused due
to
a) Air induced friction borne by meteoroids
b) Charged particle emission by meteoroids
c) Destruction of ozone layer by meteoroids
d) Ion exchange between Ionosphere and meteoroids

Solution: A
Justification: The small pieces of rocks which move around the sun are called meteoroids. Sometimes these
meteoroids tend to drop upon it. During this process due to friction with the air they get heated up and burn.
It causes a flash of light. Sometimes, a meteor without being completely burnt, falls on the earth and creates
a hollow.

EARTH AND ITS PROPERTIES

12. Consider the following statements.


1. Beyond the low earth orbit, spacecrafts face hazards from cosmic rays and solar flares.
2. Astronauts on interplanetary missions face health threats from cosmic rays beyond earth’s
magnetosphere.
Which of the above is/are correct?
a) 1 only
b) 2 only
c) Both 1 and 2
d) None

Solution: C
Justification: Statement 1: We have covered this partly in the previous tests. Solar energetic particles consist
primarily of protons accelerated by the Sun to high energies via proximity to solar flares and coronal mass
ejections. Heavy ions and low energy protons and helium particles are highly ionizing forms of radiation,
which produce distinct biological damage compared to X-rays and gamma-rays. Solar cells, integrated circuits,
and sensors can be damaged by radiation. Geomagnetic storms occasionally damage electronic components
on spacecraft. Electronics on satellites must be hardened against radiation to operate reliably. The Hubble
Space Telescope, among other satellites, often has its sensors turned off when passing through regions of
intense radiation.
Statement 2: The radiation environment of deep space is different from that on the Earth's surface or in low
Earth orbit, due to the much larger flux of high-energy galactic cosmic rays (GCRs), along with radiation from
solar proton events (SPEs) and the radiation belts. Galactic cosmic rays (GCRs) consist of high energy protons
(85%), helium (14%) and other high energy nuclei (HZE ions). The health threat from cosmic rays is the danger
posed by galactic cosmic rays (GCR) and solar energetic particles to astronauts on interplanetary missions or
any missions that venture through the Van-Allen Belts or outside the Earth's magnetosphere. They are one of
the most important barriers standing in the way of plans for interplanetary travel by crewed spacecraft, but
space radiation health risks also occur for missions in low Earth orbit such as the International Space Station
(ISS).

Direct PDF - https://telegram.me/UPSC_PrelimsTest


https://telegram.me/UPSC_PDF

OGP 2019 GEOGRAPHY MODULE MCQS

13. Duration of day and night on earth are altered or affected due to
1. Axial tilt of the earth and revolution of earth around the Sun
2. Polar bulge
3. Refraction of light by the atmosphere of the earth
Select the correct answer using the codes below.
a) 1 and 3 only
b) 2 and 3 only
c) 1 only
d) 1, 2 and 3

Solution: A
Justification: Statement 1: Given that the Earth's own axis of rotation is inclined by about 23.5 degrees from
the perpendicular (as compared to its orbital plane), the length of the daytime period varies with seasons on
the planet's surface, depending on the observer's latitude. Areas experiencing summer are tilted toward the
sun. Their tilt toward the sun leads to over half of the day being in daylight and warmer temperatures due to
the increased directness of the sun's rays, the longer day period itself and less absorption of sunlight in the
atmosphere.
Statement 3: The Earth's atmosphere bends and diffuses light from the Sun and lengthens the period of
sunrise and sunset. For a certain period after sunset and before sunrise, indirect light from the Sun lightens
the sky on Earth; this period is often referred to as twilight.

14. Consider the following about the International Date Line.


1. It is a straight line that connects North Pole and South Pole.
2. It demarcates the change of one calendar day to the next.
3. The line passes through the Indian Ocean.
Select the correct answer using the codes below.
a) 2 only
b) 1 and 2 only
c) 2 and 3 only

Direct PDF - https://telegram.me/UPSC_PrelimsTest


https://telegram.me/UPSC_PDF

OGP 2019 GEOGRAPHY MODULE MCQS

d) 1, 2 and 3

Solution: A
Justification: Statement 1: It is not a straight line. For parts of its length, the IDL follows the meridian of 180°
longitude, roughly down the middle of the Pacific Ocean. To avoid crossing nations internally, the IDL deviates
west around the US Aleutian Islands, separating them from islands in the far east of Russia, and further south,
it deviates east around various island nations in the Pacific such as Kiribati, Samoa, Tonga and Tokelau.
Statement 2: A person who goes around the world from east to west (the same direction as Magellan's
voyage) would gain or set their clock back one hour for every 15° of longitude crossed, and would gain 24
hours for one circuit of the globe from east to west if they did not compensate by setting their clock forward
one day when they crossed the IDL. In contrast, a west-to-east circumnavigation of the globe loses an hour
for every 15° of longitude crossed but gains back a day when crossing the IDL. The IDL must therefore be
observed in conjunction with the Earth's time zones: on crossing it in either direction, the calendar date is
adjusted by one day.
Statement 3: It passes through the middle of the Pacific Ocean.

15. The movement of Sun from aphelion to Perihelion does not bring a lot of variation in the solar output to the
earth, despite earth being closer to the Sun on a perihelion, due to which of these factors?
1. Land and sea breezes
2. Discharge of particles due to solar wind
3. Milankovitch effect
Select the correct answer using the codes below.
a) 2 and 3 only
b) 1 and 2 only
c) 1 only
d) 1, 2 and 3

Solution: C
Justification: At perihelion, earth is the nearest to the sun receiving more solar insolation than at aphelion.
Statement 2: The solar wind is a stream of charged particles released from the upper atmosphere of the Sun.
Planets with a weak or non-existent magnetosphere is subject to atmospheric stripping by the solar wind. It is
not related with the distribution of weather.
Statement 1: Distribution of land and sea, heat transfer by way of sea and land breezes etc. redistribute the
increased insolation so that the effects are not felt largely.
Statement 3: Milankovitch effect refer to change in axis, shape and tilt of earth’s spin. It is relevant to long
term climate change, and not in this context which is about short-term weather changes.

16. A Lunar Eclipse can occur only at the night of a full moon because it is only then that
a) The sun, Earth, and moon are aligned in a straight line with the Moon in the middle
b) The sun, Earth, and moon are aligned in a left angled position with the Moon at the right angle vortex
c) The sun, Earth, and moon are aligned in a right angled position with the earth at the right angle vortex
d) The sun, Earth, and moon are aligned in a straight line with the earth in the middle

Solution: D
Learning: A total lunar eclipse has the direct sunlight completely blocked by the earth's shadow. The only light
seen is refracted through the earth's shadow. Unlike a solar eclipse, which can be viewed only from a certain
relatively small area of the world, a lunar eclipse may be viewed from anywhere on the night side of the
Earth. A lunar eclipse lasts for a few hours, whereas a total solar eclipse lasts for only a few minutes at any
given place, due to the smaller size of the Moon's shadow

Direct PDF - https://telegram.me/UPSC_PrelimsTest


https://telegram.me/UPSC_PDF

OGP 2019 GEOGRAPHY MODULE MCQS

17. What would happen if the Earth stopped spinning immediately?


1. The atmosphere would come to a complete standstill instantly.
2. Objects on top of the earth would be thrown away with great speed.
3. Every spot in the Earth would have permanent daytime or night time all year long.
4. It would freeze in a very short period of time.

Select the correct answer using the codes below.


a) 1, 2 and 3 only
b) 2 only
c) 2, 3 and 4 only
d) 1 and 3 only

Solution: B
Justification: Statement 1: If the Earth stopped spinning suddenly, the atmosphere would still be in motion
with the Earth's original 1100 mile per hour rotation speed at the equator.
Statement 2: All of the land masses would be scoured clean of anything not attached to bedrock. This means
rocks, topsoil, trees, buildings, animals, and so on, would be swept away into the atmosphere. This is because
earth’s spin provided a centrifugal (a pseudo force) force towards the centre of the earth. When this
vanishes, there is a sudden jerk and due to your inertia (as you were moving with the earth) you would be
thrown away at great speed.
Statement 3: No, if the earth kept revolving (even if it stopped spinning), all places will see day and night. But,
the duration of 24 hours would turn into months.

Direct PDF - https://telegram.me/UPSC_PrelimsTest


https://telegram.me/UPSC_PDF

OGP 2019 GEOGRAPHY MODULE MCQS

Statement 4: We cannot conclude anything about the temperature drop for the entire earth. However, those
parts would freeze which fail to get abundant sunlight.

18. With reference to the International Date Line, consider the following statements.
1. It was established in response to Cold war situations after the Second World War
2. It passes through the mid-Atlantic Ocean
3. It functions as a ‘line of demarcation’ separating two consecutive calendar dates.
4. It has no legal international status and countries are free to choose the dates that they observe.

Select the correct answer using the codes below.


a) 1, 2 and 3 only
b) 1, 3 and 4 only
c) 3 and 4 only
d) 1 and 2 only

Solution: C
Justification: Statement 1 and 2: Established in 1884, it passes through the mid-Pacific Ocean and roughly
follows a 180 degrees longitude north-south line on the Earth. It is located halfway round the world from the
prime meridian—the zero degrees longitude established in Greenwich.
Statement 3: When you cross the date line, you are virtually time travelling! Cross to the west and it’s one
day later; cross back and you are a day behind.
Statement 4: Despite its name, the International Date Line has no legal international status. While the date
line generally runs north to south from pole to pole, it zigzags around political borders such as eastern Russia
and Alaska’s Aleutian Islands.

19. Geographically speaking, which of these countries is most likely to have at least three to four time zones?

a) Countries that stretch extensively from east to west.


b) Countries located near the sea.
c) Countries that have a large latitudinal stretch.
d) Countries that are located farthest from Greenwich.

Solution: A
Justification: Larger the longitudinal extent, larger the number of time zones. 15 degrees of longitude covers
nearly 1 hour of time zone. So, a country like Russia which is spread across 165 degrees of longitude is likely
to have around 11 time zones. Latitudinal stretch does not matter much if longitudinal stretch is lower. The
USA and Canada have six time zones extending from the Pacific coast to the Atlantic coast.

20. At equinox

a) Neither of the poles is tilted towards the sun


b) Sun’s rays do not fall directly over the equator
c) A solar eclipse is necessarily impending
d) A larger portion of the Southern Hemisphere gets light

Solution: A
Justification: There are three major events: summer solstice, winter solstice and equinox. In both solstice,
either northern or southern pole is inclined towards the Sun. This causes continuous daylight for about six
months at either of the poles depending on the kind of solstice. On 21st March and September 23rd, direct
rays of the sun fall on the equator. At this position, neither of the poles is tilted towards the sun; so, the
whole earth experiences equal days and equal nights. This is called an equinox.

Direct PDF - https://telegram.me/UPSC_PrelimsTest


https://telegram.me/UPSC_PDF

OGP 2019 GEOGRAPHY MODULE MCQS

21. Which of these are nearly of equal length?


1. Tropic of Cancer and Tropic of Capricorn
2. Equator and both tropics
3. Prime Meridian and other longitudes

Select the correct answer using the codes below.


a) 1 and 2 only
b) 3 only
c) 1 and 3 only
d) 2 only

Solution: C
Justification: Statement 1: Latitudes equidistant from equator will have more or less equal length.
Statement 2: Length of latitudes goes on decreasing towards poles. So, equator is the longest latitude
(imaginary line).
Statement 3: Unlike parallels of latitude, all meridians are of equal length.
Facts about lines of latitude:
 Are known as parallels.
 Run in an east-west direction.
 Measure distance north or south from the Equator.
 Are parallel to one another and never meet.
 Cross the prime meridian at right angles.
 Lie in planes that cross the Earth's axis at right angles.

22. Areas lying between the Arctic Circle and the North Pole in the Northern Hemisphere are very cold.
It is because
1. Earth’s effective radius is lesser at these points.
2. Sun rises above the horizon at these areas almost the entire year.
3. Air pressure at these locations is much lesser as compared to lower latitudes.

Select the correct answer using the codes below.


a) 1 only
b) 2 and 3 only
c) 2 only
d) None of the above

Solution: D
Justification: This is also the case with the area between Antarctic Circle and the South Pole in the Southern
Hemisphere.
Statement 1: An absurd statement.
Statement 2: Here the sun does not rise much above the horizon. Therefore, its rays are always slanting. So,
these areas receive much lesser solar insolation (heat) and remain cold. They are, therefore, called Frigid
Zones.
Statement 3: It is actually higher. Colder the air higher is its pressure.

23. The mid-day sun is exactly overhead at least once a year on

a) All latitudes excluding the polar latitude


b) All latitudes in between the Tropic of Cancer and the Tropic of Capricorn
c) Only the equator
d) All latitudes beyond both tropics

Solution: B

Direct PDF - https://telegram.me/UPSC_PrelimsTest


https://telegram.me/UPSC_PDF

OGP 2019 GEOGRAPHY MODULE MCQS

Justification: This is because earth is tilted at 23.5 degrees on its axis and keeps revolving around the Sun.
Due to this, sun’s rays do not always fall vertically above these latitudes. See diagram below.

Thus, the area between both tropics receives the maximum heat and is called the Torrid Zone.

24. Consider the following statement. By measuring the angle of the Pole Star from your place, you can know the
of your place.
1. Latitude
2. Longitude
3. Altitude

Select the correct answer using the codes below.


a) 1 only
b) 2 and 3 only
c) 1 and 2 only
d) 3 only

Solution: A
Justification:

Direct PDF - https://telegram.me/UPSC_PrelimsTest


https://telegram.me/UPSC_PDF

OGP 2019 GEOGRAPHY MODULE MCQS

Statement 1: If you were standing at the North Pole, the North Star would be directly above you. If you were
standing at the Earth's equator, the North Star would be right along the horizon. If you measure the angle of
the North Star above the horizon, that will be the same as your latitude. The North Pole is a latitude of 90
degrees north, and the equator is 0 degrees north. At the North Pole, the North Star is above, which is 90
degrees above the horizon. And, at the equator, the North Star is on the horizon, which is 0 degrees above
the horizon.
Statement 2: If you look at the diagram above, you will notice that you can’t decide your longitude because
the angle measurements from any longitude would be the same.

GEOMAGNETISM

25. Earth is largely protected from the solar wind by


a) Magnetic field
b) Ozone Layer
c) Tropospheric weather phenomena
d) Continuous revolution around the Sun

Solution: A
Justification:

Direct PDF - https://telegram.me/UPSC_PrelimsTest


https://telegram.me/UPSC_PDF

OGP 2019 GEOGRAPHY MODULE MCQS

Concept: The solar wind is a stream of charged particles released from the upper atmosphere of the Sun. This
plasma consists of mostly electrons, protons and alpha particles. The solar winds flow outward supersonically
at varying speeds and have the potential of stripping a planet’s atmosphere. Where the solar wind intersects
with a planet that has a well-developed magnetic field (such as Earth, Jupiter and Saturn), the particles are
deflected by the Lorentz force. This region, known as the magnetosphere, causes the particles to travel
around the planet rather than bombarding the atmosphere or surface. So, (a) is correct.
However, on earth some of the charged particles are trapped in the Van Allen radiation belt. The only time
the solar wind is observable on the Earth is when it is strong enough to produce phenomena such as the
aurora and geomagnetic storms.

26. Van Allen radiation belts would not have existed without

a) Headley cells on higher latitudes formed on earth


b) Magnetic field of earth
c) Seasonal change in the equators
d) Movement of cold currents in higher latitudes

Solution: B
Justification: The Van Allen belts are a collection of charged particles, gathered in place by Earth’s magnetic
field. Most of the particles that form the belts are thought to come from solar wind and other particles by
cosmic rays. By trapping the solar wind, the magnetic field deflects those energetic particles and protects the
Earth's atmosphere from destruction. Presumably the magnetic field of the Earth is generated by a dynamo
effect that involves its rotation. If the Earth stopped rotating, its magnetic field would no longer be
regenerated and it would decay away to some low, residual value due to the very small component which is
‘fossilized’ in its iron-rich rocks. There would be no more 'northern lights' and the Van Allen radiation belts
would probably vanish, as would our protection from cosmic rays and other high-energy particles. This is a
significant biohazard.

Direct PDF - https://telegram.me/UPSC_PrelimsTest


https://telegram.me/UPSC_PDF

OGP 2019 GEOGRAPHY MODULE MCQS

THE ORIGIN OF EARTH AND EVOLUTION

27. Evidence of multiple advances and retreats of glaciers, and the sediment deposits in glacial lakes reveal
1. The occurrence of warm and cold periods in the history of earth
2. Repeated attacks of permafrost on the glacial moraines over centuries
Which of the above is/are correct?
a) 1 only
b) 2 only
c) Both 1 and 2
d) None

Solution: A
Justification: Statement 1: An interglacial period (or alternatively interglacial) is a geological interval of
warmer global average temperature lasting thousands of years that separates consecutive glacial periods
within an ice age. It is a strong evidence of climate change.
Statement 2: Moraines are glacial deposits (soil). Permafrost is frozen carbon within rocks. It does not attack
or erode moraines. Permafrost is a deposit that is revealed only when the rocks melt and are exposed. Tree
rings also show evidence for wet and dry periods. Also, for e.g. near Rajasthan, 3000-1700 BC was a period of
higher rainfall, with 2000-1700 BC supporting Harappan civilization. Dry conditions accentuated since then.

28. Which of the following oceanic regions are the primary sites of generation of new crust, hosting mineral
resources and supporting unique ecosystems?
a) Littoral region
b) Continental Shelf
c) Mid-oceanic ridges
d) Continental Rise

Solution: C
Justification: The Mid-Ocean Ridge system forms the most extensive chain of mountains on Earth, with more
than 90 percent of the mountain range lying in the deep ocean - with a total length of about 60,000 km. Mid-
ocean ridges are geologically important because they occur along divergent plate boundaries, where new
ocean floor is created as the Earth’s tectonic plates spread apart. As the plates separate, some molten rock
rises to the seafloor, producing enormous volcanic eruptions of basalt, and building the longest chain of
volcanoes in the world. Because most of these eruptions occur deep under the water, they often go
unnoticed. This uplifting of the ocean floor occurs when convection currents rise in the mantle beneath the
oceanic crust and create magma where two tectonic plates meet at a divergent boundary.
Option B: It may host mineral resources, but it is not the region where new continental crust can be
generated since it may be devoid of the tectonic process that are pretty active in the mid-oceanic ridges

Direct PDF - https://telegram.me/UPSC_PrelimsTest


https://telegram.me/UPSC_PDF

OGP 2019 GEOGRAPHY MODULE MCQS

Direct PDF - https://telegram.me/UPSC_PrelimsTest


https://telegram.me/UPSC_PDF

OGP 2019 GEOGRAPHY MODULE MCQS

CONTINENAL DRIFT THEORY

29. The term “pole fleeing force” relates to the


a) Forces responsible for the bulging at the earth’s poles
b) Magnetic force as emitted through the poles aligning landforms on earth
c) Outward directed forces associated with the spinning of Earth
d) Forces that causes oscillations in the tilted axis of the earth

Solution: C
Justification: It is a geophysical concept invoked in 1915 by Alfred Wegener to explain his ideas of Continental
drift. He suggested that a differential gravitational force (horizontal component of centrifugal force) and the
Earth's flattening would cause continental masses to drift slowly towards the equator. This force is caused
due to the spinning of earth. The hypothesis was expanded in 1920 but the force is now known to be far too
weak to cause plate tectonics. The toughness of the sub layers of the Earth's crust is much stronger than
assumed by Wegener.

PLATE TECTONIC

30. Oceanic Crust is than/to the Continental Crust.


1. Older
2. Denser
3. Thicker

Direct PDF - https://telegram.me/UPSC_PrelimsTest


https://telegram.me/UPSC_PDF

OGP 2019 GEOGRAPHY MODULE MCQS

4. of the same chemical composition


Select the correct answer using the codes below.
a) 2 only
b) 1 and 3 only
c) 2 and 4 only
d) 1, 2 and 3

Solution: A
Justification: Oceanic crust is the result of erupted mantle material originating from below the plate, cooled
and in most instances, modified chemically by seawater. Oceanic crust is about 6 km thick, Continental about
10 Km. It is composed of several layers, not including the overlying sediment. The topmost layer, about 500
metres (1,650 feet) thick, includes lavas made of basalt (that is, rock material consisting largely of plagioclase
[feldspar] and pyroxene). Oceanic crust differs from continental crust in several ways: it is thinner, denser,
younger, and of different chemical composition. Like continental crust, however, oceanic crust is destroyed in
subduction zones. It is primarily composed of mafic rocks, or sima, which is rich in iron and magnesium;
continental is composed of Sial – i.e. Silica and Aluminum.

31. With reference to Plate tectonics, consider the following statements.


1. It is the theory that Earth's outer shell is divided into several plates that glide over the mantle.
2. It is another name for the theory of continental drift.
3. It discards the conventional geological view that there is convectional current flowing in the mantle.
Select the correct answer using the codes below.
a) 1 only
b) 2 and 3 only
c) 3 only
d) 1 and 3 only

Solution: A
Justification: The plates act like a hard and rigid shell compared to Earth's mantle. This strong outer layer is
called the lithosphere. Plate tectonics is the modern version of continental drift, a theory first proposed by
scientist Alfred Wegener in 1912. Wegener didn't have an explanation for how continents could move around
the planet, but researchers do now. Plate tectonics is thus said to be the unifying theory of geology. The
driving force behind plate tectonics is convection in the mantle. Hot material near the Earth's core rises, and
colder mantle rock sinks. In terms of analogy, it is kind of a pot boiling on a stove. The convection drive plates
tectonics through a combination of pushing and spreading apart at mid-ocean ridges and pulling and sinking
downward at subduction zones.

32. Which of the following about the tectonic ‘Indian Plate’ is NOT correct?
a) It is a major tectonic plate.
b) It forms a convergent late boundary with the Himalayas.
c) The Indian plate is tectonically separated from the Peninsular India plate.
d) The plate extends to Pakistan and Myanmar as well.

Solution: C
Justification: Plates are divided between major and minor plates based on their geographical coverage.
Indian plate is thus a major plate. So, (a) is correct.
The subduction zone along the Himalayas forms the northern plate boundary in the form of continent—
continent convergence. So, (b) is correct.
Peninsular plate is an integral part of the Indian plate. So, (c) is incorrect.
In the east, it extends through Rakinyoma Mountains of Myanmar towards the island arc along the Java
Trench. The Western margin follows Kirthar Mountain of Pakistan. So, (d) is correct

Direct PDF - https://telegram.me/UPSC_PrelimsTest


https://telegram.me/UPSC_PDF

OGP 2019 GEOGRAPHY MODULE MCQS

33. Lithospheric plates move around very slowly – just a few millimetres each year. What is the reason behind
this movement?

a) Ocean currents
b) Movement of magma inside earth
c) Formation of folds on earth’s crust
d) Rotation of earth

Solution: B
Justification: The lithosphere is broken into a number of plates known as the Lithospheric plates. The molten
magma inside the earth moves in a circular manner. These plates move because of the movement of the
molten magma inside the earth. Lateral movements between lithospheric plates create transform faults at
the sites of plate slippage.

INTERIOR OF THE EARTH

34. Most of the Earth’s iron is stored in


a) Crust
b) Mantle
c) Core
d) Lithosphere

Solution: C
Justification: Iron makes up 5 percent of Earth’s crust and is second in abundance to aluminium among the
metals and fourth in abundance behind oxygen, silicon, and aluminium among the elements. Iron, which is
the chief constituent of Earth’s core, is the most abundant element in Earth as a whole (about 35 percent)
and is relatively plentiful in the Sun and other stars. In the crust the free metal is rare, occurring as terrestrial
iron

Direct PDF - https://telegram.me/UPSC_PrelimsTest


https://telegram.me/UPSC_PDF

OGP 2019 GEOGRAPHY MODULE MCQS

35. Consider the following statements.


1. The crust is richer in Silicon than it is in oxygen and iron.
2. The crust is relatively enriched in less dense compounds and relatively depleted in iron compared to the
rest of the earth.
Which of the above is/are correct?
a) 1 only
b) 2 only
c) Both 1 and 2
d) None

Solution: B
Justification: The most common chemical elements in the crust are oxygen (46.6%), silicon (27.7), aluminium
(8.1), iron (5.0), calcium (3.6), potassium (2.8), sodium (2.6), and magnesium (2.1). Typical mantle rocks have
a higher magnesium to iron ratio and a smaller proportion of silicon and aluminium than the crust. The solid,
inner core of earth is iron and has a radius of about 760 miles. It is surrounded by a liquid, outer core
composed of a nickel-iron alloy.

36. Consider the following statements


1. The gravitation force (g) is not the same at different latitudes on the surface of the earth.
2. Uneven distribution of mass of material within the earth influences the gravitational force as experienced
within the earth.

Which of the above is/are correct?


a) 1 only
b) 2 only
c) Both 1 and 2
d) None

Solution: C
Justification: Statement 1: It is greater near the poles and less at the equator. This is because of the distance
from the centre at the equator being greater than that at the poles.
Statement 2: Distribution of mass affects the effective mass that is applying the gravitational force. Since not
all earthly mass has same density, there is bound to be a difference in the way gravitational force is
experienced within the earth.

DISTRIBUTION OF OCEANS AND CONTNENTS

37. The annual range of surface temperature is much greater in the North Atlantic and North Pacific oceans than
in the southern oceans because
1. Cold winds blow from the continents toward the oceans and greatly reduce the winter temperatures
2. Westerlies from mid-latitude regions increase the maximum annual variations in the sea surface
temperature
Which of the above is/are correct?
a) 1 only
b) 2 only
c) Both 1 and 2
d) None

Solution: A
Justification: The annual variation of surface temperature in any ocean depends upon several factors, the
most important among which are the annual variations of radiation income and the prevailing winds. The

Direct PDF - https://telegram.me/UPSC_PrelimsTest


https://telegram.me/UPSC_PDF

OGP 2019 GEOGRAPHY MODULE MCQS

annual range over most of the oceans of the world is less than 5 oc. Maximum annual variations in the sea
surface temperature occur generally in mid-latitude regions of the oceans. The ranges are generally greater
close to the land, due to the land effect and are greater in the northern hemispheric oceans. The annual
range of surface temperature is much greater in the North Atlantic and North Pacific oceans than in the
southern oceans. The high ranges in the northern oceans are associated with the character of the prevailing
winds. In these parts of the oceans, cold winds blow from the continents toward the oceans and greatly
reduce the winter temperatures. So, annual variations of up to 10c are found sometimes in these oceans due
to low winter temperatures and high summer temperatures.

38. Consider the following about the Geography of the Antarctica continent.
1. It is positioned symmetrically around the South Pole.
2. It lies largely north of the Antarctic Circle.
Which of the above is/are correct?
a) 1 only
b) 2 only
c) Both 1 and 2
d) None

Solution: D
Justification: Positioned asymmetrically around the South Pole and largely south of the Antarctic Circle,
Antarctica is the southernmost continent and is surrounded by the Southern Ocean. Alternatively, it may be
considered to be surrounded by the southern Pacific, Atlantic, and Indian Oceans, or by the southern waters
of the World Ocean.

LANDFORMS
MINERALS AND ROCKS

39. Fossils of ancient living things are best preserved in


a) Igneous rocks
b) Sedimentary rocks
c) Metamorphic rocks
d) Crystalline igneous rocks

Solution: B
Justification: Examples of fossils include bones, shells, exoskeletons, stone imprints of animals or microbes,
hair, petrified wood, oil, coal, and DNA remnants. In igneous rocks that are made entirely of lava deposits, or
in metamorphic rocks that may be buried deep beneath the earth, these are hard to find. These are best
preserved in sedimentary rocks that are layered rocks containing evidence of the evolution of life on earth.
Metamorphic rocks sometimes contain fossils if they were formed from a sedimentary rock, but the fossils
are usually squashed out of shape. The observation in the 19th century that certain fossils were associated
with certain rock strata led to the recognition of a geological timescale and the relative ages of different
fossils. The development of radiometric dating techniques in the early 20th century allowed scientists to
quantitatively measure the absolute ages of rocks and the fossils they host.

40. Texturally mature sediment is usually well rounded and well-sorted because
1. Rounding of sediment increases with transport distance and time
2. Sorting gets better as larger chunks are left behind and smaller chunks are carried away
Which of the above is/are correct?
a) 1 only
b) 2 only
c) Both 1 and 2
d) None

Direct PDF - https://telegram.me/UPSC_PrelimsTest


https://telegram.me/UPSC_PDF

OGP 2019 GEOGRAPHY MODULE MCQS

Solution: C
Justification: Sediment Maturity refers to the length of time that the sediment has been in the sedimentary
cycle. Texturally mature sediment is sediment that is well rounded, (as rounding increases with transport
distance and time) and well sorted (as sorting gets better as larger clasts are left behind and smaller clasts are
carried away. If the fragments are angular, it indicates that they have not been transported very far and the
sediment is poorly sorted.

41. Metamorphic rocks are formed due to


1. Cementation of sedimentary rocks
2. Close proximity of sedimentary rocks with molten magma
3. Cooling and solidification of lava
Select the correct answer using the codes below.
a) 1 only
b) 2 and 3 only
c) 2 only
d) 1 and 2 only

Solution: C
Justification: Metamorphic rocks are made by either heating up or squashing the earth's crust. They are
often found in mountainous regions. One example is slate. Slate was originally a black mud laid down on the
bottom of the sea or lake. Fossils can sometimes be found in it but they are often squashed. Other common
metamorphic rocks are called marble, gneiss, schist. Earth movements can cause rocks to be deeply buried or
squeezed. As a result, the rocks are heated and put under great pressure. They do not melt, but the minerals
they contain are changed chemically, forming metamorphic rocks. Sometimes, metamorphic rocks are
formed when rocks are close to some molten magma, and so get heated up. When a metamorphic rock is
formed under pressure, its crystals become arranged in layers.

42. Where hard and soft rocks exist parallel to each other, which of these drainage patterns is most likely to be
created?
a) Trellis
b) Dendritic
c) Radial
d) Rectangular

Solution: A
Justification: The streams within a drainage basin form various patterns depending on the slope of land,
underlying rock structure as well as the climatic conditions of the area. These are dendritic, trellis,
rectangular, and radial patterns. The dendritic pattern develops where the river channel follows the slope of
the terrain. The stream with its tributaries resembles the branches of a tree, thus the name dendritic. A river
joined by its tributaries, at approximately right angles, develops a trellis pattern. A trellis drainage pattern
develops where hard and soft rocks exist parallel to each other. A rectangular drainage pattern develops on a
strongly jointed rocky terrain. The radial pattern develops when streams flow in different directions from a
central peak or dome like structure.

43. Consider the following:


1. Metamorphism
2. Convergent plate boundaries
3. Volcanic activity
Which of the following rocks is often associated with the above?
a) Gneiss
b) Rock Salt
c) Limestone

Direct PDF - https://telegram.me/UPSC_PrelimsTest


https://telegram.me/UPSC_PDF

OGP 2019 GEOGRAPHY MODULE MCQS

d) Bituminous coal
Solution: A
Justification: Gneiss usually forms by regional metamorphism at convergent plate boundaries. It is a high-
grade metamorphic rock in which mineral grains recrystallized under intense heat and pressure associated
with mountain building and volcanic activity. This alteration increases the size of the mineral grains and
segregated them into bands, a transformation which made the rock and its minerals more stable in their
metamorphic environment. Gneiss can form in several different ways. The most common path begins with
shale, which is a sedimentary rock. Regional metamorphism can transform shale into slate, then phyllite, then
schist, and finally into gneiss.

44. Which of the following factors aid the weathering of rocks?


1. Intense heating during the day and rapid cooling at night
2. Pore pressure of water seeping into rocks
3. Thawing of rocks
Select the correct answer using the codes below.
a) 1 only
b) 1 and 3 only
c) 2 and 3 only
d) 1, 2 and 3

Solution: D
Justification: Block and Granular Disintegration type of weathering is carried out in deserts by rapid changes
of temperature, or in mountains through frost action. Desert regions experience a large diurnal range of
temperature due to direct heating by the sun during the day and rapid radiation at night. The rocks
successively expand and contract. The repeated expansion and contraction of the rocks tends to enlarge
joints and large rock masses ultimately break into angular blocks. This process is called block disintegration.
Rocks having vertical joints are more susceptible to this form of weathering. The rocks having curvilinear
joints get weathered. If water seeps in the cracks of the rocks, it exerts a further pressure from inwards
resulting in cracking of the rock.

45. The basic source of all minerals on earth is


a) Ocean basin churns
b) Solar winds and ion showers
c) Magma
d) Asteroids and meteorites

Solution: C
Justification: Magma cools slowly as it rises towards Earth’s surface. It can take thousands to millions of
years to become solid when it is trapped inside Earth. As the magma cools, solid rocks form (igneous). These
rocks are mixtures of minerals. Granite is a common rock that forms when magma cools. Granite contains the
minerals quartz, plagioclase feldspar, and potassium feldspar. The same igneous rock can be eroded,
weathered, transported, deposited and consolidated as sedimentary rocks which are the source of non-
metallic minerals.

46. The major characteristics of Archean rock system is that


a) It was formed before the appearance of life in the geologic sequence.
b) It was the first metamorphic sedimentary rock.
c) It hosts the major coal deposits of India.
d) These rocks are largely igneous.

Solution: A
Justification: Azoic Age was used to describe the age of rocks formed before the appearance of life in the
geologic sequence. Also known as Pre-Cambrian rocks these are the oldest rocks of the earth’s crust. The

Direct PDF - https://telegram.me/UPSC_PrelimsTest


https://telegram.me/UPSC_PDF

OGP 2019 GEOGRAPHY MODULE MCQS

Archean period covers over 85% of the total geological history time of earth and therefore is very significant.
This period marks the development of first photosynthesis, the life support atmosphere. The Archean or
Purana rock system in India is found in Aravallis mountains, 2/3rd of the Deccan peninsula and some parts of
north east. These rocks have abundant metallic and non-metallic minerals such as iron, copper, manganese,
bauxite, lead, zinc, gold, silver, tin, tungsten, mica, asbestos, graphite, etc. They serve as the basement
complex or fundamental gneisses.

47. What is/are the difference(s) between extrusive and intrusive rocks?
1. Extrusive rocks are formed from magma, whereas intrusive rocks are formed from lava.
2. Extrusive rocks are fine grained, whereas intrusive rocks are coarse-grained.
3. Extrusive rocks form over a much longer duration of time compared to intrusive rocks.
Select the correct answer using the codes below.
a) 2 only
b) 1 and 3 only
c) 3 only
d) 1 only

Solution: A
Justification: The key difference between intrusive and extrusive rocks is that the intrusive rocks are formed
from magma whereas the extrusive rocks are formed from lava. Rest of the differences follow from this basic
structure.
Intrusive rocks: With no air to cool the magma, these rocks are formed very slowly. Composition of these
rocks reflects presence of large crystals. These crystals interlock to form the rock. These rocks take a very
large amount of time to solidify and they remain buried deep inside the surface of the earth being
surrounded by country rocks that have been there already. Very slow cooling means that these rocks remain
coarse-grained. Some of the perfect examples of intrusive rocks are the diorite, gabbro, and granite. Much of
the core of various mountain ranges around the world is made up of these intrusive rocks.
Extrusive Rocks: Sometimes, molten rocks find a way to come out of the surface of the earth through cracks
and openings. This magma flows in the form of lava and cools down quickly as it comes into contact with air.
Igneous rocks that are formed from the magma that pours out of the surface of the earth are called extrusive
rocks. As these rocks cool down and solidify very quickly, they do not get sufficient time to form large crystals.
Thus, they have small crystals and boast a fine texture.

48. Which of the following processes contributes to the enrichment of minerals in rocks?

a) Weathering of rocks
b) Erosional and depositional action of rivers
c) Both (a) and (b)
d) None of the above

Solution: C
Justification: Enrichment of many low-grade ores occurs when the metal-bearing solutions of these valuable
metals drain downward along the vein or other deposits and then are re-precipitated into concentrated,
bonanza grade deposits at shallow depths. Some of these enriched deposits are quite large, being the product
of long, continued forces of weathering with the descending mineral salts acting to concentrate the metal
values from a great thickness of vein matter and rocks that were once located above the current outcrop, but
now have been removed by erosion. This process of natural concentration is how this the action of
weathering can produce such amazingly rich ore bodies.

49. Which of these is/are sign(s) that a rock might contain copper?
1. The rock will necessarily have a red appearance.
2. The rock will be exfoliated and consist of multiple layers.

Direct PDF - https://telegram.me/UPSC_PrelimsTest


https://telegram.me/UPSC_PDF

OGP 2019 GEOGRAPHY MODULE MCQS

Which of the above is/are correct?


a) 1 only
b) 2 only
c) Both 1 and 2
d) None

Solution: D
Justification: Statement 1: If a rock contains copper, it looks blue in colour. For e.g. Azurite contains copper,
which gives its blue colour and a specific gravity of 3.7 to 3.9, which is exceptionally high for a non-metallic
mineral.
Statement 2: This is not necessary. Rocks contain exfoliation or layers depending on their terrain and
geographical environment around.

50. The oldest rocks in the world can be found in which of these regions?

a) Western Australia
b) Southern Europe
c) North America
d) Eastern Asia

Solution: A
Justification: Rocks in Western Australia date from 4,300 million years ago, only 300 million years after the
earth was formed. These are Zircon stones. Confirmation of the zircon age holds enormous implications for
models of early Earth. Trace elements in the oldest zircons from Australia's Jack Hills range suggest they came
from water-rich, granite-like rocks such as granodiorite or tonalite, other studies have reported. That means
Earth cooled quickly enough for surface water and continental-type rocks just 100 million years after the
moon impact, the massive collision that formed the Earth-moon system.

51. Consider the following:


1. Brazil is the largest producer of high grade iron-ore in the world.
2. China is the biggest producer of copper mineral.
3. Bolivia is the world’s largest producer of diamond.

Select the correct answer using the codes below.


a) 1 only
b) 2 and 3 only
c) 1 and 3 only
d) 2 only

Solution: A
Justification: Statement 1: The world's largest producer of iron ore is the Brazilian mining corporation Vale,
followed by Anglo-Australian companies Rio Tinto Group and then BHP Billiton.
Statement 2: Chile is the top producer of copper. The second top producer is China, but the copper mining
industry here is nowhere near the production capacity of Chile.
Statement 3: Brazil and Bolivia are among the world’s largest producers of tin. South America also has large
deposits of gold, silver, zinc, chromium, manganese, bauxite, mica, platinum, asbestos and diamond.
Mineral oil is found in Venezuela, Argentina, Chile, Peru and Columbia.

52. Which of these rock formations of plains and young fold are most likely to contain non-metallic minerals?
a) Igneous
b) Metamorphic
c) Sedimentary
d) Any of the above depending on the rainfall in the region

Direct PDF - https://telegram.me/UPSC_PrelimsTest


https://telegram.me/UPSC_PDF

OGP 2019 GEOGRAPHY MODULE MCQS

Solution: C
Justification: Minerals occur in different types of rocks. Some are found in igneous rocks, some in
metamorphic rocks while others occur in sedimentary rocks. Generally, metallic minerals are found in igneous
and metamorphic rock formations that form large plateaus. Iron-ore in north Sweden is an example.
Sedimentary rock formations contain non-metallic minerals like limestone. Limestone deposits of Caucasus
region of France is an example. Mineral fuels such as coal and petroleum are also found in the sedimentary
strata.

GEOMORPHIC PROCESSES

53. Geological folds are undulation or waves in the stratified rocks of the Earth’s crust. They can be formed due
to
1. Hydrostatic and pore pressure
2. Temperature gradient on the surface of the earth
Which of the above is/are correct?
a) 1 only
b) 2 only
c) Both 1 and 2
d) None

Solution: C
Justification: Folds form under varied conditions of stress, hydrostatic pressure, pore pressure, and
temperature gradient, as evidenced by their presence in soft sediments.
Statement 1: Water pressure from underneath the rocks is tremendous and it can exert differential pressure
on the rocks to deform them from one or the other sides causing the formation of a fold.
Statement 2: Temperature gradient can cause elongation of one part and shortening of the other part leading
to formation of faults.

54. ‘Hot Spots’ within the earth help produce Geothermal Energy. What are these ‘Hot Spots’?
a) Areas of intense pressure inside the mantle
b) Region in crust where hot molten rocks are trapped
c) Regions of high volcanism on earth’s surface
d) Areas of intense magnetic activity within the upper mantle

Solution: B
Justification: Due to geological changes, molten rocks formed in the deeper hot regions of earth’s crust are
pushed upward and trapped in certain regions called ‘hot spots’. When underground water comes in contact
with the hot spot, steam is generated. Sometimes hot water from that region finds outlets at the surface.
Such outlets are known as hot springs. The steam trapped in rocks is routed through a pipe to a turbine and
used to generate electricity. The cost of production would not be much, but there are very few commercially
viable sites where such energy can be exploited. There are number of power plants based on geothermal
energy operational in New Zealand and United States of America.

LANDFORMS AND THEIR EVOLUTION

55. Conditions that favour the formation of deltas include


1. Coasts without shelters with a large tidal variation
2. Frequent water diversions from the main river that forms the delta
3. Presence of Continental shelf

Direct PDF - https://telegram.me/UPSC_PrelimsTest


https://telegram.me/UPSC_PDF

OGP 2019 GEOGRAPHY MODULE MCQS

Select the correct answer using the codes below.


a) 1 and 2 only
b) 3 only
c) 2 and 3 only
d) 1 and 3 only

Solution: B
Justification: Conditions Favourable for Delta Formation
 Active erosion of the river in its upper course to provide extensive gravel, sand and silt to be eventually
deposited as deltas.
 The coast should be sheltered preferably tideless, else delta will be washed away.
 There should be no strong current running at right angle to the river mouth, as it can wash away the
sediments.
 Any large lake in the way or river course can filter off the sediments, thus unfavourable for delta
formation.
 The sea should be shallow adjoining the delta as the sediments will disappear in the deep waters of the
sea. This is why a continental shelf is preferred.

56. Consider the following statements.


1. The work of river in the lower course is mainly Erosional rather than depositional due to the volume of
water being carried by it.
2. When the river flows normally its bed is raised through accumulation of deposits.
Which of the above is/are correct?
a) 1 only
b) 2 only
c) Both 1 and 2
d) None

Solution: B
Justification: Statement 1: River slows down in the lower course which allows the sediments to settle down.
This is opposite to the river’s action in the upper courses where the river mainly erodes.
Statement 2: As the debris or silt is deposited, it reduces the depth available for the water to flow, and thus
the bed is raised. Floodplains are created as a result of both erosion and deposition. Material is also
deposited on the sides forming raised banks called Levees. When the river reaches the sea, the fine materials
it has not dropped yet are deposited at its mouth, forming a fan shaped alluvial area called a delta.

57. Consider the following statements.


Assertion (A): Seamounts tend to be found on oceanic crust near mid-ocean ridges and island arcs.
Reason (R): Most seamounts are volcanic in origin.

In the context of the above, which of these is correct?


a) A is correct, and R is an appropriate explanation of A.
b) A is correct, but R is not an appropriate explanation of A.
c) A is correct, but R is incorrect.
d) Both A and R are incorrect.

Solution: A
Justification: You have read the concept of Mid-Oceanic ridges in 11th Standard Geography NCERT. They are
underwater mountain range, formed by plate tectonics. This uplifting of the ocean floor occurs when
convection currents rise in the mantle beneath the oceanic crust and create magma where two tectonic
plates meet at a divergent boundary. Seamounts are found close to these ridges. Seamounts can be found in
every ocean basin in the world, distributed extremely widely both in space and in age. A seamount is
technically defined as an isolated rise in elevation of 1,000 m or more from the surrounding seafloor, and

Direct PDF - https://telegram.me/UPSC_PrelimsTest


https://telegram.me/UPSC_PDF

OGP 2019 GEOGRAPHY MODULE MCQS

with a limited summit area, of conical form. Because of their abundance, seamounts are one of the most
common marine ecosystems in the world. Interactions between seamounts and underwater currents, as well
as their elevated position in the water, attract plankton, corals, fish, and marine mammals alike. Their
aggregational effect has been noted by the commercial fishing industry, and many seamounts support
extensive fisheries. There are ongoing concerns on the negative impact of fishing on seamount ecosystems

58. The fine dust blown beyond the desert limits is deposited on neighbouring lands as ‘X’. It is a yellow, friable
material and is usually very fertile. ‘X’ can be?
a) Seifs
b) Ventifacts
c) Zeugen
d) Loess

Solution: D
Justification: Loess is fine loam, rich in lime, very coherent and extremely porous. Water sinks in readily so
that the surface is always dry. Streams cut deep valleys through the thick mantle of soft loess and badland
topography may develop. It is so soft that roads constructed through a loess region soon sink and their walls
rise steeply. The most extensive deposit of loess is found in north-west China in the loess plateau of the
Hwang- Ho basin. It is estimated to cover an area of 250,000 square miles, and the deposits have
accumulated to a depth of 200 to 500 feet! In China, such yellowish wind-borne dust from the Gobi Desert is
called ‘Hwangtu’ — the yellow earth! But the original tern loess actually comes from a village in Alsace,
France bearing that name, where such deposits occurred.

59. Consider the following statements.


1. Limestone is a sedimentary rock.
2. Limestone increases the natural acidity of the soil.
3. The solubility of limestone in water and weak acid solutions leads to karst topography.
4. Limestones may contain the skeletal remains of corals and molluscs.
Select the correct answer using the codes below.
a) 1 and 2 only
b) 1, 3 and 4 only
c) 3 and 4 only
d) 1, 2 and 3 only

Solution: B
Justification: Statement 1 and 2: Limestone is a sedimentary rock, composed mainly of skeletal fragments of
marine organisms such as coral, forams and molluscs. Its major materials are the minerals calcite and
aragonite, which are different crystal forms of calcium carbonate (CaCO3). So, Limestone neutralizes the
natural acidity of the soil.
Statement 3 and 4: About 10% of sedimentary rocks are limestones. The solubility of limestone in water and
weak acid solutions leads to karst landscapes, in which water erodes the limestone over thousands to millions
of years. Most cave systems are through limestone bedrock. Limestone dominated areas have little
agricultural activities and are sparsely populated. Solubility of limestone makes it ineffective to grow major
crops. Moreover, the landforms made by limestone make an area difficult as a habitat for humans.
Limestone has numerous uses: as a building material, an essential component of concrete (Portland cement),
as aggregate for the base of roads, as white pigment or filler in products such as toothpaste or paints, as a
chemical feedstock for the production of lime, as a soil conditioner, or as a popular decorative addition to
rock gardens

60. Understanding Karst topography is important because


1. Nearly ten percent of the Earth's surface is occupied by karst landscape
2. Such topography occurs only in tropical and alpine environments.
3. Nearly a quarter of the world's population depends upon water supplied from karst areas

Direct PDF - https://telegram.me/UPSC_PrelimsTest


https://telegram.me/UPSC_PDF

OGP 2019 GEOGRAPHY MODULE MCQS

Select the correct answer using the codes below.

a) 1 and 2 only
b) 2 and 3 only
c) 1 and 3 only
d) None of the above

Solution: C
Justification: The term karst describes a distinctive topography that indicates dissolution (also called chemical
solution) of underlying soluble rocks by surface water or ground water. Although commonly associated with
carbonate rocks (limestone and dolomite) other highly soluble rocks such as evaporates (gypsum and rock
salt) can be sculpted into karst terrain.
Statement 2: Though most abundant in humid regions where carbonate rock is present, karst terrain occurs in
temperate, tropical, alpine and polar environments. Karst features range in scale from microscopic (chemical
precipitates) to entire drainage systems and ecosystems which cover hundreds of square miles, and broad
karst plateaus. Although karst processes sculpt beautiful landscapes, karst systems are very vulnerable to
ground water pollution due to the relatively rapid rate of water flow and the lack of a natural filtration
system. This puts local drinking water supplies at risk of being contaminated

61. Mid-Oceanic Ridges form an interconnected chain of mountain system within the ocean. It is characterised by
a central rift system at the crest. The rift system at the crest is the
a) Zone of maximum tectonic stability
b) Zone of intense volcanic activity
c) Plateau’s offshoot that has been stable over past some time
d) Zone of Wegner’s Oscillations

Solution: B
Justification: It is the longest mountain-chain on the surface of the earth though submerged under the
oceanic waters. It is characterised by a central rift system at the crest, a fractionated plateau and flank zone
all along its length. The rift system at the crest is the zone of intense volcanic activity. There is no such zone of
Wegner’s Oscillations.

62. Consider the following regions and their predominant geographical features:
1. Canadian Shield: Limestone rocks
2. Appalachians: Alternating ridgelines and valleys
3. Western Cordilleras: Intermontane basins and plateaus
4. Lake Superior: World’s largest salt water lake

Select the correct answer using the codes below.


a) 1 and 4 only
b) 2 and 3 only
c) 1 and 3 only
d) 2 and 4 only

Solution: B
Justification: Statement 1: It is a large area of exposed Precambrian igneous and high-grade metamorphic
rocks (geological shield) that forms the ancient geological core of the North American continent. Composed
of igneous rock resulting from its long volcanic history, the area is covered by a thin layer of soil.
Statement 2: They are a system of mountains in eastern North America. The Appalachian chain is a barrier to
east-west travel, as it forms a series of alternating ridgelines and valleys oriented in opposition to most roads
running east or west.

Direct PDF - https://telegram.me/UPSC_PrelimsTest


https://telegram.me/UPSC_PDF

OGP 2019 GEOGRAPHY MODULE MCQS

Statement 3: The North American Cordillera covers an extensive area of mountain ranges, intermontane
basins, and plateaus in western North America, including much of the territory west of the Great Plains. It is
also sometimes called the Western Cordillera.
Statement 4: Lake Superior is the largest of the Great Lakes of North America. It is generally considered the
largest freshwater lake in the world by surface area. It is the world's third-largest freshwater lake by volume
and the largest by volume in North America

63. Which of these geographical regions are known as sub-continents?


1. Indian subcontinent
2. Arabian Peninsula
3. Greenland
4. Alaskan peninsula
Select the correct answer using the codes below.
a) 1 and 2 only
b) 1, 3 and 4 only
c) 2 and 3 only
d) D. 1, 2, 3 and 4

Solution: D
Justification: Certain parts of continents are recognized as subcontinents, especially the large peninsulas
separated from the main continental landmass by geographical features. The most notable examples are the
Indian subcontinent and the Arabian Peninsula. The southern cone of South America and Alaskan peninsula of
North America are other examples. In many of these cases, the "subcontinents" concerned are on different
tectonic plates from the rest of the continent, providing a geological justification for the terminology.
Greenland, generally reckoned as the world's largest island on the north-eastern periphery of the North
American Plate, is sometimes referred to as a subcontinent. This is a significant departure from the more
conventional view of a subcontinent as comprising a very large peninsula on the fringe of a continent.

64. Which of these geographical features is most likely to have the lowest mean elevation?

a) Plateaus
b) Islands
c) Deltas
d) Plains

Solution: C
Justification: Option A: They are elevated land tops, and maintain a rugged high profile of terrain.
Option B: Islands although situated at sea level can acquire higher elevation due to presence of features like
mountains, volcanoes etc.
Option C: Deltas occur near the sea mouth, i.e. very close to Mean sea level, which can be the lowest
elevation possible.
Option D: Rivers flow further from plains to join the sea. So, plains are usually at a higher elevation than delta.

65. Consider the following matches of mountains or mountain ranges with the region they are found in.
1. Mount Kosciuszko: Europe
2. Appalachian Mountains: North America
3. Atlas Mountains: Asia
4. Ural Mountains: Russia

Select the correct answer using the codes below.


a) 2 and 3 only
b) 1 and 4 only
c) 2 and 4 only

Direct PDF - https://telegram.me/UPSC_PrelimsTest


https://telegram.me/UPSC_PDF

OGP 2019 GEOGRAPHY MODULE MCQS

d) D. 1, 2, 3 and 4

Solution: C
Justification: Statement 1: It is the highest mountain of Australia located in New South Wales.
Statement 2: They are the great highland system of North America, the eastern counterpart of the Rocky
Mountains.
Statement 3: They are in North Africa.
Statement 4: They run across Western Russia, and form the border of Asia with Europe.

66. Acute mountain sickness or altitude sickness is usually caused due to


1. Increase in air pressure and density
2. Low levels of oxygen at high altitudes

Which of the above is/are correct?


a) 1 only
b) 2 only
c) Both 1 and 2
d) None

Solution: B
Justification: Statement 1: The density of the atmosphere varies with height. It is maximum at the sea level
and decreases rapidly as we go up, so does the air pressure.
Statement 2: Higher altitudes have lower levels of oxygen and decreased air pressure. When you travel in a
plane, drive or hike up a mountain, or go skiing, your body may not have enough time to adjust. This can
result in acute mountain sickness. So, many have to carry with them oxygen cylinders to be able to breathe at
high altitudes.

67. What is a catcher beach?

a) A beach with low shorelines


b) A tidal estuary with no opening mouth
c) A place where marine debris tends to pile up or aggregate
d) A beach devoid of mangroves and corals

Solution: C
Justification: A catcher beach should not be confused with a dumping ground or heavily trashed public
beach. A catcher beach typically receives its accumulations of debris due to its shape and location in
combination with high-energy waves, storms, or winds. A specific example of a catcher beach can be found
along the shores of Gore Point, Alaska. The geography of this location makes it a very high-density catcher
beach, as it sticks out like a hook into the Gulf of Alaska current.

CLIMATE

68. All changes in the weather are ultimately caused by the


a) Rotation of Earth
b) Energy of the Sun
c) Hydrological cycle on earth
d) Primordial heat inside the earth

Solution: B

Direct PDF - https://telegram.me/UPSC_PrelimsTest


https://telegram.me/UPSC_PDF

OGP 2019 GEOGRAPHY MODULE MCQS

Justification: Option A Rotation of earth does cause wind movements; change of day and night; distribution
of heat on earth etc. However, it does not explain several other phenomena such as seasons on earth;
extreme heat and cold in Poles etc.
Option B Sun’s energy causes all these, and is ultimately responsible for life and activity on earth, which also
subsumes weather phenomena.
Option C Earth’s hydrological cycle is only partly responsible for the weather. For e.g. rainfall pattern,
movements of ocean water etc.
Option D Primordial heat inside the earth manifests itself in form of moving molten magma inside the earth.
This however is responsible for tectonic and geo-morphological processes, not weather phenomenon.
Learning: sun is the primary source of energy that causes changes in the weather. Energy absorbed and
reflected by the earth’s surface, oceans and the atmosphere play important roles in determining the weather
at any place. Heat from Sun causes changes in temperature; pressure; evaporation; biological activity etc. All
these in turn determine the weather at a place.

COMPOSITION AND STRUCTURE OF ATMOSPHERE

69. Earth’s atmosphere consists of several layers. Consider the following statements with reference to it.
Assertion (A): Troposphere is the hottest layer of the atmosphere.
Reason (R): It gets more heat radiation from below, earth’s surface, as compared to other atmospheric layers.
In the context of the above, which of these is correct?

a) A is correct, and R is an appropriate explanation of A.


b) A is correct, but R is not an appropriate explanation of A.
c) A is correct, but R is incorrect.
d) Both A and R are incorrect.

Solution: A
Justification: The troposphere is the lowest layer of Earth's atmosphere. The troposphere is heated from
below. Sunlight warms the ground or ocean, which in turn radiates the heat into the air right above it. This
warm air tends to rise. That keeps the air in the troposphere "stirred up". Air is warmest at the bottom of the
troposphere near ground level. Higher up it gets colder. Nearly all of the water vapour and dust particles in
the atmosphere are in the troposphere. That is why most clouds are found in this lowest layer, too. The
thickness of the troposphere varies around the planet.

70. Troposphere is thickest at


a) Poles
b) Equator
c) Sub-tropics
d) Temperate regions

Solution: B
Justification: The troposphere is thicker at the equator than at the poles because the equator is warmer. The
convection currents of air expand the thickness of the troposphere (atmosphere) at poles. Thus the simple
reason is thermal expansion of the atmosphere at the equator and thermal contraction near the poles. Also,
the rotation of the earth causes centrifugal force which is strongest near the equator and pushes the
atmosphere to greater heights. The thickness of the troposphere also varies with season. The troposphere is
thicker in the summer and thinner in the winter all around the planet. At the poles in winter, the atmosphere
is uniformly very cold and the troposphere cannot be distinguished from other layers.

71. The Northern Lights are the result of


1. Collisions between gaseous particles in the Earth's atmosphere with charged particles released from the
sun's atmosphere

Direct PDF - https://telegram.me/UPSC_PrelimsTest


https://telegram.me/UPSC_PDF

OGP 2019 GEOGRAPHY MODULE MCQS

2. Interaction of volcanic soot with the air mass movements in the stratosphere
3. High speed discharge of radio waves near the ionosphere

Select the correct answer using the codes below.


a) 1 only
b) 2 and 3 only
c) 3 only
d) 2 only

Solution: A
Justification: Blown towards the earth by the solar wind, the charged particles are largely deflected by the
earth's magnetic field. However, the earth's magnetic field is weaker at either pole and therefore some
particles enter the earth's atmosphere and collide with gas particles. These collisions emit light that we
perceive as the dancing lights of the north (and the south).

72. Nitrogen is a major constituent of the atmosphere comprising about seventy-nine per cent of the
atmospheric gases. Consider the following about it.
1. It is an essential constituent of organic compounds such as the amino acids and protein.
2. The principal source of free nitrogen is the action of soil micro-organisms and associated plant roots on
atmospheric nitrogen.

Which of the above is/are correct?


a) 1 only
b) 2 only
c) Both 1 and 2
d) None

Solution: C
Justification: Statement 1: These organic compounds such as the amino acids, nucleic acids, proteins,
vitamins and pigments.
Statement 2: Generally, nitrogen is usable only after it is fixed. Ninety per cent of fixed nitrogen is biological.
Only a few types of organisms like certain species of soil bacteria and blue green algae are capable of utilising
it directly in its gaseous form.

SOLAR RADIATION, HEAT BALANCE AND TEMPERATURE

73. The amount of solar energy received by the earth varies according to latitude mainly due to
a) Distribution of Ozone in stratosphere
b) Wind flow in troposphere
c) Global movement of ocean Currents
d) Earth’s curvature

Solution: D
Justification: There is a difference between the solar energy received and solar energy circulated (which
affects the temperature of a region). All the factors from options A to C only circulate the solar energy that is
already received by the earth; they do not as such directly affect the solar energy incident on earth. There is a
minor case for the cloudiness created by movement of ocean currents or winds that often blocks solar
insolation; but the major cause for variation is earth’s curvature.

74. Despite Sun being overhead throughout the year at the equator, it receives comparatively less insolation than
the tropics. This can be due to
1. Heavy cloudiness due to continuous evaporation that reduces the amount of solar insolation

Direct PDF - https://telegram.me/UPSC_PrelimsTest


https://telegram.me/UPSC_PDF

OGP 2019 GEOGRAPHY MODULE MCQS

2. Absence of Coriolis force at the equator


Which of the above is/are correct?
a) 1 only
b) 2 only
c) Both 1 and 2
d) None

Solution: A
Justification: Statement 1: Generally, at the same latitude the insolation is more over the continent than over
the oceans, because oceans are cloudier due to greater evaporation. Similarly in equator, evaporation is high,
cloudiness is high and rains are frequent. Clouds reflect incoming solar insolation and send it back to space.
Statement 2: Zero coriolis force would mean that winds are not deflected with great force at the equator, as
they are the sub-tropics. But, direction of wind will only affect the distribution of heat, and not the amount of
insolation actually received from the Sun.

75. Temperature inversion is most common in


a) Coastal regions
b) Mountain valleys
c) Plains
d) Equatorial forests

Solution: B
Justification: Usually, within the lower atmosphere (the troposphere) the air near the surface of the Earth is
warmer than the air above it, largely because the atmosphere is heated from below as solar radiation warms
the Earth's surface, which in turn then warms the layer of the atmosphere directly above it. Given the right
conditions, the normal vertical temperature gradient is inverted such that the air is colder near the surface of
the Earth. This can occur when, for example, a warmer, less-dense air mass moves over a cooler, denser air
mass. It can also happen in valleys are the cold air slides down the mountain slope and warm air moves up.

76. Factors that affect temperature at a particular region include


1. Circulation of planetary and local winds

Direct PDF - https://telegram.me/UPSC_PrelimsTest


https://telegram.me/UPSC_PDF

OGP 2019 GEOGRAPHY MODULE MCQS

2. Altitude and terrain of the place


3. Distance of the region from poles or equator
4. Movement of Ocean waves
Select the correct answer using the codes below.
a) 2 and 3 only
b) 1, 2 and 3 only
c) 1 and 4 only
d) D. 1, 2, 3 and 4

Solution: B
Justification: Ocean Waves do not induce movement of water in the ocean from one place to another.
Movement of water is done by ocean currents. If water is not moved from one place to the other, there is no
circulation of temperature, nutrients, water density or salinity. So, ocean waves affect the temperature
distribution the least. Ocean ‘currents’ do. Ocean currents, warm or cold, carry heat/coolness from one sea
region to the other and affect the temperature distribution.

77. Generally, at the same latitude the insolation is more over the continent than over the oceans, because
1. Oceans are cloudier
2. Continents have varied relief
Which of the above is/are correct?
a) 1 only
b) 2 only
c) Both 1 and 2
d) None

Solution: A
Justification: Statement 1: This is due to greater evaporation. Similarly, in equator, evaporation is high,
cloudiness is high and rains are frequent. Clouds reflect incoming solar insolation and send it back to space.
Statement 2: Despite the varied terrain and relief, amount of insolation received by land isn’t affected so
significantly as to cause such a difference in the total insolation received by land and seas.

78. The earth as a whole neither accumulates heat nor loses it due to solar insolation because
a) Circulation of magma inside the earth and primordial heat maintain the temperature of the earth.
b) The earth in turn radiates back solar insolation received from the sun.
c) Radioactive decay in the core regulates surface temperature.
d) There is a strong conventional wind current blowing in the earth’s atmosphere.

Solution: B
Justification: The earth receives almost all of its energy from the sun. The earth in turn radiates back to space
the energy received from the sun. As a result, the earth neither warms up nor does it get cooled over a period
of time

Direct PDF - https://telegram.me/UPSC_PrelimsTest


https://telegram.me/UPSC_PDF

OGP 2019 GEOGRAPHY MODULE MCQS

79. Out of the total solar insolation that reaches the earth’s surface, most is used by plants for

a) Respiration
b) Photosynthesis
c) Storage
d) Movement of minerals and fluids

Solution: A
Justification: Only a very small fraction (0.1 per cent) is fixed in photosynthesis. More than half is used for
plant respiration and the remaining part is temporarily stored or is shifted to other portions of the plant. This
energy captured from Sun further reduces when we pass to higher trophic levels.

ATMOSPHERIC CIRCULATION AND WEATHER SYSTEMS

80. Trade winds blowing counter-clockwise in the Northern Hemisphere and blowing clockwise in the Southern
Hemisphere is mainly due to
a) Presence of more number of cyclones in the Northern Hemisphere
b) Pronounced land effect or continentality in the Northern Hemisphere
c) Rotation of the earth on its own axis
d) Thickness of the atmosphere over the equator that produces a reverse jet stream in both the hemispheres

Solution: C
Justification: The opposite direction of circulation is due to the Coriolis Effect. The surface air that flows from
these subtropical high-pressure belts toward the Equator is deflected toward the west in both hemispheres
by the Coriolis Effect. These winds blow predominantly from the northeast in the Northern Hemisphere and
from the southeast in the Southern Hemisphere.
Option A: Tropical cyclones typically form over large bodies of relatively warm water, so they are equally
predominant in both the hemispheres wherever there are conducive conditions.

81. Horse latitudes are characterized by


1. Calm winds
2. Cloudy skies throughout the year

Direct PDF - https://telegram.me/UPSC_PrelimsTest


https://telegram.me/UPSC_PDF

OGP 2019 GEOGRAPHY MODULE MCQS

3. Precipitation equivalent to the equatorial regions


Select the correct answer using the codes below.
a) 1 and 2 only
b) 2 and 3 only
c) 1 only
d) 1 and 3 only

Solution: C
Justification: The horse latitudes are located at about 30 degrees north and south of the equator. It is
common in this region of the subtropics for winds to diverge and either flow toward the poles (known as the
prevailing westerlies) or toward the equator (known as the trade winds). These diverging winds are the result
of an area of high pressure, which is characterized by calm winds, sunny skies, and little or no precipitation.
According to legend, the term comes from ships sailing to the New World that would often become stalled for
days or even weeks when they encountered areas of high pressure and calm winds. Many of these ships
carried horses to the Americas as part of their cargo. Unable to sail and resupply due to lack of wind, crews
often ran out of drinking water.

82. Which of the following characterize Chinook winds?


1. Dry winds
2. Harbinger of Snow
3. Easterly wind
Select the correct answer using the codes below.
a) 1 only
b) 2 and 3 only
c) 2 only
d) 3 only

Solution: A
Justification: The chinook, a native word meaning "snow eater," belongs to a family of winds experienced in
many parts of the world where long mountain chains lie more or less at right angles to the prevailing wind. It
is a warm and dry westerly wind that blows down the Rocky Mountains into the mountains' eastern slopes
and the western prairies. The Chinook wind provides a welcome respite from the long winter chill.

83. The "doldrums" is a popular nautical term that refers to the belt around the Earth near the equator where
sailing ships sometimes get stuck on windless waters. The windless waters are associated with
1. Intense solar heating near the equator
2. Absence of trade winds at the equator
3. The relative absence of landmass at the equator compared to the Northern hemisphere
Select the correct answer using the codes below.
a) 1 only
b) 2 and 3 only
c) 1 and 3 only
d) 1 and 2 only

Solution: A
Justification: Statement 3: The excess or absence of Landmass does not seem to have a bearing on the
windlessness of this region which is more dependent on the culmination of trade winds.
Statement 1: Known to sailors around the world as the doldrums, the Inter-Tropical Convergence Zone,
(ITCZ), is a belt around the Earth extending approximately five degrees north and south of the equator. Here,
the prevailing trade winds of the northern hemisphere blow to the southwest and collide with the southern
hemisphere’s driving northeast trade winds. Due to intense solar heating near the equator, the warm, moist
air is forced up into the atmosphere like a hot air balloon. As the air rises, it cools, causing persistent bands of
showers and storms around the Earth’s midsection. The rising air mass finally subsides in what is known as

Direct PDF - https://telegram.me/UPSC_PrelimsTest


https://telegram.me/UPSC_PDF

OGP 2019 GEOGRAPHY MODULE MCQS

the horse latitudes, where the air moves downward toward Earth’s surface. Because the air circulates in an
upward direction, there is often little surface wind in the ITCZ. That is why sailors well know that the area can
becalm sailing ships for weeks. And that’s why they call it the doldrums.

84. Consider the following statements.


1. Low pressure systems are usually characterized by dry and settled weather.
2. High pressure systems usually bring wind and precipitation.
Which of the above is/are correct?
a) 1 only
b) 2 only
c) Both 1 and 2
d) None

Solution: D
Justification: Statement 1: Low pressure systems attract wind from high pressure areas nearby, sometimes
even forming cyclonic depressions.

85. Consider the following about South-East trade Winds.

Direct PDF - https://telegram.me/UPSC_PrelimsTest


https://telegram.me/UPSC_PDF

OGP 2019 GEOGRAPHY MODULE MCQS

1. They blow mainly in the Northern hemisphere near the equator.


2. The winds are deflected toward the East by the Coriolis Effect
Which of the above is/are correct?
a) 1 only
b) 2 only
c) Both 1 and 2
d) None

Solution: D
Justification:

South-East trade Winds blow in the Southern Hemisphere. North-east trade winds flow in Northern
Hemisphere. The surface air that flows from these subtropical high-pressure belts toward the Equator is
deflected toward the west in both hemispheres by the Coriolis Effect. These winds blow predominantly from
the northeast in the Northern Hemisphere and from the southeast in the Southern Hemisphere. Because
winds are named for the direction from which the wind is blowing, these winds are called the north-easterly
trade winds in the Northern Hemisphere and the south-easterly trade winds in the Southern Hemisphere. The
trade winds of both hemispheres meet at the doldrums.

86. Katabatic winds occur due to


a) Density differences in wind at different altitudes
b) Differential warming of the ocean surface near coasts
c) Local thunderstorms guided by high convection rates
d) Seasonal land breeze in the interior of continents

Solution: A
Justification: These are downslope winds flowing from high elevations of mountains, plateaus, and hills
down their slopes to the valleys or planes below. A katabatic wind originates from radiational cooling of air
atop a plateau, a mountain, glacier, or even a hill. Since the density of air is inversely proportional to
temperature, the air will flow downwards, warming approximately adiabatically as it descends. The
temperature of the air depends on the temperature in the source region and the amount of descent.

Direct PDF - https://telegram.me/UPSC_PrelimsTest


https://telegram.me/UPSC_PDF

OGP 2019 GEOGRAPHY MODULE MCQS

87. If two places on a map fall on the same isochrone line, it implies that
a) They are situated at similar magnetic latitude.
b) They are located on similar terrains.
c) They take equal amount of time to reach from a particular location.
d) They experience the same weather conditions at the same time in a year.

Solution: C
Justification: In selecting the mode of transport, distance, in terms of time or cost, is the determining factor.
Isochrone lines are drawn on a map to joins places equal in terms of the time taken to reach them. Based on
these lines, an appropriate transportation mode is chosen.

88. Known to sailors around the world as the ‘doldrums’, these (are)
1. Exist at the edge of the tropics
2. Also known as the Inter-Tropical Convergence Zone
3. Popular for high wind speeds throughout the year
Select the correct answer using the codes below.
a) 1 only
b) 2 only
c) 1 and 3 only
d) 2 and 3 only

Solution: B
Justification: Statement 1 and 2: It is a belt around the Earth extending approximately five degrees north and
south of the equator. Here, the prevailing trade winds of the northern hemisphere blow to the southwest and
collide with the southern hemisphere’s driving northeast trade winds.
Statement 3: Due to intense solar heating near the equator, the warm, moist air is forced up into the
atmosphere like a hot air balloon. As the air rises, it cools, causing persistent bands of showers and storms
around the Earth’s midsection. The rising air mass finally subsides in what is known as the horse latitudes,
where the air moves downward toward Earth’s surface.
Because the air circulates in an upward direction, there is often little surface wind in the ITCZ. That is why
sailors well know that the area can becalm sailing ships for weeks. And that’s why they call it the doldrums.

89. Coriolis force plays a role in which of the following?


1. Formation of meanders
2. Direction of trade winds
3. Directing Jet Streams
4. Impact crater
Select the correct answer using the codes below.
a) 1, 2 and 4 only
b) 3 only
c) 1, 2 and 3 only
d) 2 only

Solution: C
Justification: Statement 1: Coriolis force acting on the fluid water deflecting it like it deflects the wind. This
causes rivers to ‘meander’.
Statement 2: It deflects the wind to the right direction in the northern hemisphere and to the left in the
southern hemisphere.
Statement 3: Jet streams typically cover long distances and can be thousands of miles long. They can be
discontinuous and often meander across the atmosphere. The meanders in the jet stream flow slower than
the rest of the air and are called Rossby Waves. They move slower because they are caused by the Coriolis
Effect and turn west in respect to the flow of air they are embedded in.

Direct PDF - https://telegram.me/UPSC_PrelimsTest


https://telegram.me/UPSC_PDF

OGP 2019 GEOGRAPHY MODULE MCQS

Statement 4: It is a circular depression in the surface of a planet, moon, or other solid body in the Solar
System. Hence incorrect.
90. Consider the following about Sand and dust storms across the world.
1. They can travel thousands of kilometres across continents and oceans.
2. Chronic exposure to fine dust contributes to premature deaths from respiratory and cardiovascular
diseases.
3. Deforestation and unsustainable agricultural practices such as excessive withdrawal of water from water
bodies can lead to sand and dust storms.
Select the correct answer using the codes below.
a) 2 only
b) 1, 2 and 3
c) 2 and 3 only
d) 1 and 3 only

Solution: B
Justification: S1: They result from strong turbulent winds eroding sand, silt and clay particles from arid
landscapes and impoverishing their soils. The storms can raise large quantities of particles high into the air.
They can travel thousands of kilometres across continents and oceans, entraining other pollutants on the way
and depositing particles far from their origin.
S2: Dust harms both humans and animals. Chronic exposure to fine dust contributes to premature deaths
from respiratory and cardiovascular diseases, lung cancer and acute lower respiratory infections.
S3: The anthropogenic causes of sand and dust storms include deforestation and unsustainable agricultural
practices as well as excessive water extraction and the modification of water bodies for irrigation and other
purposes. Sand and dust storms are thus connected with a range of environmental and development issues
that extend across national, regional, and continental boundaries.

91. What is/are the consequence(s) of the blowing of Chinook winds in America?
1. Vegetation and crops freeze as temperature is lowered.
2. Onset of monsoonal winds in the region causing rainfall sporadically in short intervals of time

Which of the above is/are correct?


a) 1 only
b) 2 only
c) Both 1 and 2
d) None

Solution: D
Justification: It is a warm and dry westerly wind that blows down the Rocky Mountains into the mountains'
eastern slopes and the western prairies. It is a hot wind that blows in winter and therefore raises the
temperature in a short time. This increase in temperature results in the melting of snow, making pasture land
available for grazing of animals.

92. What are the horse latitudes?

a) Subtropical regions known for calm winds and little precipitation


b) Temperate regions known for intense cyclones
c) Polar latitudes that are known for hosting the coldest currents on the planet
d) Equatorial region that are famous for reverse currents

Solution: A
Justification: The horse latitudes are located at about 30 degrees north and south of the equator. It is
common in this region of the subtropics for winds to diverge and either flow toward the poles (known as the

Direct PDF - https://telegram.me/UPSC_PrelimsTest


https://telegram.me/UPSC_PDF

OGP 2019 GEOGRAPHY MODULE MCQS

prevailing westerlies) or toward the equator (known as the trade winds). These diverging winds are the result
of an area of high pressure, which is characterized by calm winds, sunny skies and little or no precipitation.

WATER IN THE ATMOSPHERE

93. Consider the following statements.


Assertion (A): The eastern coasts of continents within the tropics have much heavier rainfall than the
interiors or the west coasts.
Reason (R): All western coasts fall in the rain shadow zone.
In the context of the above, which of these is correct?
a) A is correct, and R is an appropriate explanation of A.
b) A is correct, but R is not an appropriate explanation of A.
c) A is correct, but R is incorrect.
d) Both A and R are incorrect.

Solution: C
Justification: Islands and coastal areas 10° to 20° north or south of the equator usually have a tropical marine
climate. The ocean is the main influence in creating the tropical marine climate. The temperature ranges from
25°C to 35°C. The trade winds blow all year round. The trade winds are moist, as they have passed over warm
seas. Since they are easterlies, they cause greater precipitation on the eastern coasts and run dry on the
western coasts and interiors.

94. Which of the following factors help in the formation of fog over the sea?
1. Tropical winds passing over cooler waters
2. Presence of condensation nuclei in the form of Ocean salt
Which of the above is/are correct?
a) 1 only
b) 2 only
c) Both 1 and 2
d) None

Solution: C
Justification: Statement 1: When wind blows moist air over a cool surface the air will cool and advection fog
will form. Advection fog is very common at sea when tropical winds pass over cooler waters and on land
when a warm front passes over heavy snow.
Statement 2: Sea fog forms when the condensation nuclei is salt. Salt is, of course, very common near the
ocean where it is kicked up into the atmosphere by the breaking waves. Salt is a unique condensation nuclei
in that it will allow fog to form when the humidity is as low as 70%.

95. In percentage terms, water vapour in the atmosphere, generally


1. Increases as we move to higher grounds
2. Decreases towards the poles if we start from equator
Which of the above is/are correct?
a) 1 only
b) 2 only
c) Both 1 and 2
d) None

Solution: B
Justification: Water vapour depends on both temperature and precipitation. In the warm and wet tropics, it
may account for four per cent of the air by volume, while in the dry and cold areas of desert and Polar
Regions, it may be less than one per cent of the air. Since polar regions have a cold climate and poor

Direct PDF - https://telegram.me/UPSC_PrelimsTest


https://telegram.me/UPSC_PDF

OGP 2019 GEOGRAPHY MODULE MCQS

precipitation, the capacity of the air as well as the moisture available as lesser, leading to a lower water
vapour content.

96. Rainfall generally increases as we proceed from


a) Lower to higher altitude
b) Coastal areas to interior areas
c) Poles to equator
d) All of the above

Solution: C
Justification: Option A: Nothing concrete can be said about the pattern here. It depends on the latitude and
surrounding terrain. For e.g. for similar altitudes, ladakh and NE states would witness different precipitation.
Option B: Coastal areas higher evaporation, moisture and cloud formation than interior areas. Moreover, any
moisture-laden winds from ocean will first hit the coastal areas and then interior areas. Moisture would go on
reducing as the wind advance further in. So, B is incorrect.
Option C: Poles receive lesser solar insolation, have lesser evaporation and rainfall. Reverse is true for
equator.

97. Rain clouds appear black in colour despite having the Sun above them because
1. Clouds accumulate electrostatic charge
2. Rain-bearing clouds absorb most of the solar insolation falling on them.
3. Clouds scatter light received by them.
Select the correct answer using the codes below.
a) 1 only
b) 1 and 2 only
c) 1 and 3 only
d) 3 only

Solution: D
Justification: The colour of any object depends on its ability to reflect selected wavelengths of light.
Consequently, if the object does not reflect any colour, it appears black. The tiny droplets of water in rain
clouds scatter the white light of the sun. Red is scattered first, and violet the last. This happens at such a high
altitude that no rays of the sun reach the bottom of the cloud. Thus, we do not see the rays of the sun, and
this results in a dark effect. As normal clouds have no water, the light is not scattered and thus look white.

98. Generally, the capacity of air to hold moisture in the form of water vapour is likely to be greater in
a) Coastal regions
b) Tropical rainforests
c) Hot sub-tropical deserts
d) Cold Tundra regions

Solution: C
Justification: Water vapour capacity increases with temperature increase. At low temperatures, such as in
Tundra regions, the air will hold very less moisture as it becomes more compact. So, (d) is wrong.
Coastal regions generally have moderate temperatures and the air is already has a lot of moisture. So, (a)
can’t be the answer. The case of option (b) is also similar due to high rainfall and not so high temperatures.
In, hot sub-tropical deserts, you will notice that evaporation is very fast, as the air can contain a lot of water
vapour due to the aridity and high temperature. So, (a) is the answer.

Direct PDF - https://telegram.me/UPSC_PrelimsTest


https://telegram.me/UPSC_PDF

OGP 2019 GEOGRAPHY MODULE MCQS

WORLD CLIMATE AND CLIMATE CHANGE

99. EI-Nino is a complex weather system that appears once every three to seven years. In the Southern and
Central Pacific Ocean, it results in
1. Distortion of equatorial atmospheric circulation
2. Irregularities in the evaporation of sea water
3. Increase in the amount of diatoms in the ocean
Select the correct answer using the codes below.
a) 1 and 2 only
b) 3 only
c) 1 and 3 only
d) 1, 2 and 3

Solution: A
Justification: Statement 1 and 2: EI-Nino is merely an extension of the warm equatorial current which gets
replaced temporarily by cold Peruvian current or Humboldt current (locate these currents in your atlas). The
system involves oceanic and atmospheric phenomena with the appearance of warm currents off the coast of
Peru in the Eastern Pacific and affects weather in many places including India. This current increases the
temperature of water on the Peruvian coast by 10°C. This affects the local temperature, and thus ocean
currents, wind pattern leading to rainfall changes. It brings drought, floods and other weather extremes to
different parts of the world. EI-Nino is used in India for forecasting long range monsoon rainfall.
Statement 3: It actually reduces the amount of phytoplankton and also diatoms because of the increase in the
sea temperature. Planktons need a more ambient climate to survive.

100. There is a certain definite pattern to the location of the world’s deserts. Almost all the deserts are
confined within the
a) 15 to 30 degrees parallels of latitude north and south of the equator
b) 30 to 45 degrees parallels of latitude north and south of the equator
c) Above 30 degrees parallels of latitude north of the equator
d) Above 30 degrees parallels of latitude south of the equator

Solution: A
Justification: They lie in the trade wind belt on the western parts of the continents where Trade winds are
off-shore. They are bathed by cold currents which produced a ‘desiccating effect’ so that moisture is not
easily condensed into precipitation. Dryness or aridity is the key note. Such deserts are tropical hot deserts or
‘Trade wind deserts’. They include the Great Sahara Desert; Arabian, Iranian and Thar Deserts; Kalahari,
Namib, and Atacama Deserts; the Great Australian Deserts and the deserts of the south-west U.S.A, and
northern Mexico. In the continental interiors of the mid-latitudes, the deserts such as the Gobi and Turkestan
are characterised by extremes of temperatures.

101. Consider the following statements. El Nino Modoki


1. is a Tropospheric phenomenon congruent with the occurrence of El- Nino on the hydrosphere
2. Affects the land temperature rather than the sea temperature as in the case with El Nino
Which of the above is/are correct?
a) 1 only
b) 2 only
c) Both 1 and 2
d) None

Solution: D
Justification: El Nino Modoki is a coupled ocean-atmosphere phenomenon in the tropical Pacific. It is
different from another coupled phenomenon in the tropical Pacific namely, El Nino. Conventional El Nino is
characterized by strong anomalous warming in the eastern equatorial Pacific. However, El Nino Modoki is

Direct PDF - https://telegram.me/UPSC_PrelimsTest


https://telegram.me/UPSC_PDF

OGP 2019 GEOGRAPHY MODULE MCQS

associated with strong anomalous warming in the central tropical Pacific and cooling in the eastern and
western tropical Pacific. Associated with this distinct warming and cooling patterns the teleconnections are
very different from teleconnection patterns of the conventional El Nino. The El Niño Modoki significantly
influences regional climates in China, Japan and US

102. At equatorial regions temperature is usually observed to be moderate despite the high solar insolation.
This can be due to
a) Presence of cold ocean currents
b) Blowing of North-easterly trade winds
c) Katabatic winds
d) Cloudiness and heavy precipitation

Solution: D
Justification: Cloudiness and heavy precipitation around 150 – 250 cm of rainfall or more in a year, helps to
moderate the temperature, so that even at the equator, climate is not unbearable. Cloudiness reduces solar
insolation, and precipitation lowers the temperature even from the consistent yearlong solar insolation that
is received at the equator.

103. The Savanna climate is


1. A transitional type of climate between the equatorial forest and the hot deserts
2. Confined within the tropics with distinct wet and dry seasons
Which of the above is/are correct?
a) 1 only
b) 2 only
c) Both 1 and 2
d) None
Solution: C
Justification: It is a transitional type of climate between the equatorial forest and the trade wind hot deserts.
It is confined within the tropics and is best developed in the Sudan where the dry and wet seasons are most
distinct, hence its name the Sudan climate. The belt includes West African Sudan, and then curves southward
into east Africa and southern Africa north of the tropic of Capricorn. In South America, there are two distinct
regions of savannah north and south of the equator

104. As you move from Taiga to Tundra region


1. The average vegetation height reduces
2. Depth of permafrost decreases
3. Vegetation becomes more scattered
Select the correct answer using the codes below.
a) 1 only
b) 1 and 3 only
c) 2 and 3 only
d) 2 only
Solution: B
Justification: The diagram from GC Leong shows the general trend. The causal factors are of course lower
temperature, absence of fertile soil and poor precipitation

Direct PDF - https://telegram.me/UPSC_PrelimsTest


https://telegram.me/UPSC_PDF

OGP 2019 GEOGRAPHY MODULE MCQS

105. The total precipitation of Siberian climate is decided by


1. Proximity to the Poles
2. Temperate monsoons
3. Penetration of cyclones
Select the correct answer using the codes below.
a) 1 only
b) 1 and 3 only
c) 2 and 3 only
d) 1, 2 and 3

Solution: D
Justification: Statement 1: Factors such as exposure to westerlies (on western part of continents), temperate
monsoons (on the eastern side) and cyclones affect rainfall considerably. Eastern Siberia also has over 20
inches, being moistened by the SE Monsoon from the Pacific Ocean. The cold, dry air of North is incapable of
holding moisture, and in the South are semi-arid steppes.

106. Regions with cool temperate British type climate can be found in
1. Eastern United States
2. Southern Chile
3. South Africa
4. East Asia

Select the correct answer using the codes below.


a) 1 and 2 only
b) 2 only
c) 3 and 4 only
d) 2 and 4 only

Solution: B
Justification:

Direct PDF - https://telegram.me/UPSC_PrelimsTest


https://telegram.me/UPSC_PDF

OGP 2019 GEOGRAPHY MODULE MCQS

107. Grape cultivation is a speciality of the Mediterranean region. What climatic conditions in the
Mediterranean region support this?
1. Long growing seasons of moderate to warm temperatures
2. Very low precipitation in grape growing seasons
Which of the above is correct?
a) 1 only
b) 2 only
c) Both 1 and 2
d) None

Solution: A
Justification: The presence of adequate sun, heat and water are all vital to the healthy growth and
development of grapevines during the growing season. In general, grapevines thrive in temperate climates
which grant the vines long, warm periods during the crucial flowering, fruit set and ripening periods. On
average, a grapevine needs around 710 mm (28 in) of water for sustenance during the growing season, not all
of which may be provided by natural rain fall. In Mediterranean and many continental climates, the climate
during the growing season may be quite dry and require additional irrigation. So, 2 is incorrect. However, the
advantage of Mediterranean agriculture is that more valuable crops such as fruits and vegetables are grown
in winters when there is great demand in European and North American markets

108. Which of the following climate types is NOT witnessed in Europe?


a) Temperate maritime
b) Continental Interior
c) Mediterranean Climate
d) Humid Subtropical climate

Solution: D
Justification: Option A and option C: These climate types have been explained in another question.
Option B: The continental interior stretches from central Europe westwards into Russia. The climate here is
one of hot damp summers and very cold dry winters.
The main reason for this climate's characteristics is distance from the Sea. Inland areas are warmer in
summer and colder in winter than coastal places. Being far from the sea they also tend to be drier

Direct PDF - https://telegram.me/UPSC_PrelimsTest


https://telegram.me/UPSC_PDF

OGP 2019 GEOGRAPHY MODULE MCQS

109. The Siberia climate is absent in the Southern Hemisphere due to


1. Relative absence of land in the Southern Hemisphere in the higher latitudes
2. Strong oceanic influence on the climate reduces the severity of winter
Which of the above is/are correct?
a) 1 only
b) 2 only
c) Both 1 and 2
d) None

Solution: C
Justification: The Siberian climate is conspicuously absent in the Southern Hemisphere because of the
narrowness of the Southern Hemisphere in the high latitudes. The strong oceanic influence reduces the
severity of the winter and coniferous forests are found only on the mountainous uplands of Southern Chile,
New Zealand, Tasmania and SE Australia

110. Consider the following statements. Despite being in the cool temperate region, Western Europe does not
experience extreme climate because
1. The western margins of Europe are under the influence of easterlies throughout the year.
2. The North Atlantic drift moderates the overall temperature.
Which of the above is/are correct?
a) 1 only
b) 2 only
c) Both 1 and 2
d) None

Solution: B
Justification: The climate of Western Europe is a marine climate. Its abundant moisture comes from the
Atlantic Ocean. The ocean moderates the temperature during the seasons, making the winter moist and mild,

Direct PDF - https://telegram.me/UPSC_PrelimsTest


https://telegram.me/UPSC_PDF

OGP 2019 GEOGRAPHY MODULE MCQS

and the summer moist and cool. This climate primarily stays to the north of the Alps, thus considering the
Alps a climatic divide.
Statement 1: It is under the influence of westerlies throughout the year.
Statement 2: The relatively warm waters of the North Atlantic Drift are responsible for moderating the
climate of Western Europe, so that winters are less cold than would otherwise be expected at its latitude

111. The greatest single band of coniferous forests in the World is in


a) Siberia
b) Western Europe
c) Northern United States
d) Southern Peru

Solution: A
Justification: The pre-dominant vegetation of this Siberian "sub-Arctic" type of climate is evergreen
coniferous forest. It stretches in a great, continuous belt across North America, Europe and Asia. The greatest
single band of coniferous forest is the Taiga in Siberia. There are small patches of coniferous forests in
European countries like Germany due to the high altitude.

112. Moderate and Milder climate in the Southern Hemisphere as compared to the Northern hemisphere, for
the same latitude, can be explained by
1. Continentality
2. Photoperiodism
Which of the above is/are correct?
a) 1 only
b) 2 only
c) Both 1 and 2
d) None

Solution: A
Justification: This is so because the Southern Hemisphere has significantly more ocean and much less land;
water heats up and cools down more slowly than land. The movement of sea and land breeze helps to
moderate the climate. On the other hand, where in Northern hemisphere there is more land, temperatures
reach both extremes, for e.g. in Central Asia or Siberia.
Statement 2: Photoperiodism is related to plants reaction to light, it isn’t relevant here.

WATER (OCEANS)
WATER

113. Consider the following statements.


1. Glaciers form only on land.
2. Most of the glacial ice is contained within ice sheets in the Polar Regions.
3. Glacial ice is the largest reservoir of fresh water on Earth.
Select the correct answer using the codes below.
a) 1 and 2 only
b) 2 and 3 only
c) 1 and 3 only
d) 1, 2 and 3

Solution: D
Justification: Statement 1: Glaciers are made up of fallen snow that, over many years, compresses into large,
thickened ice masses. Glaciers form when snow remains in one location long enough to transform into ice.

Direct PDF - https://telegram.me/UPSC_PrelimsTest


https://telegram.me/UPSC_PDF

OGP 2019 GEOGRAPHY MODULE MCQS

Statement 2: Presently, glaciers occupy about 10 percent of the world's total land area, with most located in
Polar Regions like Antarctica, Greenland, and the Canadian Arctic. On Earth, 99% of glacial ice is contained
within vast ice sheets in the Polar Regions, but glaciers may be found in mountain ranges on every continent
including Oceania's high-latitude oceanic islands such as New Zealand and Papua New Guinea.
Statement 3: Many glaciers from temperate, alpine and seasonal polar climates store water as ice during the
colder seasons and release it later in the form of meltwater as warmer summer temperatures cause the
glacier to melt, creating a water source that is especially important for plants, animals and human uses when
other sources may be scant.
Within the past 750,000 years, scientists know that there have been eight Ice Age cycles, separated by
warmer periods called interglacial periods. Currently, the Earth is nearing the end of an interglacial, meaning
that another Ice Age is due in a few thousand years. This is part of the normal climate variation cycle.
Scientists still have many questions to answer about climate change. Although glaciers change very slowly
over long periods, they may provide important global climate change signals.

114. Consider the following statements.


Assertion (A): The sea surface, in terms of elevation, is changing at the same rate at all points around the
globe.
Reason (R): All the major oceans of the World are connected.
In the context of the above, which of these is correct?
a) A is correct, and R is an appropriate explanation of A.
b) A is correct, but R is not an appropriate explanation of A.
c) A is correct, but R is incorrect.
d) A is incorrect, but R is correct.

Solution: D
Justification: Global sea level trends and relative sea level trends are different measurements. Just as the
surface of the Earth is not flat, the surface of the ocean is also not flat — in other words, the sea surface is not
changing at the same rate at all points around the globe. Sea level rise at specific locations may be more or
less than the global average due to many local factors: subsidence, ocean currents, variations in land height,
and whether the land is still rebounding from the compressive weight of Ice Age glaciers. Sea level is primarily
measured using tide stations and satellite laser altimeters.

115. The thermocline layer represents vertical zone of oceanic water below the first layer and is characterized
by rapid rate of decrease of temperature with increasing depth. The largest number of such layers are usually
found in
a) Tropics
b) Poles
c) Temperate regions
d) Southern Ocean

Solution: A
Justification: Oceans absorb more than 80% of the solar radiation and water which has highest specific heat is
the remarkable capacity of storing the heat. The uppermost 10% of the oceans has more heat than the entire
atmosphere of earth. With respect to temperature, there are three layers in the oceans from surface to the
bottom in the tropics as follows:
 The first layer represents the top-layer of warm, oceanic water and is 500m thick with temperature
ranging between 20° and 25°C.
 This layer is present within the tropics throughout the year but it develops in mid· latitudes only during
summer.
 The third layer is very cold and extends upto the deep ocean floor. The polar areas have only one layer of
cold water from surface (sea level) to the deep ocean floor.

116. A lot of interest has been recently shown in seafloor hydrothermal systems on account of their

Direct PDF - https://telegram.me/UPSC_PrelimsTest


https://telegram.me/UPSC_PDF

OGP 2019 GEOGRAPHY MODULE MCQS

1. High concentration of base metals


2. High biodiversity
Which of the above is/are correct?
a) 1 only
b) 2 only
c) Both 1 and 2
d) None

Solution: C
Justification: The discovery of some modern hydrothermal systems in the deep oceanic realm along the mid-
ocean ridges has kindled a lot of interest primarily on account of the high concentration of base metals (Cu
and Zn) and many noble metals (Au, Ag, Pd, Pt) in them. Apart from their economic potential, sea-floor
hydrothermal vents are characterized by dense biologic communities. Over 700 species have been reported,
making the vents among the most productive ecosystems on Earth. The basic paradigm of hydrothermal
activity involves sub-seafloor convective circulation of seawater through permeable rocks mainly driven by
the upper mantle heat sources. At water depths up to 3,700 metres, hydrothermal fluids, having seeped from
the ocean into subterranean chambers where they are heated by the molten rock beneath the crust, are
discharged from the black smokers at temperatures up to 400° C. As these fluids mix with the cold
surrounding seawater, metal sulphides in the water are precipitated onto the chimneys and nearby seabed.
These sulphides, including galena (lead), sphalerite (zinc) and chalcopyrite (copper), accumulate at and just
below the seafloor, where they form massive deposits that can range from several thousands to about 100
million tonnes.

117. Sea level rise at specific locations may be more or less than the global average due to many factors.
Which of these factors can create differences between local sea level and global sea level?
1. Local land subsidence

Direct PDF - https://telegram.me/UPSC_PrelimsTest


https://telegram.me/UPSC_PDF

OGP 2019 GEOGRAPHY MODULE MCQS

2. Ocean currents
3. Variations in land height

Select the correct answer using the codes below.


a) 1 only
b) 2 only
c) 2 and 3 only
d) 1, 2 and 3

Solution: D
Justification: Global sea level trends and relative sea level trends are different measurements. Just as the
surface of the Earth is not flat, the surface of the ocean is also not flat — in other words, the sea surface is not
changing at the same rate at all points around the globe. This is due to many local factors: subsidence, ocean
currents, variations in land height, and whether the land is still rebounding from the compressive weight of
Ice Age glaciers. For e.g. land subsidence involves gradual settling or sudden sinking of the Earth’s surface.
This can short-term changes in local sea levels.

MOVEMENT OF OCEAN WATER

118. Water that rises to the surface as a result of Ocean Upwelling is typically
a) Colder and poor in nutrients
b) Warmer and poor in nutrients
c) Colder and rich in nutrients
d) Warmer and rich in nutrients

Solution: C
Justification: Winds blowing across the ocean surface push water away. Water then rises up from beneath
the surface to replace the water that was pushed away. This process is known as “upwelling.” Upwelling
occurs in the open ocean and along coastlines. The reverse process, called “downwelling,” also occurs when
wind causes surface water to build up along a coastline and the surface water eventually sinks toward the
bottom. Water that rises to the surface as a result of upwelling is typically colder and is rich in nutrients.
These nutrients “fertilize” surface waters, meaning that these surface waters often have high biological
productivity. Therefore, good fishing grounds typically are found where upwelling is common.

119. The meeting of the cold Labrador current and warm Gulf stream current leads to
1. Dense mist and fog
2. An Arid climate Zone
Which of the above is/are correct?
a) 1 only
b) 2 only
c) Both 1 and 2
d) None

Solution: A
Justification: Statement 1: When cold and warm ocean current meet, the warm ocean current condenses
after getting in contact with cold air and it results into fog.
Statement 2: It also leads to high rate of precipitation in the region.

Direct PDF - https://telegram.me/UPSC_PrelimsTest


https://telegram.me/UPSC_PDF

OGP 2019 GEOGRAPHY MODULE MCQS

The circulation in the area where the Labrador Current and Gulf Stream meet is one of the most complex and
variable regions in the world.

120. Which of these forces help in the movement or generation of ocean currents?
1. Coriolis force
2. Gravitational force
3. Solar insolation
4. Movement of wind
Select the correct answer using the codes below.
a) 2 and 3 only
b) 1 and 4 only
c) 2, 3 and 4 only
d) D. 1, 2, 3 and 4
Solution: D
Justification: Statement 1 and 4: Surface currents are generated largely by wind. Their patterns are
determined by wind direction, Coriolis forces from the Earth’s rotation, and the position of landforms that
interact with the currents.
Statement 2: Near the equator the ocean water is about 8 cm higher in level than in the middle latitudes. This
causes a very slight gradient and water tends to flow down the slope.
Statement 3: Solar energy makes current warm or cold and decides their circulatory patterns.

121. It is the western boundary current of the southwest Indian Ocean. It acts as an oceanic convergence
zone. This ocean current is
a) Labrador
b) Gulf current
c) Benguela
d) Agulhas

Solution: D
Justification: It flows down the east coast of Africa from 27°S to 40°S. It is narrow, swift and strong. It is even
suggested that the Agulhas is the largest western boundary current in the world ocean. The Agulhas acts as
an oceanic convergence zone. Due to mass continuity this drives surface waters down, resulting in the
upwelling of cold, nutrient rich water south of the current. Additionally, the convergence tends to increase
the concentration of plankton in and around the Agulhas. Both of these factors result in the area being one of
enhanced primary productivity as compared to the surrounding waters.

122. World’s best fishing grounds are formed where


a) Coastal areas adjoin high density of warm currents

Direct PDF - https://telegram.me/UPSC_PrelimsTest


https://telegram.me/UPSC_PDF

OGP 2019 GEOGRAPHY MODULE MCQS

b) Warm and cold currents meet


c) There is a severe lack of phytoplankton
d) Deep marine trenches are found

Solution: B
Justification: The following conditions are good for fisheries production & catch:
 Coastal upwelling of nutrient rich colder water
 Abundance of phytoplankton
 Meeting of warm and cold currents
 Shallow continental shelves in cold regions
It is exemplified by the fact that one of the world's richest fishing grounds situated around the coast of
Iceland.

123. Within the Earth's ocean and atmosphere, Rossby waves form as a result of

a) Rotation of the planet


b) Churning of oceans
c) Tectonic movement beneath ocean plates
d) Collision of clouds

Solution: A
Justification: Waves in the ocean come in many different shapes and sizes. Slow-moving oceanic Rossby
waves are fundamentally different from ocean surface waves. Unlike waves that break along the shore,
Rossby waves are huge, undulating movements of the ocean that stretch horizontally across the planet for
hundreds of kilometres in a westward direction. Also known as planetary waves, they naturally occur in
rotating fluids. They are so large and massive that they can change Earth's climate conditions.

INDIAN PHYSICAL GEOGRAPHY


PHYSIOGRAPHY
STRUCTURE AND PHYSIOGRAPHY

MOUNTAINS AND HILLS

124. Consider the following statements.


Assertion (A): The Himalayas continue to increase in height.
Reason (R): The Indian plate continues to move northwards into Asia.
In the context of the above, which of these is correct?
a) A is correct, and R is an appropriate explanation of A.
b) A is correct, but R is not an appropriate explanation of A.
c) A is correct, but R is incorrect.
d) Both A and R are incorrect.

Solution: A
Justification: About 80 million years ago, India was located roughly 6,400 km south of the Asian continent,
moving northward at a rate of about 9 m a century. When India rammed into Asia about 40 to 50 million
years ago, its northward advance slowed by about half. The collision and associated decrease in the rate of
plate movement are interpreted to mark the beginning of the rapid uplift of the Himalayas. The Himalayas
and the Tibetan Plateau to the north have risen very rapidly. In just 50 million years, peaks such as Mt.

Direct PDF - https://telegram.me/UPSC_PrelimsTest


https://telegram.me/UPSC_PDF

OGP 2019 GEOGRAPHY MODULE MCQS

Everest have risen to heights of more than 9 km. The impinging of the two landmasses has yet to end. The
Himalayas continue to rise more than 1 cm a year -- a growth rate of 10 km in a million years! If that is so,
why aren't the Himalayas even higher? Scientists believe that the Eurasian Plate may now be stretching out
rather than thrusting up, and such stretching would result in some subsidence due to gravity.

125. Consider the following statements. Fold mountains are


1. Created where two or more of Earth’s tectonic plates are pushed together
2. Often associated with continental crust
3. The most common type of mountain in the world
4. Composed mainly of igneous rocks
Select the correct answer using the codes below.
a) 1, 2 and 3 only
b) 2 and 3 only
c) 1 and 4 only
d) D. 1, 2, 3 and 4

Solution: A
Justification: Statement 1: At these colliding, compressing boundaries, rocks and debris are warped and
folded into rocky outcrops, hills, mountains, and entire mountain ranges.
Statement 2: They are created at convergent plate boundaries, sometimes called continental collision zones
or compression zones. Convergent plate boundaries are sites of collisions, where tectonic plates crash into
each other. Compression describes a set of stresses directed at one point in a rock or rock formation.

Direct PDF - https://telegram.me/UPSC_PrelimsTest


https://telegram.me/UPSC_PDF

OGP 2019 GEOGRAPHY MODULE MCQS

Statement 3: Himalayas, Andes, and Alps are all active fold mountains. The Andes are the world’s longest
mountain chain.
Statement 4: Most Fold Mountains are composed of sedimentary rock and metamorphic rock formed under
high pressure and relatively low temperatures. Many fold mountains are also formed where an underlying
layer of ductile minerals, such as salt, is present. Fold Mountains are caused by large-scale earth's
movements, when stresses may be due to the increased load of the underlying rocks, flow movements in the
mantle, magnetic intrusions into the crust, or the expansion or contraction of some part of the earth

126. Consider the following hills and their locations.


1. Shevroy Hills: Eastern Ghats
2. Cardamom Hills: Western Ghats
3. Anaimalai Hills: Garhjat Range

Select the correct answer using the codes below.


a) 1 and 2 only
b) 2 only
c) 1 and 3 only
d) 1, 2 and 3

Solution: A
Justification: Statement 1: The Servarayans form part of the southern ranges of the Eastern Ghats System. It
also represents the highest peak in southern part of the Eastern Ghats, with the Solaikaradu peak.
Statement 2: They conjoin the Anaimalai Hills to the northwest, the Palni Hills to the northeast and the
Agasthyamalai Hills to the south as far as the Aryankavu. The crest of the hills forms the boundary between
Kerala and Tamil Nadu. Anamudi in Eravikulam National Park, is the highest peak in Western Ghats and also
the highest point in India south of the Himalayas
Statement 3: They form a southern portion of the Western Ghats. Anamala / Anaimalai Hills are south of
where the Western Ghats are broken by the Palakkad Gap, which in turn is south of the Nilgiri Hills. They
border the state of Kerala on the Southwest and the Cardamom Hills to the southeast.

127. Consider the following statements about the Himalayas.


1. The altitudinal variations are greater in the eastern half than those in the western half.
2. The core of Eastern Himalayas is made up of Sandstone, but that of Western Himalayas is made up of
Basalt and Granite.
Which of the above is/are correct?
a) 1 only
b) 2 only
c) Both 1 and 2
d) None

Solution: A
Justification: Statement 1: This means that the altitude of Western Himalayas peaks does not differ
significantly as compared to the Eastern Himalayas.
Statement 2: The folds of Great Himalayas are asymmetrical in nature. The core of this part of Himalayas is
composed of granite. It is perennially snow bound, and a number of glaciers descend from this range. The
outer most range of the Himalayas (Shiwaliks) is made up of unconsolidated deposits.

128. These are some of the highest peaks of Himalayas. Which of them are situated in India?
1. Dhaulagiri
2. Kanchenjunga
3. Namcha Barwa
4. Nanda Devi
5. Mt. Everest

Direct PDF - https://telegram.me/UPSC_PrelimsTest


https://telegram.me/UPSC_PDF

OGP 2019 GEOGRAPHY MODULE MCQS

Select the correct answer using the codes below.


a) 1, 3 and 5 only
b) 1, 2 and 4 only
c) 2, 3 and 4 only
d) 1, 2, 3 and 5 only

Solution: C
Justification: The highest peak of Himalayas is outside India, but the second highest is in India. If you visit hill
stations situated in NE India or travel across Himadri (northern Himalayas), you can see most of the
prominent peaks.

Direct PDF - https://telegram.me/UPSC_PrelimsTest


https://telegram.me/UPSC_PDF

OGP 2019 GEOGRAPHY MODULE MCQS

129. Places in the mountains are usually cooler than places on the plains due to the presence of
a) Leeward wind flow on the mountains
b) Moisture-laden winds on the mountains
c) Thin air on the mountains
d) All of the above

Solution: C
Justification: Density of air reduces with altitude, and so it is its ability to transfer the heat that the earth
receives from the Sun. You know that the atmosphere is not directly heated by the Sun, but by the air that is
in contact with the earth. So, thin air reduces the warming potential in the mountains. Temperatures decline
with gain in altitude. For example, Agra and Darjeeling are located on the same latitude, but temperature of
January in Agra is 16°C whereas it is only 4°C in Darjeeling.

130. The Hindu Kush Himalayan (HKH), often referred to as the ‘Third Pole’ of the earth, is the source of ten
large Asian river systems that include
1. Ganga
2. Mekong
3. Brahmaputra
4. Yellow River
5. Indus

Select the correct answer using the codes below.


a) 2, 4 and 5 only
b) 1, 3 and 5 only
c) C. 1, 2, 3, 4 and 5

Direct PDF - https://telegram.me/UPSC_PrelimsTest


https://telegram.me/UPSC_PDF

OGP 2019 GEOGRAPHY MODULE MCQS

d) D. 1 and 3 only

Solution: C
Justification: The Hindu Kush Himalayan (HKH) region is one of the greatest mountain systems in the world,
encompassing an area of over 4.3 million km2. Outside of the North and South Poles, the region contains the
largest area of permanent ice cover in the world, which is why it is sometimes referred to as the ‘Third Pole’
of the earth. Its terrain encompasses the source of 10 major river systems, 4 global biodiversity spots, 330
important bird areas, and hundreds of mountain peaks over 6,000 masl. The region provides water,
ecosystem services, food, energy, and livelihood to 210 million people directly. The region extends 3,500 km
over all or part of eight countries from Afghanistan in the west to Myanmar in the east. The ten large Asian
river systems that it includes are -– the Amu Darya, Indus, Ganges, Brahmaputra (Yarlungtsanpo), Irrawaddy,
Salween (Nu), Mekong (Lancang), Yangtse (Jinsha), Yellow River (Huanghe), and Tarim (Dayan) - basins of
these rivers provide water to 1.3 billion people, a fifth of the world’s population. Everest, K2, Dhaulagiri,
Annapurna are some of the highest peaks in the Hindu Kush region. It has been predicted that this vital Hindu
Kush Himalayan (HKH) region will suffer a loss of 33 per cent of total ice volume in the by the end of the
century, leading to dire consequences for people living there.

131. What are the major differences between the physical and climatic conditions of the eastern and western
Himalayas?
1. Western Himalayas receive most of the precipitation in the winter months and Eastern Himalayas in the
summer months.
2. Western Himalayas are lower in average latitude than the eastern Himalayas.
3. Snowline occurs at a higher altitude in Eastern Himalayas than in the Western Himalayas.

Select the correct answer using the codes below.


a) 1 and 2 only
b) 1 and 3 only
c) 3 only
d) 1, 2 and 3

Solution: B
Justification: There are certain differences between the physical and climatic conditions of the eastern and
western Himalayas which also have an effect on the glaciers and their terminal level.
These are: (a) Western Himalayas receive most of the precipitation in the winter months and Eastern
Himalayas in the summer months. Western Himalayas are in higher latitude than the eastern Himalayas.
Ladakh glaciers come down gentler slope than those of Eastern Himalayas. Winter precipitation is at a lower
temperature than the summer precipitation. Even in Eastern Himalayas, winter snowline comes down to
2000 m.
Statement 3: During winters snow and sleet falling on the slopes which are cooler collects and on the snow
and ice surfaces it induces freezing. During winter months on the glacial surface whatever snow falls collects
and it adds to the thickness of the glacier. Summer precipitation is at a higher temperature and it is mostly in
the form of rains up to 4266 m. Rain water induces melting and thinning of the glacial surface. In Eastern
Himalayas where most of the precipitation is in the summers both the snowline and the terminal end of the
glaciers are higher.

132. Arrange the following mountain peaks from North to South.


1. Nanda Devi
2. Kangchenjunga
3. Saramati
4. Kangju Kangri

Select the correct answer using the codes below.


a) 1234

Direct PDF - https://telegram.me/UPSC_PrelimsTest


https://telegram.me/UPSC_PDF

OGP 2019 GEOGRAPHY MODULE MCQS

b) 4123
c) 1324
d) 4132

Solution: B
Justification: Look at the Northern section of the map below.

Saramati is the highest mountain range of Nagaland. Saramati is one of the ultra-prominent peaks of
Southeast Asia.
Kangju Kangri is a mountain in the Karakoram located in the state of Jammu and Kashmir.
With a summit elevation of 6,725 meters above sea level, it is the highest peak of the Pangong range, a
subrange of the Karakoram

133. The Arakan Mountains


1. Have acted as a barrier between the peoples of central Burma and those of the Indian subcontinent
2. Are submerged in the Bay of Bengal for a long stretch emerging again in the form of the Andaman and
Nicobar Islands
Which of the above is/are correct?

Direct PDF - https://telegram.me/UPSC_PrelimsTest


https://telegram.me/UPSC_PDF

OGP 2019 GEOGRAPHY MODULE MCQS

a) 1 only
b) 2 only
c) Both 1 and 2
d) None

Solution: C
Justification: They are a series of parallel ridges arcing through Assam, Nagaland, Mizoram, Burma and are
submerged in the Bay of Bengal for a long stretch emerging again in the form of the Andaman and Nicobar
Islands. The Arakan Mountains (also called Rakhine ranges) and the parallel arcs to the west and east were
formed by compression as the Indian Plate collided with the Eurasian Plate approximately along the boundary
between India and Nepal. The Arakan Mountains act as a barrier to the south-western monsoon rains and
thus shield the central Myanmar area, making their western slopes extraordinarily wet during the monsoon.

134. Consider the following statements.


1. They are consecutive series of alluvial fans forming along the edge of a linear mountain range.
2. They usually occur in arid climates.

The above refer to?


a) Bajada
b) Loess
c) Sial
d) Ex-bow fans

Solution: A
Justification: A bajada consists of a series of coalescing alluvial fans along a mountain front. These fan-shaped
deposits form from the deposition of sediment within a stream onto flat land at the base of a mountain.
Surface of this feature undulates in a rolling fashion as one moves from the centre of one alluvial fan to
another.

135. Consider the following statements.

Direct PDF - https://telegram.me/UPSC_PrelimsTest


https://telegram.me/UPSC_PDF

OGP 2019 GEOGRAPHY MODULE MCQS

1. Southern ranges in eastern Himalayas have lower snowline than the northern ranges due to higher
precipitation towards the North.
2. The Eastern Ghats and Deccan Plateau are drier than Western Ghats and consequently have less diverse
vegetation.

Which of the above is/are correct?


a) 1 only
b) 2 only
c) Both 1 and 2
d) None

Solution: B
Justification: Statement 1: Himalayas are oriented east-west and their southern slopes are in direct sunshine
for a larger part of the year so the snowline on the southern slopes of the ridges is higher than the northern
slopes. Also, volume of precipitation decreases from the south towards the north, therefore southern ranges
in eastern Himalayas have lower snowline than the northern ranges because they receive more rainfall
allowing for snow to form at lower altitudes.
Statement 2: This is because Eastern Ghats are in the rain shadow zone of Monsoon winds, and Western
Ghats receives more rainfall than both Deccan plateau and Eastern Ghats.

136. The Vindhyan system of rocks is one of the important rock systems of India. It is known for the
production of
a) Uranium and thorium
b) Iron ore, coal and manganese
c) Precious stones and building materials
d) Bauxite and mica

Solution: C
Justification: The Vindhyan Mountains form a dividing line between the Ganges plain and Deccan Plateau.
This system rocks are extensively distributed in India from Chittorgarh in Rajasthan to Sasaram in Bihar. The
well-known Panna and Golconda diamonds are found in this formation. The important series of this system
are Bhander series, Bijwar series and Kaimur series. All are rich sources of Building material. The rock systems
of India (dharwar, Archean, cuddapah, vindhyan, tertiary) are very important. You should read their major
features from a credible source.

137. Why a very large thickness of sedimentary rocks is found in Fold Mountains?
a) Due to the folding of sediments into niber folds
b) Due to the folding of plains into mountains
c) Due to accumulation of sediments in a geo-syncline
d) Most fold mountains are formed from igneous rocks that are later eroded to form sedimentary rocks

Solution: C
Justification: Formation of Fold Mountains: Rivers deposit huge quantities of sediments in depressions called
geosynclines. Over millions of years the sediments are compressed into sedimentary rocks such as sandstone
and limestone. The plates move together (a compressional plate boundary) forcing the sedimentary rocks
upwards into a series of folds by the movement of tectonic plates. This causes the thickness of the rocks in
Fold Mountains.

138. They are a series of parallel ridges arcing through Assam, Nagaland, Mizoram, Burma and are submerged
in the Bay of Bengal for a long stretch emerging again in the form of the Andaman and Nicobar Islands. The
mountains are
a) Namcha Barwa Ranges
b) Trans-Alay Range

Direct PDF - https://telegram.me/UPSC_PrelimsTest


https://telegram.me/UPSC_PDF

OGP 2019 GEOGRAPHY MODULE MCQS

c) Dhaulagiri Mountains
d) Arakan Mountains

Solution: D
Justification: The Arakan Mountains (also called Rakhine ranges) and the parallel arcs to the west and east
were formed by compression as the Indian Plate collided with the Eurasian Plate approximately along the
boundary between India and Nepal. The Arakan Mountains divide the Rakhine coast from the rest of Burma,
and thus have acted as a barrier between the peoples of central Burma and those of the Indian subcontinent.
The Arakan Mountains act as a barrier to the south-western monsoon rains and thus shield the central
Myanmar area, making their western slopes extraordinarily wet during the monsoon.

139. Which of the following description best suits Himalayan mountains?

a) Young fold mountains


b) Old block mountains
c) Rugged and young block mountains
d) Rounded volcanic mountains

Solution: A
Justification: There are three types of mountains- Fold Mountains, Block Mountains and the Volcanic
Mountains.
Option A: The Himalayan Mountains and the Alps are young fold mountains with rugged relief and high
conical peaks. Fold mountains are created when two plates collide which is the case with Himalayas. The
Aravali range in India is one of the oldest fold mountain systems in the world.
Option B: Block Mountains are created when large areas are broken and displaced vertically. This didn’t
happen with Himalayas.

140. The vast majority of the highest mountain peaks in the World are located near
a) Ural region, Eurasia
b) Nepal and Tibet
c) Central Antarctica
d) Andes chain, South America

Solution: B
Justification: There are at least 109 mountains on Earth with elevations greater than 7,200 metres (23,622 ft)
above sea level. The vast majority of these mountains are located on the edge of the Nepal and Tibet, with
some peaks in Central Asia. Almost all mountains in the list are located in the Himalaya and Karakoram ranges
to the south and west of the Tibetan plateau. In fact, all 7,000 m (23,000 ft) peaks in the world are located in
the centre of Asia (East Asia, Central Asia and South Asia). The highest mountain outside of Asia is Aconcagua
(6,962 m or 22,841 ft), which one list has ranking 189th in the world amongst mountains with a 500 m or
1,640 ft prominence cutoff.

PLATEAU

141. Which of the following fall under the peninsular block of India?
1. Karbi Anglong Plateau
2. Rajmahal Hills
3. Meghalaya Plateau
Select the correct answer using the codes below:
a) 1 and 2 only
b) 2 and 3 only
c) 1 only

Direct PDF - https://telegram.me/UPSC_PrelimsTest


https://telegram.me/UPSC_PDF

OGP 2019 GEOGRAPHY MODULE MCQS

d) 1, 2 and 3

Solution: D
Justification: The northern boundary of the Peninsular Block may be taken as an irregular line running from
Kachchh along the western flank of the Aravali Range near Delhi and then roughly parallel to the Yamuna and
the Ganga as far as the Rajmahal Hills and the Ganga delta Apart from these, the Karbi Anglong and the
Meghalaya Plateau in the northeast and Rajasthan in the west are also extensions of this block. The
northeastern parts are separated by the Malda fault in West Bengal from the Chotanagpur plateau. In
Rajasthan, the desert and other desert–like features overlay this block

142. Consider the following statements about the Deccan Plateau.


1. Most Deccan plateau rivers flow south.
2. It is composed of the old crystalline igneous and metamorphic rocks.
3. It is higher along its Eastern edge gently slopes towards the Arabian Sea in the West.

Select the correct answer using the codes below.


a) 1 only
b) 1 and 2 only
c) 1 and 3 only
d) 2 and 3 only

Solution: B
Justification: S3: It forms a raised triangle within the South-pointing triangle of the Indian subcontinent's
coastline. The western edge is higher and slopes towards the Bay of Bengal in the east. Most of the northern
part of the plateau is drained by the Godavari River and its tributaries, including the Indravati River, starting
from the Western Ghats and flowing east towards the Bay of Bengal. The climate of the region varies from
semi-arid in the north to tropical in most of the region with distinct wet and dry seasons. It is bounded by
Aravalli Hills in the North-West, Hazaribagh & Rajmahal Hills in the North-East, the Western Ghats in the West
and the Eastern Ghats in the East.

143. The mineral belt in the North-Western Region of the country that extends along Aravali in Rajasthan and
parts of Gujarat is mainly associated with which system of rocks?
a) Archean
b) Dharwar
c) Cuddapah
d) Carboniferous

Solution: B
Justification: Dharwar Rock System is special because it is the first metamorphic sedimentary rocks in India.
Dharwar system is later than the Archean system but older than the other systems. The Dharwar period of
rock formation has been largely fixed from 2500 million years ago to 1800 million years ago. They are named
Dharwar system because they were first studied in Dharwar region of Karnataka. But they are also found in
Aravallis, Tamil Nadu, Chotanagpur plateau, Meghalaya, Delhi, and the Himalayas region. The Dharwar rocks
are rich in iron ore, manganese, lead, zinc, gold, silver etc. The Champions series containing gold mines lie
within this system.

144. Changthang plateau of Ladakh region is known for the production of


a) Thinnest cotton in the world
b) Cross-breed Moonga
c) Pashmina Wool
d) Golden Silk

Solution: C

Direct PDF - https://telegram.me/UPSC_PrelimsTest


https://telegram.me/UPSC_PDF

OGP 2019 GEOGRAPHY MODULE MCQS

Justification: India produces the finest Pashmina (Cashmere) wool in the world, coming from the Changthang
plateau of Ladakh region of J&K State. The Changthang is a high altitude plateau, home to Changpa tribes.
Pashmina wool is produced by Changra (Pashmina) goat in Ladakh and Chegu breed of goat in the eastern
parts of Himalayas. There many local type of Non-Pashmina goat breeds which come from the same larger
genetic pool but contribute little to the total Pashmina production of the region

145. Peninsular plateaus in India are considered to be very useful because


a) It is rich in mineral deposits.
b) Its lava plateaus are generally rich in fertile black soil.
c) It is younger than most mountains in India and holds greater possibility for landscape transformation.
d) It hosts a global biodiversity hotspot.

Select the correct answer using the codes below.


a) 1 and 2 only
b) 2, 3 and 4 only
c) 1, 2 and 4 only
d) 1 and 3 only

Solution: C
Justification: Statement 1: It is store house of minerals like Iron ore, bauxite, gold, copper, coal, manganese,
mica etc.
Statement 2: Black soil of the Deccan plateau is known for growing cotton.
Statement 3: Deccan plateau hosts the oldest rocks in India, so 3 is wrong. It is older than Himalayas.
Statement 4: Western Ghats is located in peninsular plateau.

146. Consider the following statements:


1. The Malwa region plateau is an extension of the Deccan Traps.
2. The Damodar River flows through Chota Nagpur Plateau.
3. Gondwana substrates are found in the Chota Nagpur Plateau.

Which of the above is/are correct?


a) 1 and 2 only
b) 1, 2 and 3
c) 2 and 3 only
d) 3 only

Solution: B
Justification: Statement 1: The Malwa region occupies a plateau in western Madhya Pradesh and south-
eastern Rajasthan with Gujarat in the west. The plateau generally slopes towards the north. The western part
of the region is drained by the Mahi River, while the Chambal River drains the central part, and the Betwa
River and the headwaters of the Dhasan and Ken rivers drain the east. The Shipra River is of historical
importance because of the Simhasth mela, held every 12 years. Other notable rivers are Parbati, Gambhir and
Choti Kali Sindh. The plateau is an extension of the Deccan Traps, formed between 60 and 68 million years
ago at the end of the Cretaceous period. In this region the main classes of soil are black, brown and bhatori
(stony) soil. The region has been one of the important producers of opium in the world. Wheat and soybeans
are other important cash crops, and textiles are a major industry.
Statement 2 and 3: The Chota Nagpur Plateau is a continental plateau—an extensive area of land thrust
above the general land. The plateau has been formed by continental uplift from forces acting deep inside the
earth. The Gondwana substrates attest to the plateau's ancient origin. The Damodar River flows through
Chota Nagpur Plateau (not Mahanadi, so 2 is wrong). Chota Nagpur plateau is a store house of minerals like
mica, bauxite, copper, limestone, iron ore and coal. The Damodar valley is rich in coal and it is considered as
the prime centre of coking coal in the country. Massive coal deposits are found in the central basin. Important

Direct PDF - https://telegram.me/UPSC_PrelimsTest


https://telegram.me/UPSC_PDF

OGP 2019 GEOGRAPHY MODULE MCQS

coalfields in the basin are Jharia, Raniganj, West Bokaro, East Bokaro, Ramgarh, South Karanpura and North
Karanpura.

OTHERS

147. Consider the following mountain passes of Western India and the regions they connect.
1. Thal Ghat -> Nasik to Mumbai
2. Shor Ghat -> Mumbai to Pune
3. Pal Ghat ->Puducherry to Tamil Nadu
4. Senkota pass -> Kodaikanal to Madurai
Select the correct answer using the codes below.
a) 1 and 2 only
b) 2, 3 and 4 only
c) 1 and 3 only
d) 2 and 4 only

Solution: A
Justification: These are some of the important passes of Western Ghats.
 Thal Ghat: It links Nasik to Mumbai.
 Shor Ghat: It links Mumbai to Rune.
 Pal Ghat: It connects Kerala to Tamil Nadu (link Kochi to Chennai).
 Senkota pass: It is located between the Nagercoil and the Cardmom hills links Thiruvanathpuram and
Madurai
The Western Ghat Mountain Range or Sahyadri Range is the India's largest mountain range after the
Himalayas which extend from the Tapi river valley to the Nilgiri. It is divided into two parts- the North
Sahyadri and the south Sahyadri.

148. Consider the following statements about the Western Ghats.


1. The Western Ghats cause orographic rain by facing the rain bearing moist winds to rise along the western
slopes of the Ghats.
2. The Western Ghats are continuous and can be crossed through passes only.

Which of the above is/are correct?


a) 1 only
b) 2 only
c) Both 1 and 2
d) None

Solution: C
Justification: Western ghats lie parallel to the western coast. They are continuous and can be crossed through
passes only, for e.g. the Thal, Bhor and the Pal Ghats. The Eastern Ghats are discontinuous and irregular and
dissected by rivers draining into the Bay of Bengal. The Western Ghats cause orographic rain by facing the
rain bearing moist winds to rise along the western slopes of the Ghats. The height of the Western Ghats
progressively increases from north to south. The Western Ghats are higher than the Eastern Ghats. Their
average elevation is 900–1600 metres as against 600 metres of the Eastern Ghats. The Eastern Ghats stretch
from the Mahanadi Valley to the Nigiris in the south.

149. Consider the following statements.


Assertion (A): The Wular lake in Jammu and Kashmir is a glacial lake.
Reason (R): All the fresh water lakes in the Himalayan region were formed from snowmelt water filled out in a
glacier dug out region.
In the context of the above, which of these is correct?

Direct PDF - https://telegram.me/UPSC_PrelimsTest


https://telegram.me/UPSC_PDF

OGP 2019 GEOGRAPHY MODULE MCQS

a) A is correct, and R is an appropriate explanation of A.


b) A is correct, but R is not an appropriate explanation of A.
c) A is correct, but R is incorrect.
d) Both A and R are incorrect.

Solution: D
Justification: Most, not all, of the fresh water lakes are in the Himalayan region. They are of glacial origin. In
other words, they formed when glaciers dug out a basin, which was later filled with snowmelt. The Wular lake
in Jammu and Kashmir, in contrast, is the result of the tectonic activity. It is the largest freshwater lake in
India. The Dal lake, Bhimtal, Nainital, Loktak and Barapani are some other important fresh water lakes.

150. Badland topography is characterized by


1. Severe action of Wind and Water erosion
2. Presence of Hard archaic granite rock system
3. Semi-arid or arid climates

Select the correct answer using the codes below.


a) 1 and 2 only
b) 2 and 3 only
c) 1 and 3 only
d) 1, 2 and 3

Solution: C
Justification: Badlands are a type of dry terrain where softer sedimentary rocks and clay-rich soils have been
extensively eroded by wind and water. They are characterized by steep slopes, minimal vegetation, lack of a
substantial regolith, and high drainage density. The term ‘badlands’ was first given to an arid area in South
Dakota, U.S.A., where the hills were badly eroded by occasional rain-storms into gullies and ravines. The
extent of water action on hill slopes and rock surfaces was so great that the entire region was abandoned by
the inhabitants. Deserts with similar features are now referred to as badlands, e.g. the Painted Desert of
Arizona, which lies south-east of the Grand Canyon of the Colorado River.

151. In India, a lake formed in upper reaches of Mountains in the Northern region is most likely to be a
1. Tectonic lake
2. Glacial lake
3. Oxbow lake
4. Crater lake

Select the correct answer using the codes below.


a) 1 or 2
b) 2 or 3
c) 1 and 4
d) 1 or 2 or 3 or 4

Solution: A
Justification: Northern region of India can be characterized by:
 Presence of Himalayas with young and unconsolidated rock systems that makes S1 possible. Wular lake
(Jammu & Kashmir) is the largest freshwater lake in India. It was formed by tectonic activity.
 Lack of volcanoes, which rules out option S4.
 High gradient of river flow, which rules out sediment deposition and the formation of ox-bow lakes, so S3
can’t be correct.
 Cold weather and presence of glaciers (and their movement), which makes S2 possible. A lake formed by
melting of glacier is called a glacial lake. Most of the lakes in the Himalayan region are glacial lakes.

Direct PDF - https://telegram.me/UPSC_PrelimsTest


https://telegram.me/UPSC_PDF

OGP 2019 GEOGRAPHY MODULE MCQS

152. Which of following characterizes the Western coasts of India?


a) Highly indented
b) Formation of wave-cut platforms in sea cliffs
c) Lagoons, deltas and swamps
d) Both (a) and (b)

Solution: D
Justification: The west coast of our country is a high rocky retreating coast. On the other hand, erosional
forms dominate in the west coast instead of depositional forms.
High rocks coast features:
 Highly indented
 Formation of wave-cut platforms in front of the sea cliff due to constant water erosional action
 Material eroded in such manner, deposits along the off-shore forming wave terraces.
 Several depositions like this results in the formation of barrier bars and spits that break block the sea
water forming a lagoon.
 Low Sedimentary coast features:
 Lagoons, deltas etc. found
 Lagoons eventually turn into swamps which turn into coastal plains
 Storm and tsunami waves cause drastic changes in the supply of depositional material

153. Consider the following statements about the coasts of India.


1. The East coast of our country is a high rocky retreating coast.
2. Erosional forms dominate in the West coast instead of depositional forms.
Which of the above is/are correct?
a) 1 only
b) 2 only
c) Both 1 and 2
d) None

Solution: B
Justification: High rocks coast features:
 Highly indented
 Formation of wave-cut platforms in front of the sea cliff due to constant water erosional action
 Material eroded in such manner, deposits along the off-shore forming wave terraces.
 Several depositions like this results in the formation of barrier bars and spits that break block the sea
water forming a lagoon.
 Low Sedimentary coast features:
 Lagoons, deltas etc. found
 Lagoons eventually turn into swamps which turn into coastal plains
 Storm and tsunami waves cause drastic changes in the supply of depositional material

154. Eastern Ghats are eroded and cut through by


a) Chambal and Mahanadi
b) Godavari and Krishna
c) Pennar and Gandak
d) Tapti and Kosi

Solution: B
Justification: The Eastern Ghats run from West Bengal state in the north, through Odisha and Andhra Pradesh
to Tamil Nadu in the south passing some parts of Karnataka.

Direct PDF - https://telegram.me/UPSC_PrelimsTest


https://telegram.me/UPSC_PDF

OGP 2019 GEOGRAPHY MODULE MCQS

Option A: Chambal forms part of the Gangetic drainage system, so it can’t cut Eastern Ghats which are not
spread till the Ganges drainage system. They are eroded and cut through by the four major rivers of
peninsular India, known as the Godavari, Mahanadi, Krishna, and Kaveri.
The Eastern Ghats are a discontinuous range of mountains along India's eastern coast. The mountain ranges
run parallel to the Bay of Bengal. The Deccan Plateau lies to the west of the range, between the Eastern Ghats
and Western Ghats. The coastal plains, including the Coromandel Coast region, lie between the Eastern Ghats
and the Bay of Bengal. The Eastern Ghats are not as high as the Western Ghats.

155. There has been constant strife between India and China over the Pangong Tso lake region as both assert
territorial possession. The lake is called as an “endorheic basin” which means

a) An internal drainage system with no external flows


b) A water body that freezes from inside
c) A system that has a net zero impact on the environment
d) A lake in a region of rugged relief

Solution: A
Justification: It is a "high grassland lake", located in the Himalayas stretching out from India to China. One-
third of water body is in Indian control while the rest of the 90 km is under Chinese control.
Option A: An endorheic basin is a closed drainage basin that normally retains water and allows no outflow to
other external bodies of water, such as rivers or oceans, but converges instead into lakes or swamps,
permanent or seasonal, that equilibrate through evaporation. Such a basin may also be referred to as a closed
or terminal basin or as an internal drainage system. The lake is in the process of being identified under the
Ramsar Convention as a wetland of international importance. This will be the first trans-boundary wetland in
South Asia under the convention.

DRAINAGE SYSTEM

156. Peninsular rivers do not carry a large quantity of silt as well as are characterised by absence of meanders.
This can be due to which of these reasons?
1. These are fast flowing rivers due to which formation of meanders become difficult.
2. There rivers pass through hard rocky terrain due to erosion capacity of the river is not effective.

Which of the above is/are correct?


a) 1 only
b) 2 only
c) Both 1 and 2
d) None

Solution: C
Justification: Peninsular rivers flow at fast speeds. When they slow down, lateral bank cutting is predominant.
Any river that flow at a considerable speed can erode both lateral banks as well as river bed. However, this
erosion can take place only when the bank is soft or river bed is made of erodible soft rocks. Since, peninsular
blocks are hard, rocky, old and consolidated, little silt is accumulated by these rivers. Same is true for
meanders, for meanders to form rivers must slow down and the terrain must be soft.

157. Cauvery river is an inter‐State basin. Consider the following with reference to it.
1. The river originates in the western Ghats near Mahabaleshwar.
2. It flows through Karnataka and Tamil Nadu, among others.
3. It falls in the Bay of Bengal.
4. The river is also called Krishnaveni.
5. Delta of the river was home to ancient Satavahana and Ikshvaku Sun Dynasty kings.

Direct PDF - https://telegram.me/UPSC_PrelimsTest


https://telegram.me/UPSC_PDF

OGP 2019 GEOGRAPHY MODULE MCQS

Select the correct answer using the codes below.


a) 1, 4 and 5 only
b) 2 and 3 only
c) 2, 4 and 5 only
d) 1, 2 and 5 only

Solution: B
Justification: Statement 1: It originates in Karnataka.
Statement 2 and 3: It flows through Karnataka, Tamil Nadu and Puduchery before out-falling in Bay of Bengal.
Statement 4 and 5: Both of these statements refer to the Krishna river which is a major source of irrigation for
Maharashtra, Karnataka, Telangana and Andhra Pradesh. The sharing of waters of the Cauvery has been the
source of a serious conflict between the states of Tamil Nadu and Karnataka. The genesis of this conflict rests
in two agreements in 1892 and 1924 between the erstwhile Madras Presidency and Kingdom of Mysore.

158. Consider the following statements. Most of the rivers of peninsular India
1. Originate in the Western Ghats
2. Flow towards the Arabian Sea
Which of the above is/are correct?
a) 1 only
b) 2 only
c) Both 1 and 2
d) None

Solution: A
Justification: Statement 1: The Peninsular rivers have shorter and shallower courses as compared to their
Himalayan counterparts. Most of them come from Western Ghats, however, some of them originate in the
central highlands and flow towards the west. Rest of them all flow to the east in the Bay of Bengal. A large
number of the peninsular rivers are seasonal, as their flow is dependent on rainfall. During the dry season,
even the large rivers have reduced flow of water in their channels. Moreover, due to flowing on hard terrain
and short courses, they carry much lesser silt as compared to Himalayan Rivers.

159. The Brahmaputra tends to form river islands mainly due to


1. High gradient during flow
2. Larger volume of water flow in Tibet (upper reaches) as compared to India (lower reaches)
3. Its braided channel
Select the correct answer using the codes below.
a) 1 only
b) 2 and 3 only
c) 3 only
d) 1, 2 and 3

Solution: C
Justification: The Brahmaputra is highly braided due to low gradient and tends to form river islands. Unlike
other north Indian rivers, the Brahmaputra is marked by huge deposits of silt on its bed causing the river bed
to rise. The river also shifts its channel frequently.
Statement 2: In Tibet the river carries a smaller volume of water and less silt as it is a cold and a dry area. In
India it passes through a region of high rainfall. Here the river carries a large volume of water and
considerable amount of silt. The Brahmaputra has a braided channel in its entire length in Assam and forms
many riverine islands. Every year during the rainy season, the river overflows its banks, causing widespread
devastation due to floods in Assam and Bangladesh

160. The most part of the river Basin of Indus lies in

Direct PDF - https://telegram.me/UPSC_PrelimsTest


https://telegram.me/UPSC_PDF

OGP 2019 GEOGRAPHY MODULE MCQS

a) India
b) China
c) Pakistan
d) Afghanistan

Solution: C
Justification: A little over a third of the Indus basin is located in India in the states of Jammu and Kashmir,
Himachal Pradesh and the Punjab and the rest is in Pakistan. According to the regulations of the Indus Water
Treaty (1960), India can use only 20 per cent of the total water carried by Indus river system. This water is
used for irrigation in the Punjab, Haryana and the southern and western parts of Rajasthan.

161. Tributaries of the so called “Dakshin Ganga” include


1. Musi
2. Pranhita
3. Manjra
4. Wainganga

Select the correct answer using the codes below.


a) 1 and 2 only
b) 2, 3 and 4 only
c) 1 and 4 only
d) 2 and 3 only

Solution: B
Justification: The Godavari is the largest peninsular river. Because of its length and the area it covers, it is
also known as the ‘Dakshin Ganga’. The Godavari is joined by a number of tributaries such as the Purna, the
Wardha, the Pranhita, the Manjra, the Wainganga and the Penganga. The last three tributaries are very large.
Godavari rises from the slopes of the Western Ghats in the Nasik district of Maharashtra. Its length is about
1500 km. It drains into the Bay of Bengal. Its drainage basin is also the largest among the peninsular rivers.
The basin covers parts of Maharashtra (about 50 per cent of the basin area lies in Maharashtra), Madhya
Pradesh, Orissa and Andhra Pradesh.

162. Which of the following characterize backwaters?


1. It usually has a higher rate of flow than the main stream that formed it.
2. It is always formed near a river delta.
Which of the above is/are correct?
a) 1 only
b) 2 only
c) Both 1 and 2
d) None

Solution: D
Justification: Statement 1: Backwater is any stretch of water that has become bypassed by the main flow of a
stream, although still joined to it. If a river has developed one or more alternative courses in its evolution,
one channel is usually designated the main course, and secondary channels may be termed backwaters. The
main river course will usually have the fastest stream and will likely be the main navigation route; backwaters
may be shallower and flow more slowly, if at all. It has little or no current.
Statement 2: It is not necessary. It can be formed anywhere en route the river.

163. Which of the following is NOT a tributary of Ganga river?


a) Ghaghara
b) Gandak
c) Mandakini

Direct PDF - https://telegram.me/UPSC_PrelimsTest


https://telegram.me/UPSC_PDF

OGP 2019 GEOGRAPHY MODULE MCQS

d) Brahmani
Solution: D
Justification: Son, Ghaghara, Gomti, Yamuna and Gandak are some of the most important tributaries of
Ganga. Brahmani is a tributary of Mahanadi. Others are Ib, Jonk, Tel, HAsdep, Kuakhai etc. The important
tributaries of the Yamuna River are Tons, Chambal, Hindon, Betwa and Ken. Other small tributaries of the
Yamuna River include the Giri, Sind, Uttangan, Sengar and the Rind.

164. Match the following rivers to their state of origin.


1. Mahanadi : Chhattisgarh
2. Godavari: Andhra Pradesh
3. Cauvery: Kerala
4. Tapti: Madhya Pradesh

Select the correct answer using the codes below.


a) 1 and 4 only
b) 2 and 3 only
c) 1, 2 and 3 only
d) 1, 3 and 4 only

Solution: A
Justification: Statement 1: Mahanadi is a combination of many mountain streams and thus its precise source
is impossible to pinpoint. However its farthest headwaters lie 6 kilometres from Pharsiya village in Dhamtari
district of Chhattisgarh.
Statement 2: The Godavari is India's second longest river after the Ganga. Its source is in Triambakeshwar,
Maharashtra.
Statement 3: Originating in the foothills of Western Ghats at Talakaveri, Kodagu in Karnataka Cauvery flows
generally south and east through Karnataka and Tamil Nadu emptying into the Bay of Bengal.
Statement 4: The Tapti River between the Godavari and Narmada rivers starts from Betul, Madhya Pradesh,
and then flows westwards before draining through the Gulf of Khambhat into the Arabian Sea

165. Which of the following rivers does NOT flow through a rift valley?
a) Narmada
b) Tapti
c) Damodar
d) Mahanadi

Solution: D
Justification: A rift valley is a lowland region that forms where Earth's tectonic plates move apart, or rift. Such
a fault is a fracture in the terrestrial surface in which the rock material on the upper side of the fault plane
has been displaced downward relative to the rock below the fault. Narmada flows in a rift valley flowing west
between the Satpura and Vindhya ranges. The other rivers which flow through rift valley include Damodar
River in Chota Nagpur Plateau and Tapti. The Tapti River and Mahi River also flow through rift valleys, but
between different ranges.

166. Consider the following about Mahanadi river.


1. It is the second largest river in India after Ganges.
2. Odisha and Chhattisgarh are locked in dispute over sharing of water of Mahanadi River.
3. It drains in Bay of Bengal.

Select the correct answer using the codes below.


a) 1 and 3 only
b) 2 and 3 only

Direct PDF - https://telegram.me/UPSC_PrelimsTest


https://telegram.me/UPSC_PDF

OGP 2019 GEOGRAPHY MODULE MCQS

c) 1 and 2 only
d) 1, 2 and 3

Solution: B
Justification: Statement 3: Mahanadi is major river in East Central India. It drains an area of around 141,600
square kilometres and has total course of 858 kilometres. It flows through Chhattisgarh and Odisha. It
originates from highlands of Chhattisgarh through collection of array of streams and reaches Bay of Bengal. It
left bank tributaries are Shivnath, Mand, Ib, Hasdeo and right bank are Ong, parry river, Jonk, Telen.
Statement 1: The longest river in India is the Ganga followed by Godavari (1465 km). Brahmaputra, called
Tsang-po in Tibet, though having a total length of 2900 km, flows only for 916 km in India.
Statement 2: Since mid-80s, both states are in dispute. Odisha has claimed that Chhattisgarh government has
been constructing dams in upper reaches of Mahanadi, depriving its farmers who are heavily dependent on
river water. Chhattisgarh has been against the setting up of a tribunal, and argued that water sharing
agreement was with erstwhile Madhya Pradesh government, before state bifurcated in 2000.

167. Consider the following statements about Brahmaputra river.


1. The Brahmaputra rises in Tibet east of Mansarowar lake.
2. Its source is very close to that of the Indus and Satluj.
3. Overall, it is slightly shorter than the Indus.
4. Most of its course lies outside India.
5. It takes a U-turn into India on reaching Namcha Barwa.

Select the correct answer using the codes below.


a) 1, 2, 3 and 4 only
b) 2, 3 and 5 only
c) 1, 2, 4 and 5 only
d) 1, 3, 4 and 5 only

Solution: C
Justification: It flows eastwards parallel to the Himalayas. On reaching the Namcha Barwa (7757 m), it takes a
‘U’ turn and enters India in Arunachal Pradesh through a gorge. Here, it is called the Dihang and it is joined by
the Dibang, the Lohit, the Kenula and many other tributaries to form the Brahmaputra in Assam. In Tibet the
river carries a smaller volume of water and less silt as it is a cold and a dry area. In India it passes through a
region of high rainfall. Here the river carries a large volume of water and considerable amount of silt. The
Brahmaputra has a braided channel in its entire length in Assam and forms many riverine islands.

168. Main tributaries of Kaveri are


1. Bhavani
2. Amravati
3. Brahmani
4. Baitarini

Select the correct answer using the codes below.


a) 1 and 2 only
b) 2 and 3 only
c) 3 and 4 only
d) 1 and 4 only

Solution: A
Justification: The Kaveri rises in the Brahmagiri range of the Western Ghats and it reaches the Bay of Bengal
in south of Cuddalore, in Tamil Nadu. Total length of the river is about 760 km. Its main tributaries are
Amravati, Bhavani, Hemavati and Kabini. Its basin drains parts of Karnataka, Kerala and Tamil Nadu. The river

Direct PDF - https://telegram.me/UPSC_PrelimsTest


https://telegram.me/UPSC_PDF

OGP 2019 GEOGRAPHY MODULE MCQS

Kaveri makes the second biggest waterfall in India. It is known as Sivasamudram. The Damoder, the
Brahmani, the Baitarni and the Subarn rekha are major rivers flowing towards east, not tributaries.

169. Why is sedimentation harmful for reservoirs?


1. It results in rockier stream beds.
2. A lot of sedimentation may cause local flooding.
3. It adversely affects the reservoir bed habitats for the aquatic life.

Select the correct answer using the codes below.


a) 1 only
b) 2 only
c) 2 and 3 only
d) 1, 2 and 3

Solution: D
Justification: Statement 1 and 2: All rivers contain sediments. When a river is stilled behind a dam, the
sediments it contains sink to the bottom of the reservoir. As the sediments accumulate in the reservoir, so
the dam gradually loses its ability to store water for the purposes for which it was built. If sedimentation
continues, it increases water level and can even cause floods.

170. How would you usually distinguish between a Himalayan and a Peninsular River?
1. Length of course
2. Depth of course
3. Gradient of flow
4. Sediment load
Select the correct answer using the codes below.
a) 1, 3 and 4 only
b) 2 and 3 only
c) 1 and 4 only
d) D. 1, 2, 3 and 4

Solution: D
Justification: Many peninsular rivers flow over hard rocky surfaces, where bed or lateral erosion is not
significant, leading to lower sediment load. On the other hand, in Himalayan Rivers, course is deeper,
gradient higher and course longer, which results in greater sediment load. Himalayan Rivers have many other
depositional features in their floodplains. They also have well- developed Deltas. Most of the rivers of
peninsular India originate in the Western Ghats and flow towards the Bay of Bengal. Thus, a large number of
the Peninsular Rivers are seasonal, as their flow is dependent on rainfall. During the dry season, even the
large rivers have reduced flow of water in their channels.

171. Consider the following matches of rivers with their origin locations.
1. Pennar : Coorg
2. Cauvery: Kolar
3. Krishna: Mahabaleshwar
4. Godavari: Nashik

Which of the above is/are correct matches?


a) 1 and 2 only
b) 3 and 4 only
c) 1 and 4 only
d) 2 and 3 only

Solution: B

Direct PDF - https://telegram.me/UPSC_PrelimsTest


https://telegram.me/UPSC_PDF

OGP 2019 GEOGRAPHY MODULE MCQS

Justification: The image below is a comprehensive source of information.

CLIMATE, VEGETATION AND SOIL


CLIMATE

172. Relief plays a major role in determining the climate of a place in which of the following ways?
1. High mountains act as barriers for cold or hot winds.
2. High relief may cause precipitation if they lie in the path of rain-bearing winds.
Which of the above is/are correct?
a) 1 only
b) 2 only
c) Both 1 and 2
d) None

Solution: C
Justification: Statement 1: Winds are an important factor in changing the climate of a place, if hot or cold
winds cannot reach a region, there are likely to be less variations in the weather of the place.
Statement 2: This happens with Western Ghats very often, the windward side of the mountain receives very
high rainfall, whereas the leeward side remains dry.
There are six major controls of the climate of any place. They are: latitude, altitude, pressure and wind
system, distance from the sea (continentality), ocean currents and relief features.

173. With reference to Nor Westers local wind, consider the following statements.
1. With this shower, coffee flowers blossom in Kerala and nearby areas.
2. Locally, they are also known as mango showers since they help in the early ripening of mangoes.

Direct PDF - https://telegram.me/UPSC_PrelimsTest


https://telegram.me/UPSC_PDF

OGP 2019 GEOGRAPHY MODULE MCQS

Which of the above is/are correct?


a) 1 only
b) 2 only
c) Both 1 and 2
d) None

Solution: D
Justification: Some Famous Local Storms of Hot Weather Season:
 Nor Westers: These are dreaded evening thunderstorms in Bengal and Assam. Their notorious nature can
be understood from the local nomenclature of ‘Kalbaisakhi’, a calamity of the month of Baisakh. These
showers are useful for tea, jute and rice cultivation. In Assam, these storms are known as “Bardoli
Chheerha”.
 Blossom Shower: With this shower, coffee flowers blossom in Kerala and nearby areas.
 Mango Shower: Towards the end of summer, there are pre-monsoon showers which are a common
phenomenon in Kerala and coastal areas of Karnataka. Locally, they are known as mango showers since
they help in the early ripening of mangoes.
 Loo: Hot, dry and oppressing winds blowing in the Northern plains from Punjab to Bihar with higher
intensity between Delhi and Patna.

174. As per the Koeppen classification, which of the following regions of India has a Steppe Climate?
a) Central India
b) Parts of North-western India and leeward side of Western Ghats
c) North-eastern India
d) Coastal regions of south-eastern India

Solution: B
Justification: Steppe climate will have lower and irregular rainfall. North-eastern India receives either
abundant rainfall or gets too cold for being called a steppe climate.

Direct PDF - https://telegram.me/UPSC_PrelimsTest


https://telegram.me/UPSC_PDF

OGP 2019 GEOGRAPHY MODULE MCQS

175. Consider the following barchart of precipitation and temperature of a particular climate type

Direct PDF - https://telegram.me/UPSC_PrelimsTest


https://telegram.me/UPSC_PDF

OGP 2019 GEOGRAPHY MODULE MCQS

a) Equatorial Climate
b) Tropical Continental Climate
c) Mediterranean climate
d) Marine West Coast climate

Solution: B
Justification: Option B: Places with this type of climate are located in the centre of continents approximately
between latitudes 50 and 150 north and south of the Equator. It is also found on the higher land Of the East
African Plateau which straddles the Equator, for e.g. northern Nigeria.
The climate has two distinct seasons.
 A very warm. dry season when conditions are similar to those of the hot desert
 A hot, wet season when the weather more resembles that of equatorial areas
Option C: Mediterranean climate is characterized by hot, dry summers and cool, wet winters and located
between about 30° and 45° latitude north and south of the Equator and on the western sides of the
continents.
Option D: This features cool summers (relative to their latitude) and cool winters, with a relatively narrow
annual temperature range. The curve above shows warm summers

176. The climate and associated weather conditions in India are governed by
1. Pressure and surface winds
2. Upper air circulation
3. Tropical easterly jet stream

Select the correct answer using the codes below.


a) 1 only
b) 2 and 3 only
c) 3 only
d) All of the above

Solution: D

Direct PDF - https://telegram.me/UPSC_PrelimsTest


https://telegram.me/UPSC_PDF

OGP 2019 GEOGRAPHY MODULE MCQS

Justification: Consider this - during winter, there is a high-pressure area north of the Himalayas. During
summer, there is a low pressure trough in the Northern plains. Winds blow and fill the depression also
bringing monsoon. So, clearly statement 1 is correct.
For statement 2- jet streams are an important component of upper air circulation which is dominated by
westerly flow. These jet streams blow south of Himalayas, all through the year except in summer affecting
India’s climate and weather. So, statement 2 is also correct. Also, the western (not eastern) cyclonic
disturbances experienced in the north and north-western parts of the country are brought in by this westerly
flow which affects rainfall in Northern and north-western India. In summer, the subtropical westerly jet
stream moves north of the Himalayas with the apparent movement of the sun. An easterly jet stream, called
the tropical easterly jet stream blows over peninsular India, approximately over 14°N during the summer
months. So, statement 3 is also correct.

177. Westerlies come all the year round and there is a tendency towards an autumn or winter maximum of
rainfall. Light snow falls in winter. Ports are never frozen, but frosts do occur on cold nights. The seasons are
very distinct. The type of climate described here is
a) Marine West Coast climate
b) Moist Continental Mid-latitude Climate
c) Humid sub-tropical climate
d) British type climate

Solution: D
Justification: The cool temperate western margins are under the influence of the Westerlies all-round the
year. They are the regions of frontal cyclonic activity [Temperate Cyclones]. This type of climate is typical to
Britain, hence the name ‘British Type’. Also called as North-West European Maritime Climate due to greater
oceanic influence. Most pronounced in and around Britain, in Europe the climate extends inland some 2,000
km. Climatic belt stretches far inland into the lowlands of North-West Europe (northern and western France,
Belgium, the Netherlands, Denmark, western Norway and also north-western Iberia).

178. Consider the following regions of India and the climate type associated with the same.
1. Arunachal Pradesh: Cold Humid Winter with Short summer
2. Rainshadow Region of Western Ghats: Steppe Type climate
3. Monsoon with dry season in Summer: Central India
4. Monsoon type with dry Winter: Western Ghats
Select the correct answer using the codes below.
a) 1 and 2 only
b) 1, 2 and 4 only
c) 1, 2, 3 and 4
d) 3 and 4 only

Solution: A
Justification: The map below is very important and should be understood well and memorised.

Direct PDF - https://telegram.me/UPSC_PrelimsTest


https://telegram.me/UPSC_PDF

OGP 2019 GEOGRAPHY MODULE MCQS

179. Why a desert cooler cools better on a hot dry day?


1. Low humidity helps in better evaporation
2. Air pressure is higher on a hotter day as compared to a colder day
3. Latent heat of vaporization is higher for water on a drier day

Select the correct answer using the codes below.


a) 1 only
b) 2 and 3 only
c) 1 and 3 only
d) 2 only

Solution: A
Justification: A desert cooler cools better on a hot dry day because on a hot dry day temperature is high and
humidity is less which helps in better evaporation. Due to the higher rate of evaporation it gives better
cooling effect. The cooling in a desert room cooler is due to the process of evaporation.

180. When it is summers in velds, what season would it be in the prairies?

a) Summer season

Direct PDF - https://telegram.me/UPSC_PrelimsTest


https://telegram.me/UPSC_PDF

OGP 2019 GEOGRAPHY MODULE MCQS

b) Autumn season
c) Spring season
d) Winter Season

Solution: D
Justification: Velds lie in the Southern hemisphere, and prairies lie in the Northern Hemisphere. This is
because Southern hemisphere (Velds) faces the Sun whereas Northern Hemisphere comparatively receives
lesser insolation. Conditions change once earth changes position and moves towards summer solstice (for
Northern Hemisphere).

181. What are the consequences of the thin air found at high altitudes in Ladakh?
1. Heat of the sun is felt much lesser as compared to places at same latitude but lower altitude
2. Frost bite is rarest in this region due to presence of thin air mass.

Which of the above is/are correct?


a) 1 only
b) 2 only
c) Both 1 and 2
d) None

Solution: D
Justification: Due to its high altitude, the climate is extremely cold and dry. The air at this altitude is so thin
that the heat of the sun can be felt intensely. The area experiences freezing winds and burning hot sunlight.
The day temperatures in summer are just above zero degree and the night temperatures well below 30°C. It
is freezing cold in the winters when the temperatures may remain below 40°C for most of the time. You will
be surprised to know that if you sit in the sun will your feet in the shade, you may suffer from both sunstroke
and frost bite at the same time.

MONSOON

182. Why the Tamil Nadu coast remains dry during the South-west Monsoon season?
1. The Tamil Nadu coast is situated parallel to the Bay of Bengal branch of southwest monsoon.
2. Cold currents appear in South Indian coasts during northwards movement of the Monsoon.
3. Monsoon winds strike the coast and reroute due to the voluminous spits and barriers in those coasts.

Select the correct answer using the codes below.


a) 1 only
b) 1 and 3 only
c) 2 and 3 only
d) 1 and 2 only

Solution: A
Justification: There are two branches of Monsoon in India – one the Bay of Bengal side and another over the
Arabian side. Its coast is situated parallel to the Bay of Bengal branch of southwest monsoon, so monsoon
winds do not strike it and instead pass over. Also, it lies in the rainshadow area of the Arabian Sea branch of
the south-west monsoon. Since it is bereft of the moisture from both the branches it remains dry. It instead
receives rainfall from the north-eastern monsoon that come with a retreat.

183. Consider the following areas in India and the rainfall associated with them.
1. Western coast close to the Western Ghats: A. Low Rainfall
2. Western Uttar Pradesh B. High rainfall
3. Northeastern Peninsula - Orissa and Jharkhand: C. Medium rainfall

Direct PDF - https://telegram.me/UPSC_PrelimsTest


https://telegram.me/UPSC_PDF

OGP 2019 GEOGRAPHY MODULE MCQS

Select the correct answer using the codes below.


a) 1A, 2B, 3C
b) 1B, 2A, 3C
c) 1C, 2A, 3B
d) 1B, 2C, 3A

Solution: B
Justification: Areas of High Rainfall: The highest rainfall occurs along the west coast, on the Western Ghats, as
well as in the sub-Himalayan areas is the northeast and the hills of Meghalaya. Here the rainfall exceeds 200
cm. In some parts of Khasi and Jaintia hills, the rainfall exceeds 1,000 cm. In the Brahmaputra valley and the
adjoining hills, the rainfall is less than 200 cm.
Areas of Medium Rainfall: Rainfall between 100-200 cm is received in the southern parts of Gujarat, east
Tamil Nadu, northeastern Peninsula covering Orissa, Jharkhand, Bihar, eastern Madhya Pradesh, northern
Ganga plain along the sub-Himalayas and the Cachar Valley and Manipur.
Areas of Low Rainfall: Western Uttar Pradesh, Delhi, Haryana, Punjab, Jammu and Kashmir, eastern
Rajasthan, Gujarat and Deccan Plateau receive rainfall between 50-100 cm.
Areas of Inadequate Rainfall: Parts of the Peninsula, especially in Andhra Pradesh, Karnataka and
Maharashtra, Ladakh and most of western Rajasthan receive rainfall below 50 cm.

184. Consider the following statements.


1. Usually, when the tropical eastern south Pacific Ocean experiences high pressure, the tropical eastern
Indian Ocean experiences high pressure.
2. When the El-Nino appears the pressure difference between Pacific Ocean (Tahiti) and Indian Ocean
(Northern Australia) undergoes a reversal from the normal conditions.

Which of the above is/are correct?


a) 1 only
b) 2 only
c) Both 1 and 2
d) None

Solution: B
Justification: Statement 1: Both experience low pressure or high pressure together. This is the normal
situation.
Statement 2: But in certain years, there is a reversal in the pressure conditions and the eastern Pacific has
lower pressure in comparison to the eastern Indian Ocean. This periodic change in pressure conditions is
known as the Southern Oscillation or SO. The difference in pressure over Tahiti (Pacific Ocean, 18°S/149°W)
and Darwin in northern Australia (Indian Ocean, 12°30’S/131°E) is computed to predict the intensity of the
monsoons. If the pressure differences were negative, it would mean below average and late monsoons. A
feature connected with the SO is the El Nino, a warm ocean current that flows past the Peruvian Coast, in
place of the cold Peruvian current, every 2 to 5 years. The changes in pressure conditions are connected to
the El Nino. Hence, the phenomenon is referred to as ENSO (El Nino Southern Oscillations).

185. With reference to Jet streams, consider the following statements.


1. They are easterly winds of the stratosphere.
2. These blow at a constant speed throughout the year.
3. They are limited to tropical latitudes with occasional movements in the sub-tropics.

Select the correct answer using the codes below.


a) 1 and 2 only
b) 2 only
c) 2 and 3 only
d) None of the above

Direct PDF - https://telegram.me/UPSC_PrelimsTest


https://telegram.me/UPSC_PDF

OGP 2019 GEOGRAPHY MODULE MCQS

Solution: D
Justification: Jet streams are a narrow belt of high altitude (above 12,000 m) westerly winds in the
troposphere. Their speed varies from about 110 km/h in summer to about 184 km/h in winter. A number of
separate jet streams have been identified. The most constant are the mid-latitude and the sub-tropical jet
stream. Importance: These winds play an important role in affecting the climate in the Indian sub- continent.
The western cyclonic disturbances experienced in the north and north-western parts of the country are
brought in by the subtropical westerly jet stream flow. These cause rainfall in the Northern part of India in the
winter season.

186. The Indian Monsoon is affected by which of these factors?


1. The shift of the position of Inter Tropical Convergence Zone (ITCZ) in summer from the Ganga plain to
Southern India
2. The presence of the high-pressure area, east of Madagascar
3. Heating of the Tibetan plateau during summer
4. Presence of the tropical easterly jet stream over the Indian peninsula during summer

Select the correct answer using the codes below.


a) 1, 2 and 3 only
b) 2, 3 and 4 only
c) 1 and 4 only
d) D. 1, 2, 3 and 4

Solution: B
Justification: The differential heating and cooling of land and water creates low pressure on the landmass of
India while the seas around experience comparatively high pressure.
Statement 1: The shift of the position of Inter Tropical Convergence Zone (ITCZ) in summer, over the Ganga
plain (this is the equatorial trough normally positioned about 5°N of the equator – also known as the
monsoon trough during the monsoon season attracts the monsoon and affects its strength.
Statement 2: The presence of the high-pressure area, east of Madagascar, approximately at 20°S over the
Indian Ocean. The intensity and position of this high-pressure area affects the Indian Monsoon.
Statement 3: The Tibetan plateau gets intensely heated during summer, which results in strong vertical air
currents and the formation of high pressure over the plateau at about 9 km above sea level. This also affects
the strength of monsoon.
Statement 4: The movement of the westerly jet stream to the north of the Himalayas and the presence of the
tropical easterly jet stream over the Indian peninsula during summer is another factor that changes wind flow
and affects monsoon’s strength.

187. With reference to the Indian sub-continent climate conditions, consider the following statements.
Western Cyclonic Disturbances
1. Occur mainly in winter months
2. Influence the Southern part of the country the most
3. Causes tropical cyclones in the coastal belts of the country
Select the correct answer using the codes below.
a) 1 only
b) 2 and 3 only
c) 1 and 3 only
d) None of the above

Solution: A
Justification: Statement 1: The western cyclonic disturbances are weather phenomena of the winter months
brought in by the westerly flow from the Mediterranean region.
Statement 2: They usually influence the weather of the north and north-western regions of India.

Direct PDF - https://telegram.me/UPSC_PrelimsTest


https://telegram.me/UPSC_PDF

OGP 2019 GEOGRAPHY MODULE MCQS

Statement 3: Tropical cyclones occur during the monsoon as well as in October -November, and are part of
the easterly flow. These disturbances affect the coastal regions of the country.

188. Monsoon breaks in India can be attributed to


1. Appearance of cold currents on the Western flank of the Indian Peninsula
2. Occurrence of Western Tropical Cyclones coming from the Arabian Sea

Which of the above is/are correct?


a) 1 only
b) 2 only
c) Both 1 and 2
d) None

Solution: D
Justification: During the south-west monsoon period after heavy rains for a few days, if rain fails to occur for
one or more weeks, it is known as break in the monsoon.
Statement 1: These breaks in rainfall are related to the cyclonic depressions mainly formed at the head of the
Bay of Bengal, and their crossing into the mainland. Besides the frequency and intensity of these depressions,
the passage followed by them determines the spatial distribution of rainfall. Moreover, these breaks in the
different regions are due to different reasons:
 In northern India rains are likely to fail if the rain-bearing storms are not very frequent along the monsoon
trough or the ITCZ over this region.
 Over the west coast the dry spells are associated with days when winds blow parallel to the coast.
Statement 2: Western Tropical Cyclones originate from the Mediterranean Sea, and they usually withdraw
with the onset of the Monsoon.

189. Among the following, last part of India to receive the South-west Monsoon is
a) Ahmedabad
b) Nagpur
c) Bhubaneswar
d) Hyderabad

Solution: A
Justification: Monsoon winds moves from Southern and eastern India towards northern and Western India.
So, the latter would receive the winds and precipitation later.

Direct PDF - https://telegram.me/UPSC_PrelimsTest


https://telegram.me/UPSC_PDF

OGP 2019 GEOGRAPHY MODULE MCQS

190. The spatial distribution of Monsoon is uneven in the country. What do you think can be the typical
estimate in which it varies?
a) 12 cm to more than 250 cm
b) 50 cm to nearly 200 cm
c) 70 cm to about 150 cm
d) 5 cm to nearly 500 cm

Solution: A
Justification: The north-western and some of the Northern regions receive the lowest rainfall, and the north-
eastern and south-western regions receive the highest. Apart from this variation that significantly affects
weather and climate over India, monsoons play a pivotal role in the agrarian economy of India because over
three-fourths of the total rain in the country is received during the southwest monsoon season.

Direct PDF - https://telegram.me/UPSC_PrelimsTest


https://telegram.me/UPSC_PDF

OGP 2019 GEOGRAPHY MODULE MCQS

191. Abnormal accumulation of cold water occurring in the central and eastern Pacific Ocean and the
strengthening of Tropical pacific trade winds is an unusual occurrence associated with
a) El Nino
b) Indian Ocean Dipole
c) La Nina
d) Either (a) or (b)

Solution: C
Justification: La Niña refers to the extensive cooling of the central and eastern tropical Pacific Ocean, often
accompanied by warmer than normal sea surface temperatures (SSTs) in the western Pacific, and to the north
of Australia. Changes to the atmosphere and ocean circulation during La Niña events include:
 Sustained cooler-than-usual SSTs across the central and eastern tropical Pacific Ocean.
 Increased convection or cloudiness over tropical Australia, Papua New-Guinea, and Indonesia.
 An increase in strength of the trade winds (easterlies) across the tropical Pacific Ocean (but not
necessarily in the Australian region).

192. In the North Indian Ocean, there is a complete reversal of the direction of ocean currents between
summer and winter. What can be a reason for this?
1. Westerly winds succumb to the Easterly flow coming from the South East Asian nations.

Direct PDF - https://telegram.me/UPSC_PrelimsTest


https://telegram.me/UPSC_PDF

OGP 2019 GEOGRAPHY MODULE MCQS

2. Monsoon winds change direction from South-west to North- east.

Which of the above is/are correct?


a) 1 only
b) 2 only
c) Both 1 and 2
d) None

Solution: B
Justification: The currents of the Indian Ocean are modified by the landmass in the north and influenced by
the Monsoon winds. In the North Indian Ocean, a complete reversal of direction of Monsoon Current, twice a
year is observed between summer and winter because the Monsoon winds change their direction according
to seasons. South-West Monsoon Current (warm) -> in the summer season, the monsoon winds blow from
south west to north-east. Consequently, the ocean current also flows from south-west to north-east
direction. The main current moves in the Arabian Sea and the Bay of Bengal along the coast of India.
North-East Monsoon Current (warm) -> in the winter season, the monsoon winds blow from north-east to
south-west. The ocean current also flows from north-east to south-west under the influence of these winds.

193. In India, the day temperature is maximum in May and not after the summer solstice. This is due to
1. Indian Ocean Dipole and La Nino
2. Southwest monsoons occurring after Summer Solstice
3. Passing of Western cyclonic disturbances
Select the correct answer using the codes below.
a) 2 only
b) 2 and 3 only
c) 3 only
d) 1 only

Solution: A
Justification: Statement 1: The Indian Ocean Dipole (IOD), also known as the Indian Nino, is an irregular
oscillation of sea-surface temperatures. The IOD also affects the strength of monsoons over the Indian
subcontinent. But, this is not the causal factor.
Statement 2: The southwest monsoons arrive at the Kerala coast and advance further inland in June and July.
These winds have cooling effect and thus summer solstice in June is cooler than May.
Statement 3: Western disturbances largely affect India in the winter season.

194. Consider the following statements.


1. North-East Monsoon or the retreating monsoon occurs when
2. the Sun appears to retreat to the South after the monsoon season
3. the northern land mass of the Indian subcontinent begins to cool off
4. it is dry season in the Southern belt of the country
Select the correct answer using the codes below.
a) 1 and 2 only
b) 2 and 3 only
c) 1 and 3 only
d) 1, 2 and 3

Solution: A
Justification: Northeast Monsoon season is the major period of rainfall activity over south peninsula
comprising of the Coastal Andhra Pradesh, Rayalaseema and Tamilnadu-Pondicherry. For Tamilnadu this is
the main rainy season accounting for about 48% of the annual rainfall.
Statement 1 and 2: Around September, with the sun fast retreating south, the northern land mass of the
Indian subcontinent begins to cool off rapidly. With this air pressure begins to build over northern India, the

Direct PDF - https://telegram.me/UPSC_PrelimsTest


https://telegram.me/UPSC_PDF

OGP 2019 GEOGRAPHY MODULE MCQS

Indian Ocean and its surrounding atmosphere still holds its heat. This causes cold wind to sweep down from
the Himalayas and Indo-Gangetic Plain towards the vast spans of the Indian Ocean south of the Deccan
peninsula. This is known as the Northeast Monsoon or Retreating Monsoon.

195. By the end of December


1. Air pressure in Southern India is higher as compared to Northern India
2. Sun shines vertically over the Tropic of Capricorn in the southern hemisphere
Which of the above is/are correct?
a) 1 only
b) 2 only
c) Both 1 and 2
d) None

Solution: B
Justification: Statement 1: The weather in this season is characterised by feeble high pressure conditions over
the northern plain. In south India, the air pressure is slightly lower. As a result, winds start blowing from
north-western high pressure zone to the low air pressure zone over the Indian Ocean in the south.
Statement 2: In the summer, sun is overhead Tropic of cancer in Northern Hemisphere.

196. The north-east monsoon, commonly known as winter monsoon blows from
a) Sea to land
b) Plains to mountains
c) Upper air circulation
d) Land to Sea

Solution: D
Justification: The climate of India may be broadly described as tropical monsoon type. It is affected by two
seasonal winds - the northeast monsoon and the southwest monsoon. The north-east monsoon, commonly
known as winter monsoon blows from land to sea, whereas south-west monsoon, known as summer
monsoon blows from sea to land after crossing the Indian Ocean, the Arabian Sea, and the Bay of Bengal. The
south-west monsoon brings most of the rainfall during a year in the country.

197. Which of the following describe most accurately the character of precipitation in India?
a) Rainfall is dispersed equally over the year.
b) Most of the rainfall is concentrated in few months of time.
c) Rainfall period is highly uneven in India, with sudden spurts occurring at non-definite times.
d) There is no clear pattern of precipitation in India.

Solution: B
Justification: Monsoonal showers supply over 80% of India's annual rainfall. Some of the states that do not
receive this rainfall get rainfall by retreating Monsoon, like TN, parts of AP. Some states in the North-east see
rainfall for most of the year. But, the larger pattern of precipitation in India is monsoonal.

VEGETATION

198. With reference to Mangroves, consider the following statements.


1. It is the only species of tree which flourishes in sea water.
2. It provides a vital structure to coastlines by their roots that trap sediments.
3. It protects vulnerable coastal communities from the impact of waves.
4. It acts as a net carbon sink.

Select the correct answer using the codes below.

Direct PDF - https://telegram.me/UPSC_PrelimsTest


https://telegram.me/UPSC_PDF

OGP 2019 GEOGRAPHY MODULE MCQS

a) 2 and 3 only
b) 1, 3 and 4 only
c) 2 and 4 only
d) D. 1, 2, 3 and 4

Solution: D
Justification: Mangrove trees possess several ingenious and unique adaptations making them one of the
most important trees on Earth:
 They regenerate quickly from propagules.
 They provide vital structure to coastlines, their arching roots trap sediment that would otherwise be
washed back by the waves.
 Act as a barrier between sea and land, protecting vulnerable coastal communities from the impact of
waves.
 Mangroves and their root systems provide habitats for; bats, spawning fish, lobsters, manatees and birds.
According to a status report of the Government of India publication, the total area of the mangroves in India,
was reckoned at about 6,740 km2 which has now reduced to 4,474 km2 which may be due to several reasons
such as
 grazing by domestic cattle and exploitation of mangrove woods for fuel and timber
 the neo-tectonic movement of river courses
 abatement of upstream freshwater discharges due to construction of dams and reservoirs
 rapid trend of reclamation of mangrove forests for habitations
 Pollutant discharges from cities and industries etc.

199. Consider the following about Nilambur teaks.


1. They are found in dry deciduous climate zones.
2. The teak is resistance to fungal decay.
3. The tree has anti-oxidant properties.

Select the correct answer using the codes below.


a) 1 and 2 only
b) 2 and 3 only
c) 3 only
d) 1, 2 and 3 only

Solution: B
Justification: Kerala’s Nilambur teak aka Malabar teak has found its place in the Geographical Indications (GI)
Registry. The golden brown teak is known for its log dimensions, desired wood figure and wide reputation in
the world of trade. Its durability is attributed to the synergistic effect of total extensive components and the
resistance to fungal decay to naphthoquinone and derivatives it contains. The hydrophobicity, anti-oxidant
properties and oily nature are due to a caoutchouc compound. The Nilambur forests are also known for their
tribal settlements, vast rain forests, waterfalls and the world's first Teak Museum.

200. Which of the following conditions are conducive for the luxuriant growth of Rhododendron species?
a) Rainy plains near Coromandel Coast
b) Hot and warm conditions of the Central India
c) Cold, moist slopes and deep valleys of the eastern Himalayas
d) Salt marshes and dry terrain of the Western India

Solution: C
Justification: Found in varied habitats from subtropical forest to alpine shrubs, rhododendrons range from
dwarf shrubs to large trees. North-eastern India, in the Eastern Himalayas, is home to 97% of the
Rhododendron species and sub-species in the country. However, indiscriminate felling and loss of habitat in

Direct PDF - https://telegram.me/UPSC_PrelimsTest


https://telegram.me/UPSC_PDF

OGP 2019 GEOGRAPHY MODULE MCQS

the north-eastern States has left many of these beautiful flowering plants vulnerable to extinction. Arunachal
Pradesh is home to the highest number with 119 taxa of the Rhododendron. It is the national flower of Nepal.
Most species have brightly coloured flowers which bloom from late winter through to early summer. In a bid
to conserve the dwindling rhododendron species of Tawang, a rhododendron park is being established in
Tawang, Arunachal Pradesh. Over 30 species of rhododendron will be planted and conserved in the park. It
will also have a modern nursery, an information centre, resting sheds, and a parking facility, among other
things

201. The total forest and tree cover is nearly what fraction of the geographical area of the country
a) One third
b) One fourth
c) One half
d) One sixth

Solution: B
Justification: The Merck has released India State of Forest Report (ISFR) 2017. Key Findings of ISFR 2017. It
has revealed that total forest and tree cover in India has increased of over 8,021 sq. km (about 80.20 million
hectare) which is one percent increase from 2015. The total forest and tree cover is 24.39% of geographical
area of the country. The increase in forest cover has been observed in Very Dense Forest (VDF) which absorbs
maximum carbon dioxide from the atmosphere. It is followed by increase in forest cover in open forest. India
is ranked 10th in world, with 24.4% of land area under forest and tree cover, even though it accounts for 2.4
% of the world surface area and sustains needs of 17 % of human and 18 % livestock population. India was
placed 8th in list of Top Ten nations reporting the greatest annual net gain in forest area. 15 states/UTs have
above 33% of geographical area under forest cover. About 40% of country’s forest cover is present in 9 large
contiguous patches of the size of 10, 000 sq.km, or more. 7 States/UTs have more than 75% forest cover:
Mizoram, Lakshadweep, Andaman & Nicobar Islands, Arunachal Pradesh, Nagaland, Meghalaya and Manipur

202. Which of these are species of hardwood trees?


1. Birch
2. Mahogany
3. Spruce
4. Redwood
Select the correct answer using the codes below.
a) 1 and 2 only
b) 2, 3 and 4 only
c) 1 and 3 only
d) D. 1, 2, 3 and 4

Solution: A
Justification: Hardwood is wood from dicot trees. These are usually found in broad-leaved temperate and
tropical forests. In temperate and boreal latitudes they are mostly deciduous, but in tropics and subtropics
mostly evergreen. Hardwoods are produced by angiosperm trees that reproduce by flowers, and have broad
leaves.
Statement 2: Mahogany is a tropical hardwood species indigenous to the Americas.
Ash, Alder, Aspen, Balsa, Beech, Birch, Cherry and Chestnut are some other hardwood trees

203. The Terai region is a


a) plain swampy landscape south of Shiwaliks
b) hilly region in North-eastern India
c) riverine island supporting high biodiversity
d) highland east of Tibet Plateau

Solution: A

Direct PDF - https://telegram.me/UPSC_PrelimsTest


https://telegram.me/UPSC_PDF

OGP 2019 GEOGRAPHY MODULE MCQS

Justification: It was in news due to Nepalese constitution issues. The Northern plains can be divided into four
regions – two are discussed here. The rivers, after descending from the mountains deposit pebbles in a
narrow belt of about 8 to 16 km in width lying parallel to the slopes of the Shiwaliks. It is known as bhabar. All
the streams disappear in this bhabar belt. South of this belt, the streams and rivers re-emerge and create a
wet, swampy and marshy region known as terai. This was a thickly forested region full of wildlife. The forests
have been cleared to create agricultural land and to settle migrants from Pakistan after partition

204. Which of the following characterize Boreal forests.


1. It occurs at high altitudes.
2. It contains a thin cover of acidic soil that is poor in nutrients.
3. It contains coniferous forests of regions such as North America and Northern Europe.
Select the correct answer using the codes below.
a) 1 only
b) 2 and 3 only
c) 1 and 2 only
d) 1, 2 and 3

Solution: D
Justification: The Boreal Forest is the term applied to the huge area of dense coniferous forests of North
America, northern Europe and Asia occurring at high latitudes where the climate is characterised by very cold
winters, usually about 40cm of rainfall and a short summer growing season. The dominant species are
evergreens, such as pine, spruce, fir, whose water loss through transpiration is low in winter at the time when
the soils may be permanently frozen. Unlike tropical forests which have a huge range of species, the boreal
forest usually have rather few species of trees, shrubs and plants. The species present are able to get their
nutrient requirements from the very shallow layer of soil above the permafrost. The trees present have a
notable climatic tolerance and survive on rather infertile soils.

205. You are unlikely to find what kind of vegetation in Andaman and Nicobar Islands?
1. Ferns
2. Grasslands
3. Evergreen forests
Select the correct answer using the codes below.
a) 1 only
b) 2 only
c) 1 and 3 only
d) All of the above can be found.

Solution: D
Justification: The South Andaman forests have a profuse growth of epiphytic vegetation, mostly ferns and
orchids.
The Middle Andaman harbours mostly moist deciduous forests. North Andaman is characterized by the wet
evergreen type, with plenty of woody climbers. The North Nicobar Islands (including Car Nicobar and
Battimalv) are marked by the complete absence of evergreen forests, while such forests form the dominant
vegetation in the central and southern islands of the Nicobar group.
Statement 2: Grasslands occur only in the Nicobars, and while deciduous forests are common in the
Andaman, they are almost absent in the Nicobars. The present forest coverage is claimed to be 86.2% of the
total land area.

206. Deciduous forests are least likely and evergreen forests most likely, to be found in
a) Central India
b) Eastern India
c) Gangetic plains
d) South-western coast, India

Direct PDF - https://telegram.me/UPSC_PrelimsTest


https://telegram.me/UPSC_PDF

OGP 2019 GEOGRAPHY MODULE MCQS

Solution: D
Justification: Option A: The region is relatively drier and more likely to have deciduous forests.
Option B and C: The Upper Gangetic Plains moist deciduous forests is a tropical moist broadleaf forest eco-
region of northern India. The Lower Gangetic plains moist deciduous forests is a tropical moist broadleaf
forest eco-region of Bangladesh and eastern India.
Option D: This is a region of high rainfall and thus more likely to hold evergreen forests.

207. In India, in which one of the following types of forests is Mulberry a dominant tree species?
a) Tropical moist deciduous forest
b) Tropical rain forest
c) Tropical thorn scrub forest
d) Temperate forest with grasslands

Solution: A
Justification: Tropical moist deciduous forests exist mostly in the eastern part of the country – northeastern
states, along the foothills of the Himalayas, Jharkhand, West Orissa and Chhattisgarh, and on the eastern
slopes of the Western Ghats. Teak is the most dominant species of this forest. Bamboos, sal, shisham,
sandalwood, khair, kusum, arjun, mulberry are other commercially important species.

208. Soil that is acidic, poor in nutrients with thin cover can be found in
1. Boreal forests
2. Tropical Deciduous forests
3. Temperate forests

Select the correct answer using the codes below.


a) 1 only
b) 2 and 3 only
c) 1 and 2 only
d) None of the above

Solution: A
Justification: The Boreal Forest is the term applied to the huge area of dense coniferous forests of North
America, northern Europe and Asia occurring at high latitudes where the climate is characterised by very cold
winters, usually about 40cm of rainfall and a short summer growing season. The dominant species are
evergreens, such as pine, spruce, fir, whose water loss through transpiration is low in winter at the time when
the soils may be permanently frozen. Unlike tropical forests which have a huge range of species, the boreal
forest usually have rather few species of trees, shrubs and plants. In fact the undergrowth of boreal forests is
almost non-existent. The species present are able to get their nutrient requirements from the very shallow
layer of soil above the permafrost. The trees present have a notable climatic tolerance and survive on rather
infertile soils. Pines survive on very sandy soils like those in northern Europe whereas spruce requires more
loamy and clayey soils with more nutrients.

209. These regions are hot and receive heavy rainfall throughout the year. As there is no particular dry season,
the trees do not shed their leaves altogether. This description matches
a) Temperate Deciduous forests
b) Tropical Evergreen Forests
c) Mediterranean Vegetation
d) Monsoon forests

Solution: B
Justification: Forests found near the Equator (10° North to 10° South) in Amazon & Zaire Basin and Southeast
Asia are major evergreen forests. The thick canopies of the closely spaced trees do not allow the sunlight to

Direct PDF - https://telegram.me/UPSC_PrelimsTest


https://telegram.me/UPSC_PDF

OGP 2019 GEOGRAPHY MODULE MCQS

penetrate inside the forest even in the day time. Thus, grass is not found in these forests. Maximum varieties
of trees are found. Trees are tall with large trunks. Hardwood trees like rosewood, teak, sal, ebony, and
mahogany are the common trees found here. Here the population found is very less. Due to dense forests
commercial exploitation of these forests has not be possible, making them economically backward.

210. In which of the following states/UTs you are most likely to find tropical forests as well as sub-tropical
forests, bamboo forests, pine forests, temperate forests and alpine forests all?
a) Kerala
b) Arunachal Pradesh
c) Karnataka
d) Mountain ranges and plains of Andaman

Solution: B
Justification: The vegetation of Arunachal Pradesh falls under four broad climatic categories and can be
classified in five broad forest types with a sixth type of secondary forests. These are tropical forests, sub-
tropical forests, pine forests, temperate forests and alpine forests.
Forest Cover Map of Arunachal Pradesh

Vegetation in Assam: may broadly be divided into (I) Evergreen forests, (II) Mixed Deciduous forests, (III)
Riverain forests and (IV) Savannah.
Alpine forests: The Eastern Himalayan alpine shrub and meadows extend along the north and south faces of
the Himalaya Range from the Kali Gandaki Gorge in central Nepal eastwards through Tibet and India's Sikkim
state, Bhutan, Arunachal Pradesh state, and northernmost Myanmar.

211. The growth of vegetation in a region depends on which of these factors?


1. Temperature
2. Moisture
3. Slope of Soil
4. Thickness of soil
5. Altitude

Select the correct answer using the codes below.


a) 1, 2 and 5 only
b) 2, 4 and 5 only
c) 1, 3 and 4 only
d) 1, 2, 3, 4 and 5

Solution: D

Direct PDF - https://telegram.me/UPSC_PrelimsTest


https://telegram.me/UPSC_PDF

OGP 2019 GEOGRAPHY MODULE MCQS

Justification: Statement 1 and 2: For e.g. Tundra is an area where tree growth is difficult because of cold
temperatures and short seasons. Vegetation in tundra is limited to a few shrubs, grasses, and mosses.
Statement 3: Slope not only determines the stability of soil (for vegetation growth) but also the ability of soil
to retain and hold water. On too steep slopes, water may not be allowed to seep down resulting in drier soils
and poor vegetation.
Statement 4: Thicker the soil, greater are the chances of finding minerals and organic matter in the soil.
Statement 5: With the change in height, the climate changes and that changes natural vegetation.

212. In India, the shola ecosystem is characterised by


1. Arid and semi-arid climate zones
2. Growth of trees in the depressions and folds of valleys surrounded by extensive areas of grasslands
3. Dominance of conifers in areas of dense vegetation

Select the correct answer using the codes below.


a) 2 only
b) 1 and 3 only
c) 1 only
d) 1, 2 and 3

Solution: A
Justification: These are essentially southern wet temperate forests which are found in Kerala along the crest
of Western Ghats in Palakkad, Wayanad, Idukki and Thiruvananthapuram districts. Grasslands constitute
about 80 per cent of such forests. It is found in Indira Gandhi National Park, Annamalai etc.

213. Forests are broadly classified depending on when they shed their leaves. Shedding of leaves is dependent
on which of these factors?
1. Availability of moisture
2. Latitudinal variation

Which of the above is/are correct?


a) 1 only
b) 2 only
c) Both 1 and 2
d) None

Solution: C
Justification: Statement 1: Evergreen forests do not shed their leaves simultaneously in any season of the
year. Deciduous forests shed their leaves in a particular season to conserve loss of moisture through
transpiration.
Statement 2: This is responsible for the seasonal variation which affects the timing when trees shed their
leaves.

214. Forests are further classified as tropical or temperate based on their location in different latitudes.
Consider the following climatic characteristics
1. 1. Temp. 20-30° C
2. Evenly distributed rainfall
3. Well-defined seasons and distinct winter
4. Fertile enriched soil with decaying litter

The above points out to which of these forest regions?


a) Tropical Equatorial
b) Tropical Deciduous
c) Temperate

Direct PDF - https://telegram.me/UPSC_PrelimsTest


https://telegram.me/UPSC_PDF

OGP 2019 GEOGRAPHY MODULE MCQS

d) Boreal

Solution: C
Justification: Option A: Temperature is a little higher but the seasons are not well defined. Moreover, the soil
is not fertile due to persistent leaching owing to high rainfall.
Option B: Temperature is higher in deciduous forests and rainfall is not evenly distributed. Option D: These
are tundra forests.

SOILS

215. Consider the following statements.


Assertion (A): Adding limestone to the soil frees up some of the soil minerals making them available for
absorption into the plant.
Reason (R): Calcium carbonate content of the limestone is capable of neutralising some of the acid in the soil.
In the context of the above, which of these is correct?
a) A is correct, and R is an appropriate explanation of A.
b) A is correct, but R is not an appropriate explanation of A.
c) A is correct, but R is incorrect.
d) Both A and R are incorrect.

Solution: A
Justification: Limestone deposits with a calcium carbonate content around 90% can be quarried to be used by
the farming sector. By crushing the limestone, various grades (based on particle size) can be produced. These
products are marketed as ‘agricultural lime. Soil acidity is one of the factors that can influence plant growth
and can seriously limit crop production. By spreading agricultural lime onto the paddock or soil, the calcium
carbonate content of the limestone is capable of neutralising some of the acid in the soil. This also has the
effect of freeing up some of the soil minerals, such as phosphates, and making them available for absorption
into the plant.

216. The alluvial soils


1. Vary in nature from sandy loam to clay
2. Are generally rich in potash but poor in phosphorous
3. Found usually in mountainous regions
4. Are always deposited within the flood plains

Select the correct answer using the codes below.


a) 1 and 2 only
b) 1, 2 and 4 only
c) 1 and 3 only
d) 2, 3 and 4

Solution: A
Justification: The characteristics of alluvial soils will be clear from the distinction between khadar and bangar
soils. In the Upper and Middle Ganga plain, two different types of alluvial soils have developed, viz. Khadar
and Bhangar. Khadar is the new alluvium and is deposited by floods annually, which enriches the soil by
depositing fine silts. Bhangar represents a system of older alluvium, deposited away from the flood plains.
Both the Khadar and Bhangar soils contain calcareous concretions (Kankars). These soils are more loamy and
clayey in the lower and middle Ganga plain and the Brahamaputra valley. The sand content decreases from
the west to east. The colour of the alluvial soils varies from the light grey to ash grey. Its shades depend on
the depth of the deposition, the texture of the materials, and the time taken for attaining maturity.

Direct PDF - https://telegram.me/UPSC_PrelimsTest


https://telegram.me/UPSC_PDF

OGP 2019 GEOGRAPHY MODULE MCQS

217. Consider the following statements.


1. Alkaline soils are difficult to take into agricultural production due to their high infiltration rate that does
not allow water to stand.
2. Waterlogging of the soil prevents air from getting inside resulting in a decline in the productivity of the
land.

Which of the above is/are correct?


a) 1 only
b) 2 only
c) Both 1 and 2
d) None

Solution: B
Justification: Statement 1: Alkaline soils are difficult to take into agricultural production. Due to the low
infiltration capacity, rain water stagnates on the soil easily and, in dry periods, cultivation is hardly possible
without copious irrigated water and good drainage.
Statement 2: Soil may be regarded as waterlogged when it is nearly saturated with water much of the time
such that its air phase is restricted and anaerobic conditions prevail. Various crops need air (specifically,
oxygen) to a greater or lesser depth in the soil. Waterlogging of the soil stops air getting in and reduces the
produce.

218. The rate of water percolation will be highest in which of the following soil types?
a) Loamy soil
b) Clayey soil
c) Alluvial Soil
d) Sandy Loam

Solution: D
Justification: Sandy loam soil is suitable for the growth of cotton plants as it can easily drain water and is well
aerated. Clayey soil is best suited for paddy, as it can retain water for a longer time. This is also suitable for
wheat and grams. Loamy soil is considered the best for almost all types of crops. It is suitable for lentils and
other pulses.

219. According to the weight composition of soil, which of the following weighs the least when the soil is
unsaturated?
a) Organic matter
b) Minerals
c) Water
d) Microorganisms

Solution: D
Justification:

Direct PDF - https://telegram.me/UPSC_PrelimsTest


https://telegram.me/UPSC_PDF

OGP 2019 GEOGRAPHY MODULE MCQS

220. In India, you are most likely to find Laterite soils, suitable for tree crops like cashewnut, in the region of
a) Western Ghats
b) Eastern Ghats
c) Ganges Basin
d) North-western India

Solution: A
Justification: The laterite soils develop in areas with high temperature and high rainfall. These are the result
of intense leaching due to tropical rains. With rain, lime and silica are leached away, and soils rich in iron
oxide and aluminium compound are left behind. Humus content of the soil is removed fast by bacteria that
thrives well in high temperature. These soils are poor in organic matter, nitrogen, phosphate and calcium,
while iron oxide and potash are in excess. Hence, laterites are not suitable for cultivation; however,
application of manures and fertilisers are required for making the soils fertile for cultivation. Red laterite soils
in Tamil Nadu, Andhra Pradesh and Kerala are more suitable for tree crops like cashewnut.

Direct PDF - https://telegram.me/UPSC_PrelimsTest


https://telegram.me/UPSC_PDF

OGP 2019 GEOGRAPHY MODULE MCQS

221. The factor that affects the rate of humus formation in soil the most is?
a) Minerals in the soil
b) Flora, fauna and microorganisms
c) Characteristics of parent rock
d) Slope of soil

Solution: B
Justification: Soil organic matter is made up of organic compounds and includes plant, animal and microbial
material, both living and dead. A small part of the organic matter consists of the living cells such as bacteria,
molds, and actinomycetes that work to break down the dead organic matter. Were it not for the action of
these micro-organisms, the entire carbon dioxide part of the atmosphere would be sequestered as organic
matter in the soil.

NATURAL HAZARDS AND DIASASTER: CAUSES CONSEQUENCES AND MANAGEMENT


CYCLONE

222. Though a tropical cyclone typically moves from east to west in the tropics, its track may shift pole ward
and eastward, called as “recurvature”. This may occur due to the influence of
1. Jet streams
2. Extra tropical cyclones

Direct PDF - https://telegram.me/UPSC_PrelimsTest


https://telegram.me/UPSC_PDF

OGP 2019 GEOGRAPHY MODULE MCQS

Which of the above is/are correct?


a) 1 only
b) 2 only
c) Both 1 and 2
d) None

Solution: C
Justification: Its track may shift pole ward and eastward either as it moves west of the subtropical ridge axis
or else if it interacts with the mid-latitude flow, such as the jet stream or an extra tropical cyclone. This
motion, termed "recurvature", commonly occurs near the western edge of the major ocean basins, where the
jet stream typically has a pole ward component and extra tropical cyclones are common. An example of
tropical cyclone recurvature was Typhoon Ioke in 2006.

223. How do tropical cyclones play an important role in modulating regional and global climate?
1. They carry heat energy away from the tropics and transport it toward temperate latitudes.
2. They can often bring intense rainfall to drought prone regions.
Which of the above is/are correct?
a) 1 only
b) 2 only
c) Both 1 and 2
d) None

Solution: C
Justification: Statement 2: Tropical cyclones can relieve drought conditions as they often carry moisture
laden winds. In addition to strong winds and rain, tropical cyclones are capable of generating high waves,
damaging storm surge, and tornadoes. They typically weaken rapidly over and where they are cut off from
their primary energy source. For this reason, coastal regions are particularly vulnerable to damage from a
tropical cyclone as compared to inland regions. Heavy rains. However, can cause significant flooding inland,
and storm surges can produce extensive coastal flooding up to 25 miles from the coastline.
Statement 1: They also carry heat energy away from the tropics and transport it toward temperate latitudes,
because they are large wind systems that absorb the frontal conditions of the area they flow in. Finally, due
to the high speed they blow in, they also act as the earth's filter system by cleansing the air of toxins and
pollutants out of the atmosphere.

224. Consider the following: Tropical cyclones are


1. intense low-pressure areas
2. largely confined to the areas lying between 5° N and 5° S latitudes
3. like a heat engine that is energised by the release of latent heat on account of the condensation of wind
moisture

Select the correct answer using the codes below.


a) 1 only
b) 1 and 2 only
c) 1 and 3 only
d) 2 and 3 only

Solution: C
Justification: Statement 2: They lie between 30° N and 30° S latitudes, in the atmosphere around which high
velocity winds blow. They are not found close to the equator where the Coriolis force is absent. Horizontally,
it extends up to 500-1,000 km and vertically from surface to 12-14 km.
Statement 3: A tropical cyclone or hurricane is like a heat engine that is energised by the release of latent
heat on account of the condensation of moisture that the wind gathers after moving over the oceans and
seas. Tropical cyclones are characterised by large pressure gradients. The centre of the cyclone is mostly a

Direct PDF - https://telegram.me/UPSC_PrelimsTest


https://telegram.me/UPSC_PDF

OGP 2019 GEOGRAPHY MODULE MCQS

warm and low-pressure, cloudless core known as eye of the storm. Generally, the isobars are closely placed
to each other showing high-pressure gradients. Normally, it varies between 14-17mb/100 km, but sometimes
it can be as high as 60mb/100km. Expansion of the wind belt is about 10-150 km from the centre.

225. Some initial conditions for the emergence of a tropical cyclone include
1. Strong Coriolis force
2. Large-scale movement of wind from land to the seas
3. Cold weather for frontal formation
Select the correct answer using the codes below.
a) 1 and 2 only
b) 1 only
c) 2 and 3 only
d) 1 and 3 only

Solution: B
Justification: There are differences of opinion among scientists about the exact mechanism of a tropical
cyclone. However, some initial conditions for the emergence of a tropical cyclone are:
 Large and continuous supply of warm and moist air that can release enormous latent heat.
 Strong Coriolis force that can prevent filling of low pressure at the centre (absence of Coriolis force near
the equator prohibits the formation of tropical cyclone between 0°-5° latitude).
 Unstable condition through the troposphere that creates local disturbances around which a cyclone
develops.
 Finally, absence of strong vertical wind wedge, which disturbs the vertical transport of latent heat.

226. Consider the following statements. A typhoon


1. is a temperate cyclone
2. Develops mainly between 0 degrees longitude to 20 degrees longitude east of Greenwich
3. Accounts for nearly eighty per cent of all cyclones in the world
Select the correct answer using the codes below.
a) 1 only
b) 2 and 3 only
c) 1 and 3 only
d) None of the above

Solution: D
Justification: Statement 1: A typhoon is a mature tropical cyclone that develops between 180° and 100°E.
This region is referred to as the North-western Pacific Basin. It is the most active tropical cyclone basin on
Earth, accounting for almost one-third of the world’s annual tropical cyclones.
 A typhoon differs from a cyclone or hurricane only on the basis of location.
 A hurricane is a storm that occurs in the Atlantic Ocean and north-eastern Pacific Ocean,
 A typhoon occurs in the north western Pacific Ocean,
 A cyclone occurs in the south Pacific or Indian Ocean.

227. Which of the following will increase the strength of a cyclone?


a) Proximity to land surfaces
b) Heat released from condensation of water vapour
c) Absence of cloud formation
d) A decline in the temperature of sea water

Solution: B
Justification: Option A Land surfaces break cyclone as supply of moisture is cut-off.
Option B Before cloud formation, water takes up heat from the atmosphere to change into vapour. When
water vapour changes back to liquid form as raindrops, this heat is released to the atmosphere. The heat

Direct PDF - https://telegram.me/UPSC_PrelimsTest


https://telegram.me/UPSC_PDF

OGP 2019 GEOGRAPHY MODULE MCQS

released to the atmosphere warms the air around. The air tends to rise and causes a drop in pressure. More
air rushes to the centre of the storm. This cycle is repeated. The chain of events ends with the formation of a
very low- pressure system with very high-speed winds revolving around it. It is this weather condition that we
call a cyclone. Factors like wind speed, wind direction, temperature and humidity contribute to the
development of cyclones.
Option C Absence of clouds reduces the strength of a cyclone. The process of formation of clouds involves
evaporation of water, and then rainfall. As explained below, rainfall will involve condensation which will
enhance the strength of a cyclone.
Option D Lowering temperature of sea water will reduce formation of water vapour and increase the
pressure of the eye of cyclone. This will reduce the strength instead of increasing it

228. Consider the following statements.


Assertion (A): Tropical cyclones are not formed at the equator.
Reason (R): Coriolis force is absent at the equator.
In the context of the above, which of these is correct?
a) A is correct, and R is an appropriate explanation of A.
b) A is correct, but R is not an appropriate explanation of A.
c) A is correct, but R is incorrect.
d) Both A and R are incorrect.

Solution: A
Justification: Coriolis force is absent at the equator. Due to this, winds blowing from high pressure sub-tropics
to low pressure tropics do not form a circulatory pattern around a low-pressure zone. Instead, they simply fill
the low-pressure zone. This is the reason cyclones start forming only after some distance away from the
equator.

229. Frontal cyclones are most likely to occur in


a) Tropical regions
b) Equatorial regions
c) Polar regions
d) Mid-latitude regions

Solution: D
Justification: Mid-latitude or frontal cyclones are large travelling atmospheric cyclonic storms up to 2000 km.
in diameter with centres of low atmospheric pressure. Mid-latitude cyclones are the result of the dynamic
interaction of warm tropical and cold polar air masses at the polar front. This interaction causes the warm air
to be cyclonically lifted vertically into the atmosphere where it combines with colder upper atmosphere air.
This process also helps to transport excess energy from the lower latitudes to the higher latitudes.
Option A: In tropical regions, tropical cyclones are more common. So, (a) is less appropriate than (d).

230. Consider the following statements.


1. Tropical storms bring rain, while extra-tropical storms do not.
2. Extra-tropical storms are caused due to jet streams, whereas tropical cyclone is caused due to lower
atmospheric circulations.
Which of the above is/are correct?
a) 1 only
b) 2 only
c) Both 1 and 2
d) None

Solution: B
Justification: Extra tropical storms are a global, rather than a localized, phenomena with moisture usually
carried in the upper atmosphere (unlike tropical storms where it is carried in the lower atmosphere). In the

Direct PDF - https://telegram.me/UPSC_PrelimsTest


https://telegram.me/UPSC_PDF

OGP 2019 GEOGRAPHY MODULE MCQS

case of the subcontinent, moisture is sometimes shed as rain when the storm system encounters the
Himalayas. Jet streams are an important component of upper air circulation which is dominated by westerly
flow. These jet streams blow south of Himalayas, all through the year except in summer affecting India’s
climate and weather. The western cyclonic disturbances experienced in the north and north-western parts of
the country are brought in by this westerly flow which affects rainfall in Northern and north-western India

231. Consider the following about Anticyclones.


1. These are low pressure regions.
2. Their extent is always small.
3. Cloudy and precipitation conditions exist along with the cyclone.
Select the correct answer using the codes below.
a) 1 and 2 only
b) 2 only
c) 3 only
d) 1, 2 and 3

Solution: C
Justification: Statement 1: An anti-cyclone -- also known as a high pressure area -- is a large atmospheric
circulation system with the wind flowing clockwise around it in the Northern Hemisphere, and counter-
clockwise in the Southern Hemisphere.
Statement 2: The extents are generally large and the effects are also pronounced.
Statement 3: Anticyclones form from air masses cooling more than their surroundings, which causes the air to
contract slightly making the air denser. Since dense air weighs more, the weight of the atmosphere overlying
a location increases, causing increased surface air pressure. Winds are of moderate speed, and at the outer
edges, cloudy and precipitation conditions exist. Please see this http://tinyurl.com/j5edaou
Cloudy conditions and precipitation exists at the periphery of anticyclones. Otherwise, such a high pressure
region generally shows fine and clear weather.

EARTHQUAKE AND TSUNAMI

232. The Ring of Fire does NOT pass through which of the following countries?
a) Japan
b) India
c) Sri Lanka
d) USA

Solution: C
Justification: The Ring of fire stretches from the southern tip of South America, up along the coast of North
America, across the Bering Strait, down through Japan, and into New Zealand. Sri Lanka lies outside the ring.
The southern portion is more complex, with a number of smaller tectonic plates in collision with the Pacific
plate from the Mariana Islands, the Philippines, Bougainville, Tonga etc.

Direct PDF - https://telegram.me/UPSC_PrelimsTest


https://telegram.me/UPSC_PDF

OGP 2019 GEOGRAPHY MODULE MCQS

233. Consider the following statements.


Assertion (A): Shallow‐focus earthquakes occur along transform boundaries.
Reason (R): Transform faults are found where plates slide past one another.
In the context of the above, which of these is correct?
a) A is correct, and R is an appropriate explanation of A.
b) A is correct, but R is not an appropriate explanation of A.
c) A is correct, but R is incorrect.
d) Both A and R are incorrect.

Solution: A
Justification: Most earthquakes occur at the boundaries where the plates meet. In fact, the locations of
earthquakes and the kinds of ruptures they produce help scientists define the plate boundaries. There are
three types of plate boundaries: spreading zones, transform faults, and subduction zones. Transform Plate
Boundaries are locations where two plates slide past one another. The fracture zone that forms a transform
plate boundary is known as a transform fault. Most transform faults are found in the ocean basin and connect
offsets in the mid-ocean ridges. A smaller number connect mid-ocean ridges and subduction zones. At
spreading zones, molten rock rises, pushing two plates apart and adding new material at their edges.
Earthquakes are located along the normal faults that form the sides of the rift or beneath the floor of the rift.

234. Consider the following statements.


Assertion (A): The distribution of earthquakes provides information about magma pathways and the structure
of volcanoes.
Reason (R): The world’s distribution of earthquakes coincides very closely with that of volcanoes.
In the context of the above, which of these is correct?
a) A is correct, and R is an appropriate explanation of A.
b) A is correct, but R is not an appropriate explanation of A.
c) A is correct, but R is incorrect.
d) Both A and R are incorrect.

Solution: A
Justification: Some, but not all, earthquakes are related to volcanoes. For example, most earthquakes are
along the edges of tectonic plates. This is where most volcanoes are too. However, most earthquakes are
caused by the interaction of the plates not the movement of magma. Most earthquakes directly beneath a
volcano are caused by the movement of magma. The magma exerts pressure on the rocks until it cracks the
rock. Then the magma squirts into the crack and starts building pressure again. Every time the rock cracks it
makes a small earthquake. These earthquakes are usually too weak to be felt but can be detected and

Direct PDF - https://telegram.me/UPSC_PrelimsTest


https://telegram.me/UPSC_PDF

OGP 2019 GEOGRAPHY MODULE MCQS

recorded by sensitive instruments. Once the plumbing system of the volcano is open and magma is flowing
through it, constant earthquake waves, called harmonic tremor, are recorded (but not felt).

235. Most volcanoes and earthquakes in the world are located at


a) Littoral zones of major oceans
b) Within the plates
c) The inter-junction of major mountains of the world
d) Plate margins

Solution: D
Justification: Plate margins witness several plate collisions, sliding, transformation etc. that result into
volcanism or earthquakes. Most of them are found in the Ring of Fire. Some earthquakes also occur within
the plates but not as frequently as on the plate margins. The mechanism is explained by the plate tectonics
theory.

236. Why are the states of Jammu and Kashmir, Himachal Pradesh and Uttaranchal highly vulnerable to
earthquakes?
1. The Indian plate is slowly moving towards the north and north-eastern direction which is obstructed by
the Eurasian plate.
2. Most of the active volcanoes in India lie in the Northern region of the country which are frequently
thrusted by magma and seismic movements.
Which of the above is/are correct?
a) 1 only
b) 2 only
c) Both 1 and 2
d) None

Solution: A
Justification: The Indian plate is moving at a speed of one centimetre per year towards the north and north-
eastern direction and this movement of plates is being constantly obstructed by the Eurasian plate from the
north. As a result of this, both the plates are said to be locked with each other resulting in accumulation of
energy at different points of time. Excessive accumulation of energy results in building up of stress, which
ultimately leads to the breaking up of the lock and the sudden release of energy causes earthquakes along
the Himalayan arch. Some of the most vulnerable states are Jammu and Kashmir, Himachal Pradesh,
Uttaranchal, Sikkim, and the Darjeeling and subdivision of West Bengal and all the seven states of the
northeast.

237. The impact of a tsunami is less over the ocean and more near the coast because
1. Tides are not as powerful near the coast as in Open Ocean.
2. Wavelength of tsunami water reduces as it approaches the coast.
3. Seismic forces are absent near coasts.
Select the correct answer using the codes below.
a) 1 only
b) 2 and 3 only
c) 2 only
d) 1 and 3 only

Solution: C
Justification: Earthquakes and volcanic eruptions that cause the sea-floor to move abruptly resulting in
sudden displacement of ocean water in the form of high vertical waves are called tsunamis (harbour waves)
or seismic sea waves. Normally, the seismic waves cause only one instantaneous vertical wave; but, after the
initial disturbance, a series of afterwaves are created in the water that oscillate between high crest and low
trough in order to restore the water level.

Direct PDF - https://telegram.me/UPSC_PrelimsTest


https://telegram.me/UPSC_PDF

OGP 2019 GEOGRAPHY MODULE MCQS

Statement 3: They will be present wherever there is a play of tectonics, i.e. plate movements. Coastal belts
are no exception.
Statement 2: The speed of wave in the ocean depends upon the depth of water. It is more in the shallow
water than in the ocean deep. As a result of this, the impact of tsunami is less over the ocean and more near
the coast where they cause large-scale devastations. Therefore, a ship at sea is not much affected by tsunami
and it is difficult to detect a tsunami in the deeper parts of sea. It is so because over deep water the tsunami
has very long wave- length and limited wave-height. Thus, a tsunami wave raises the ship only a metre or two
and each rise and fall takes several minutes. As opposed to this, when a tsunami enters shallow water, its
wave-length gets reduced and the period remains unchanged, which increases the wave height. Sometimes,
this height can be up to 15m or more, which causes large-scale destructions along the shores. Thus, these are
also called Shallow Water Waves. Tsunamis are frequently observed along the Pacific ring of fire, particularly
along the coast of Alaska, Japan, Philippines, and other islands of Southeast Asia, Indonesia, Malaysia,
Myanmar, Sri Lanka, and India etc.

238. In India, regions that could be characterized as “Least Damage Risk Regions” concerning tropical cyclones
are:
1. Rain shadow area of Western Ghats
2. Tripura, Mizoram and adjoining areas bordering Bangladesh
3. Areas near the Bhitarkanika Mangroves
Select the correct answer using the codes below.
a) 1 only
b) 1 and 2 only
c) 1 and 3 only
d) 2 and 3 only

Solution: A
Justification: Statement 1 and 2: Please see the map below. Statement 3: It falls in Odisha which comes under
a high risk zone. Owing to its peninsular shape surrounded by the Bay of Bengal in the east and the Arabian
Sea in the west, the tropical cyclones in India also originate in these two important locations.

Direct PDF - https://telegram.me/UPSC_PrelimsTest


https://telegram.me/UPSC_PDF

OGP 2019 GEOGRAPHY MODULE MCQS

239. Among the following, which region is the least earthquake prone?
a) Deccan plateau
b) Siliguri Corridor
c) Gulf of Kutch region
d) Terai region

Solution: A
Justification: Siliguri corridor and Terai region fall near the edge of the Indian plate along the Himalayas, so
they have to be earthquake prone.

Direct PDF - https://telegram.me/UPSC_PrelimsTest


https://telegram.me/UPSC_PDF

OGP 2019 GEOGRAPHY MODULE MCQS

240. The speed of tsunami waves in Ocean largely depends on


a) Ocean depth
b) Distance from the source of the wave
c) Density of water
d) Distance from mid-oceanic ridges

Solution: A
Justification: Tsunamis are giant waves caused by earthquakes or volcanic eruptions under the sea. Out in the
depths of the ocean, tsunami waves do not dramatically increase in height. But as the waves travel inland,
they build up to higher and higher heights as the depth of the ocean decreases. It depends on Ocean depth
rather than the distance from the source of the wave. Tsunami waves may travel as fast as jet planes over
deep waters, only slowing down when reaching shallow waters.

241. Why the eastern and western boundaries of the Pacific Ocean experience frequent earthquakes?

Direct PDF - https://telegram.me/UPSC_PrelimsTest


https://telegram.me/UPSC_PDF

OGP 2019 GEOGRAPHY MODULE MCQS

a) Due to these margins coinciding with the plate margins.


b) Due to presence of high mountain stretches along the continental belts
c) Due to continuous wave and current action
d) Due to the presence of deep ocean trenches along these margins.

Solution: A
Justification: The region in question is the Ring of Fire. The Ring of Fire is a direct result of plate tectonics/
movement and collisions of lithospheric plates

Since the margins coincide with the plate margins, the continuous tectonic movements cause earthquakes.
This region is also prone to high volcanism.

FLOODS

242. Floods can be caused due to


1. Carrying capacity of the river channels exceeding the surface run-off
2. Considerable reduction in the infiltration rate of soil in regions of high rainfall

Which of the above is/are correct?


a) 1 only
b) 2 only
c) Both 1 and 2
d) None

Solution: B

Direct PDF - https://telegram.me/UPSC_PrelimsTest


https://telegram.me/UPSC_PDF

OGP 2019 GEOGRAPHY MODULE MCQS

Justification: Floods are relatively slow in occurrences and often, occur in well-identified regions and within
expected time in a year. Floods occur commonly when water in the form of surface run-off exceeds the
carrying capacity of the river channels and streams and flows into the neighbouring low-lying flood plains. At
times, this even goes beyond the capacity of lakes and other inland water bodies in which they flow. Floods
can also be caused due to a storm surge (in the coastal areas), high intensity rainfall for a considerably longer
time period, melting of ice and snow, reduction in the infiltration rate and presence of eroded material in the
water due to higher rate of soil erosion.

243. High tide flooding is becoming more frequent in many coastal regions due to
1. Land subsidence
2. Loss of natural coastal barriers
3. Climate change

Select the correct answer using the codes below.


a) 1 and 2 only
b) 3 only
c) 1, 2 and 3
d) 2 and 3 only

Solution: C
Justification: As relative sea level increases, coastal flooding has become more frequent. Flooding now occurs
with high tides in many locations due to climate-related sea level rise, land subsidence, and the loss of natural
barriers. Land subsidence can caused even minor sea level rises to turn into major instance of coastal flooding
if there has been a large loss in natural barriers that prevent sea water intrusion.

The diagram below shows the phenomena in the US.

OTHER

244. Among the following, which of these areas is least susceptible to landslides?
a) Undulated reliefs of Aravali
b) Young mountainous regions of Himalayas
c) Windward sides of Western Ghats
d) Areas near Khasi hills that experience frequent rainfall

Direct PDF - https://telegram.me/UPSC_PrelimsTest


https://telegram.me/UPSC_PDF

OGP 2019 GEOGRAPHY MODULE MCQS

Solution: A
Justification: Two major categories of landslide prone regions can be established – high and low (or
moderate). Highly unstable, relatively young mountainous areas in the Himalayas and Andaman and
Nicobar, high rainfall regions with steep slopes in the Western Ghats and Nilgiris, the north-eastern regions,
along with areas that experience frequent ground-shaking due to earthquakes, etc. and areas of intense
human activities, particularly those related to construction of roads, dams, etc. are included in the high risk
zone.
Moderate to Low Vulnerability Zone : Areas that receive less precipitation such as Trans-Himalayan areas of
Ladakh and Spiti (Himachal Pradesh), undulated yet stable relief and low precipitation areas in the Aravali
(with stable rock structures), rain shadow areas in the Western and Eastern Ghats and Deccan plateau also
experience occasional landslides.
The remaining parts of India, particularly states like Rajasthan, Haryana, Uttar Pradesh, Bihar, West Bengal
(except district Darjeeling), Assam (except district Karbi Anglong) and Coastal regions of the southern States
are safe as far as landslides are concerned.

245. Creation of Urban heat Islands lead to


1. Increased formation of fog and reduced visibility in the city
2. Worsening of air and water quality in urban areas
Which of the above is/are correct?
a) 1 only
b) 2 only
c) Both 1 and 2
d) None

Solution: B
Justification: The urban heat island is a phenomenon when the heat gets trapped near the earth’s surface as
a result of a decline in green cover, rapid urbanisation, energy-intensive activities, and concrete structures.
Statement 2: Urban heat islands can have worse air and water quality than their rural neighbours. UHIs often
have lower air quality because there are more pollutants (waste products from vehicles, industry, and people)
being pumped into the air. These pollutants are blocked from scattering and becoming less toxic by the urban
landscape buildings, roads, sidewalks, and parking lots. Water quality also suffers. When warm water from
the UHI ends up flowing into local streams, it stresses the native species that have adapted to life in a cooler
aquatic environment.
Statement 1: The urban heat island effect is so strong in Delhi, the largest city in the region, that it saw 50%
less fog than surrounding areas. In Delhi, the heat island effect also appears to be suppressing the very
formation of fog. Scientists found that while areas outside Delhi have seen a 20 per cent increase in fog in the

Direct PDF - https://telegram.me/UPSC_PrelimsTest


https://telegram.me/UPSC_PDF

OGP 2019 GEOGRAPHY MODULE MCQS

period 2012-2016 compared with 2000-2004, Delhi itself did not see an increase

246. A Heat wave is considered by the Indian Meteorological Department (IMD) if


a) Maximum temperature of a station reaches at least 40°C or more for Plains
b) Wind velocity exceeds a certain designated limit in the summer season
c) There is no monsoon shower in a particular Meteorological station
d) All of the above occur together

Solution: A
Justification: These conditions are used to categorize heat waves : maximum temperature of a station
reaches at least 40°C or more for Plains, 37°C or more for coastal stations and at least 30°C or more for Hilly
regions.
Based on Actual Maximum Temperature (for plains only)
 Heat Wave: When actual maximum temperature ≥ 45°C
 Severe Heat Wave: When actual maximum temperature ≥47°C
To declare heat wave, the above criteria should be met at least in 2 stations in a Meteorological subdivision
for at least two consecutive days and it will be declared on the second day

247. Consider the following statements about a Cloud Burst.


1. Cloud Burst occurs only in hilly areas.
2. Cumulonimbus clouds are associated with the phenomenon.

Which of the above is/are correct?


a) 1 only
b) 2 only
c) Both 1 and 2
d) None

Solution: B
Justification: Cloud Burst is a localized weather phenomenon representing highly concentrated rainfall over a
small area in a short duration of time

Direct PDF - https://telegram.me/UPSC_PrelimsTest


https://telegram.me/UPSC_PDF

OGP 2019 GEOGRAPHY MODULE MCQS

Statement 1: Hilly areas are more prone, but such cloudbursts can happen in other favourable areas (e.g.
coasts with mountains on the side) where a large amount of moisture is available. The topographical
conditions like steep hills favour the formation of these clouds.
Statement 2: Various researches suggest that they are manifestations of intense vortices on small scale that
generate strong convection currents, which lift the moisture laden air with sufficient rapidity to form
cumulonimbus clouds shedding water load with great strength and ferocity.
There is no satisfactory technique for anticipating the occurrence of cloud bursts because of their small scale.
A network of radars is required to be able to detect the likelihood of a cloud burst but this would be
prohibitively expensive. Only the areas likely to receive heavy rainfall can be identified on a short range scale.

248. Which of these is the correct order of these regions with regard to the incidences of Landslides occurring
in them?

a) Vindhayachal> North-eastern Hills > Himalayas > Western Ghats and the Nilgiris
b) Himalayas > Vindhayachal > Western Ghats and the Nilgiris > North-eastern Hills
c) Western Ghats and the Nilgiris > Vindhayachal> North-eastern Hills > Himalayas
d) Himalayas > North-eastern Hills > Western Ghats and the Nilgiris> Vindhayachal

Solution: D
Justification: Depending on the rock structure, different regions have varying incidences of landslides. Soft
and unconsolidated formations like Himalayas have high to very high incidence of landslides. North-eastern
Hills too have High incidence. Western Ghats and the Nilgiris have moderate to low and Vindhayachal has low
incidence due to the hard and consolidated rock structure. There seems to be a relation between intensity of
rainfall and slope failures in Western Ghats. Almost all mass movements occur during monsoons (SW and NE
monsoon) in the western flank of Western Ghats and during occasional cyclonic events in the eastern flank
indicating that main triggering mechanism is the over- saturation of overburden caused by heavy rains.

INSTITUTIONS

249. National Disaster Management Authority (NDMA), as the apex body for disaster management in India,
has which of the following responsibilities?
1. Approve plans prepared by the Ministries or Departments of the Government of India in accordance with
the National Plan
2. Lay down guidelines to be followed by the different Ministries or Departments of the Government of India
for the Purpose of integrating the measures for prevention of disaster
3. Recommend provision of funds for the purpose of mitigation
4. Provide such support to other countries affected by major disasters as may be determined by the Central
Government
Select the correct answer using the codes below.
a) 1, 2 and 4 only
b) 2, 3 and 4 only
c) 1 and 3 only
d) D. 1, 2, 3 and 4

Solution: D
Justification: It is mandated to lay down the policies, plans and guidelines for Disaster Management to ensure
timely and effective response to disasters. Towards this, it lay down policies on disaster management (apart
from the ones mentioned above); Approve the National Plan; Lay down guidelines to be followed by the State
Authorities in drawing up the State Plan; Coordinate the enforcement and implementation of the policy and
plans for disaster management; Recommend provision of funds for the purpose of mitigation; Take such
other measures for the prevention of disaster, or the mitigation, or preparedness and capacity building for

Direct PDF - https://telegram.me/UPSC_PrelimsTest


https://telegram.me/UPSC_PDF

OGP 2019 GEOGRAPHY MODULE MCQS

dealing with threatening disaster situations or disasters as it may consider necessary; Lay down broad policies
and guidelines for the functioning of the National Institute of Disaster Management.
In 2005, the Government of India enacted the Disaster Management Act, which envisaged the creation of
NDMA, headed by the Prime Minister, and State Disaster Management Authorities (SDMAs) headed by
respective Chief Ministers, to spearhead and implement a holistic and integrated approach to Disaster
Management in India.

250. The Disaster Management Act has made the statutory provisions for constitution of National Disaster
Response Force (NDRF) for the purpose of specialized response to natural and man-made disasters. Which of
the following is correct with regard to the role and functions of NDRF?
1. It imparts basic and operational level training to State Response Forces like Police and Home guards.
2. It is responsible for organizing Public Awareness Campaigns.
3. It is the statutorily sanctioned force for managing armed forces causalities.
Select the correct answer using the codes below.
a) 1 and 2 only
b) 2 and 3 only
c) 1 only
d) None of the above

Solution: A
Justification: Others include:
 Specialized response during disasters.
 Proactive deployment during impending disaster situations.
 Acquire and continually upgrade its own training and skills.
 Liaison, Reconnaissance, Rehearsals and Mock Drills.
 Impart basic and operational level training to State Response Forces (Police, Civil Defence and Home
Guards).
 Community Capacity Building Programme.
A multi-agency exercise was recently conducted on the banks of Hussain Sagar Lake as the final event of
‘Pralay Sahayam’ in Hyderabad. The event demonstrated efforts of all central and state agencies, NDRF and
the Armed Forces towards jointly tackling an urban flooding scenario in Hyderabad.

251. With reference to the National Disaster Response Force (NDRF), consider the following statements:
1. President of India serves as the Director General (DG) of NDRF.
2. NDRF was established under the Disaster Management Act, 2005.
3. It is housed directly under the Prime Minister’s Office and functions from there.
Select the correct answer using the codes below.
a) 2 only
b) 1 and 3 only
c) 1 only
d) 2 and 3 only

Solution: A
Justification: NDRF is India’s elite disaster mitigation combat force established under Disaster Management
Act, 2005.
Statement 1: Senior IPS officer Sanjay Kumar was recently appointed as Director General (DG) of National
Disaster Response Force (NDRF).
Statement 3: It functions under Union Ministry of Home Affairs. It is mandated to undertake special disaster
response, relief, rescue operations and combat roles independently in the case of an event of any disaster
(natural or man-made), accident or emergency. It also assists local authorities in launching a quick rescue and
response operation to save life and property.

Direct PDF - https://telegram.me/UPSC_PrelimsTest


https://telegram.me/UPSC_PDF

OGP 2019 GEOGRAPHY MODULE MCQS

252. With reference to the legal framework of disaster management in India, consider the following
statements.
1. Public liability insurance Act, 1991 provides for mandatory general insurance coverage all families living in
hazardous zones.
2. Food Safety and Standards Act, 2006 provides for quality standards and testing so that food consumption
does not result in mass poisoning.

Which of the above is/are correct?


a) 1 only
b) 2 only
c) Both 1 and 2
d) None

Solution: B
Justification: Statement 1: It provides for insurance to give immediate relief to the persons affected by
accident occurring while handling any hazardous substance and for matters connected therewith or
incidental thereto.
Statement 2: It prohibits manufacture, storage and use of products unsafe of human consumption. Thus, it
covers aspects of disaster management.

HUMAN GEOGRAPHY
POPULATION
DISTRIBUTION

253. The greatest numbers of the world's human population resides in


a) Temperate zone including the subtropics
b) Equator
c) Southern tropical regions
d) Southern frigid zone
Solution: A
Justification: The vast majority of the world's human population resides in temperate zones, especially in the
northern hemisphere, due to its greater mass of land. The temperatures in these regions are generally
relatively moderate, rather than extremely hot or cold, and the changes between summer and winter are also
usually moderate. This is another reason for majority settlements. However, in certain areas, such as Asia and
central North America, the variations between summer and winter can be extreme because these areas are
far away from the sea, causing them to have a continental climate.

254. One way of studying the population composition of a country is by looking at the population pyramid. A
population pyramid shows which of the following information?
1. The total population divided into various age
2. Percentage of the total population subdivided into males and females
3. Gender composition based on age groups
4. Geographical distribution of population

Select the correct answer using the codes below.


a) 1, 2 and 3 only
b) 2 and 4 only
c) 1 and 4 only
d) 2 and 3 only

Direct PDF - https://telegram.me/UPSC_PrelimsTest


https://telegram.me/UPSC_PDF

OGP 2019 GEOGRAPHY MODULE MCQS

Solution: A
Justification: It shows demographic information in age groups, e.g., 5 to 9 years, 10 to 14 years. The
percentage of the total population is further shown subdivided into males and females, in each of those
groups. The shape of the population pyramid tells the story of the people living in that particular country. The
numbers of children (below 15 years) are shown at the bottom and reflect the level of births. The size of the
top shows the numbers of aged people (above 65 years) and reflects the number of deaths. So, the
population pyramid also tells us how many dependents there are in a country.

DENSITY

GROWTH

255. Natural rate of population increase per thousand was highest in India during which of the following
decades?
a) Post-liberalization period
b) Post-Bangladesh war period
c) Decade immediately after India’s independence
d) Post-Kargil war period

Solution: B
Justification: During 1971-1981 natural increase rate per thousand was around 22. It was a period of rapid
growth of population.
 Post-liberalization it hovered around 17-20.
 Immediately after independence it was in the range of 13-19.
 Post-Kargil war it was around 17.
 After 2001 population growth slowed down.
 To sum it percentage terms, during 1921 to 1951 A.D., there was an increase in population by 12% and
from 1951 to 1981, the growth showed an increase by 24.75%.

256. To stabilize India’s population, which of these is the prescribed Total Fertility Rate (TFR)?

a) A number between 3 and 4


b) Any number between 1 and 2, provided the influx of population is stopped
c) A number between 2 and 3, closer to 2
d) No TFR range can stabilize natural population growth rate

Solution: C
Justification: TFR is the number of births attributed to each child bearing woman in the country. A TFR below
2 would tend to reduce population in the long run and a TFR above 3 will cause a population growth spiral.
The Mission Parivar Vikas family planning initiative, recently launched by the government, will bring down the
Total Fertility Rate (TFR) to 2.1, which is when the population starts stabilizing, by the year 2025.
The key strategic focus of this initiative will be on improving access to contraceptives through delivering
assured services, dovetailing with new promotional schemes, ensuring commodity security, building capacity
(service providers), creating an enabling environment along with close monitoring and implementation.

257. Which of these regions is experiencing the highest annual rates of natural increase in the population?

a) West Asia and Africa


b) Eastern South America
c) Northern Australia
d) South Asia

Direct PDF - https://telegram.me/UPSC_PrelimsTest


https://telegram.me/UPSC_PDF

OGP 2019 GEOGRAPHY MODULE MCQS

Solution: A
Justification: Natural rate of population increase would be high in regions where illiteracy, standard of living,
health and education are in poor shape. Moreover, regions with peculiar socio- religious structures, like West
Asia, may experience high population growth rates.

COMPOSITION

258. Consider the following statements.


1. In both urban and rural India, there is a greater proportion of male workers than female workers.
2. Majority of female workers are from rural areas.
Which of the above is/are correct?
a) 1 only
b) 2 only
c) Both 1 and 2
d) None

Solution: C
Justification: Of the total 402 million workers, 275 million are males and 127 million females. This would
mean that 51.7 percent of the total males and 25.6 percent of the total females are workers. The number of
female workers is about less than half the number of male workers. In terms of proportion, 68.4 percent of
the workers are males and 31.6 percent females. Main workers constitute 77.8 percent of the total workers.
The remaining are marginal workers. Among the main workers, female workers, are only 23.3 % and 76.7%
are male workers. Majority of female workers (87.3 percent) are from rural areas. This is also twice that of
male workers, which may be due to their being employed predominantly in activities like cultivation and
agricultural labour. In the urban areas, majority of female workers are engaged in Households industry and
other work. Interestingly, among marginal workers females outnumber the males. In three of the four
categories, viz. cultivators, agricultural labourers and household industries, female marginal workers
outnumber male workers.

259. Ladakh is a strategically important location in Jammu and Kashmir bordering China and peace in the
region is crucial for a stable J&K. Which of these religious communities forms the majority in the region?
a) Muslims
b) Tibetan Buddhists
c) Kashmiri Pandits
d) Zoroastrians

Direct PDF - https://telegram.me/UPSC_PrelimsTest


https://telegram.me/UPSC_PDF

OGP 2019 GEOGRAPHY MODULE MCQS

Solution: A
Justification: Census 2011 data has busted the myth that Ladakh is Buddhist-dominated and also that Jammu
region is entirely Hindu-dominated with only four districts having Hindu-majority. According to Census 2011,
the cold desert region Ladakh has a population of 2,74,289 with 1,27,296 Muslims contributing to 46.40
percent of the population while 1,08,761 Buddhists contributing to 39.65 percent population and 33,223
Hindus making up 12.11 percent population of the region. Some Ladakhi activists have in recent times called
for Ladakh to be constituted as a union territory because of perceived unfair treatment by Kashmir and
Ladakh's cultural differences with predominantly Muslim Kashmir.

260. The population pyramid of a country in which birth and death rates both are high is

a) Broad at the base and rapidly narrows towards the top


b) Symmetrical throughout the vertical axis
c) Narrow at the base and rapidly broadens towards the top
d) Like a perfect rectangle

Solution: A
Justification: We covered the concept in an earlier question in this test. This shape happens because
although, many children are born, a large percentage of them die in their infancy, relatively few become
adults and there are very few old people. This situation is typified by the pyramid shown for Kenya.

URBANIZATION

261. Which of following criteria is most useful to differentiate an Indian urban area from a rural area?
a) Literacy rates
b) Occupational structure
c) Sex ratio
d) Growth rate of population

Direct PDF - https://telegram.me/UPSC_PrelimsTest


https://telegram.me/UPSC_PDF

OGP 2019 GEOGRAPHY MODULE MCQS

Solution: B
Justification: The census of India, 2011 defines urban settlements as “All places which have municipality,
corporation, cantonment board or notified town area committee and have a minimum population of 5000
persons, at least 75 per cent of male workers are engaged in non- agricultural pursuits and a density of
population of at least 400 persons per square kilometres are urban.

262. Consider the following cities.


1. Mumbai
2. Delhi
3. Bangalore
4. Hyderabad
5. Kolkata

Arrange the following cities in decreasing order of their population:


a) A. 12345
b) B. 23451
c) C. 21345
d) D. 12453

Solution: A
Justification: Top 10 cities based on population

263. The term “Sub Urbanisation” refers to


a) A pattern of urbanization where commercial and residential places are completely separated
b) Movement of people from congested urban areas to nearby satellite communities
c) Reduction of population in lower tier cities
d) Movement of population from urban areas to rural areas

Direct PDF - https://telegram.me/UPSC_PrelimsTest


https://telegram.me/UPSC_PDF

OGP 2019 GEOGRAPHY MODULE MCQS

Solution: B

Justification: It is a new trend of people moving away from congested urban areas to cleaner areas outside
the city in search of a better quality of living. Important suburbs develop around major cities and everyday
thousands of people commute from their homes in the suburbs to their work places in the city
Option A is somewhat similar (but not same) to commuter towns. Option C is a trend of concentration in
metropolitan cities.
Option D is called counter-urbanization.

MIGRATION
TYPES

CAUSES

CONSEQUENCES

HUMAN DEVELOPMENT

264. The “Basic Needs Approach” to human development was initially proposed by the International Labour
Organisation (ILO). The approach advocates
a) Higher the level of income, the higher is the level of human development
b) People should be made participants in development not only its passive recipients.
c) Building human capabilities in terms of knowledge and basic capabilities
d) Ignoring human choices and fulfilling basic needs of defined sections

Solution: D
Justification: Basic needs approaches to well-being are concerned with outcomes. They arose out of
recognition that economic approaches to development that focused on incomes and growth were
problematic. A narrow economic understanding sees increasing incomes and faster growth as proxies for
improved well-being. However, basic needs proponents argue that this depends, firstly, on whether income is
spent on satisfying basic needs where there are shortfalls or on luxuries that may ultimately have a
detrimental impact on well-being and, secondly, on how income and the positive welfare benefits of
increased income are distributed between and within different social groups. Basic needs approaches focus
on key indicators that are seen as objective assessments of well-being such as longevity, infant survival, body
mass index, educational attainment, and so on.

HUMAN SETTLEMENTS

LAND RESOURCES AND AGRICULTURE

265. The difference between Mixed cropping and Mixed Farming is that
1. The former involves both growing crops and livestock whereas the later mainly relies on intercropping.
2. The former is done on small tracts of land by marginal farmers whereas the latter is done by large
landholders.
Which of the above is/are correct?
a) 1 only
b) 2 only
c) Both 1 and 2
d) None

Solution: D

Direct PDF - https://telegram.me/UPSC_PrelimsTest


https://telegram.me/UPSC_PDF

OGP 2019 GEOGRAPHY MODULE MCQS

Justification: Mixed farming is growing crops and rearing livestock simultaneously. Mixed cropping is growing
two or more crops simultaneously on the same piece of land, for example, wheat gram, or wheat mustard, or
groundnut sunflower. This reduces risk and gives some insurance against failure of one of the crops. Different
ways of growing crops can be used to give maximum benefit. Depending upon the duration, crop rotation is
done for different crop combinations. The availability of moisture and irrigation facilities decide the choice of
the crop to be cultivated after one harvest. If crop rotation is done properly then two or three crops can be
grown in a year with good harvests.

266. Which one among the following is NOT a good source of nutritional calcium?
a) Rice
b) Ragi
c) Skimmed milk
d) Egg

Solution: A
Justification: S2: Ragi is one of the best non-dairy sources of calcium when compared to any other grains.
According to the National Institute of Nutrition in India, 100 grams of Ragi contains 344 mg calcium.
S1: In comparison, Rice contains only 10 mg of calcium per 100g. Skimmed milk and eggs are good sources of
calcium.

267. Which of the following is the only country in the world to produce all the four known varieties of silk?
a) India
b) China
c) USA
d) Japan

Solution: A
Justification: Today India is the second largest silk producer of raw silk and also has the distinction of being
the world’s largest consumer of pure silk. India is the only country in the world to produce all the four known
varieties of silk including Mulberry, Eri, Tasar and Muga. Mulberry is the largest practiced sericulture industry
accounting for almost 76 per cent of the entire silk production. The country is known the world over for the
exquisite brocade fabrics of Banaras, silks of Karnataka, tie-and–dye and Patola of Gujarat and Rajasthan,
ikats from Orissa, fine Bandhej and temple silks of Kancheepuram and Tanjore, etc., are only a few of the
myriad range of silk weaves, textures and patterns available in India

268. Consider the following statements. Almost all Indian coffee is


1. Shade grown
2. Grown as a monoculture
3. Alkaline in character
Select the correct answer using the codes below.
a) 1 only
b) 1 and 3 only
c) 1 and 2 only
d) 2 and 3 only

Solution: A
Justification: 70% of the coffee farms in India are very small, often less than 10 acres, and frequently inter-
cropped with pepper, cardamom, cinnamon, clove and nutmeg. The cup quality is generally low-acid, mild,
and well balanced, with spicy notes and full body. Currently, India is the sixth largest producer of coffee in the
world, accounting for over four percent of global coffee production. Although India was one of the earliest
growers of Arabica, it now produces a higher percentage of Robusta. And, India may soon become a net
importer of Arabica coffee as the shift in coffee production in India from Arabica to Robusta has been

Direct PDF - https://telegram.me/UPSC_PrelimsTest


https://telegram.me/UPSC_PDF

OGP 2019 GEOGRAPHY MODULE MCQS

dramatic. The rising cost of cultivation of Arabica due to lack of any scientific breakthrough in control of white
stem borer is a major problem.

269. Indian textile industry is predominantly cotton based. Consider the following statements with reference
to cotton.
1. India is the largest producer as well as exporter of cotton yarn.
2. In India, all the cotton is produced either in the Western States or the Central region.
3. China is the biggest importer of raw cotton from India.
Which of the above is/are correct?
a) 2 and 3 only
b) 1, 2 and 3
c) 3 only
d) 1 and 2 only

Solution: C
Justification: Statement 1: India is one of the largest producers as well as exporters of cotton yarn and the
Indian textile industry contributes about 11 percent to industrial production, 14 per cent to the
manufacturing sector, 4 percent to the GDP and 12 per cent to the country's total export earnings. As of
2013-14 data, India has overtaken Italy and Germany, and is now the second largest textile exporter in the
world.
Statement 2: The states of Gujarat, Maharashtra, Andhra Pradesh (AP), Haryana, Punjab, Madhya Pradesh
(MP), Rajasthan, Karnataka and Tamil Nadu (TN) are the major cotton producers in India.
Statement 3: China is the biggest importer of raw cotton from India. The other major cotton importing
countries from India are Bangladesh, Egypt, Taiwan, and Hong Kong among others.

270. With reference to Jute fibre crop in India, consider the following statements.
1. It is cultivated mainly during the winter season.
2. Jute needs a plain alluvial soil and standing water.
3. The eastern and North-eastern belt of India is a major producer of Jute.

Select the correct answer using the codes below.


a) 2 and 3 only
b) 1 and 2 only
c) 1 and 3 only
d) 3 only

Solution: A
Justification: S1 and S2: Jute needs a plain alluvial soil and standing water. The suitable climate for growing
jute (warm and wet) is offered by the monsoon climate, during the monsoon season. Temperatures from
20˚C to 40˚C and relative humidity of 70%–80% are favourable for successful cultivation. Jute requires 5–8 cm
of rainfall weekly, and more during the sowing time. Soft water is necessary for the jute production.
S3: In India, jute is mainly grown in West Bengal, Bihar and Assam. The jute plant is normally harvested when
it is at flowering stage.
Jute fibre is obtained from the stem of the jute plant. It is cultivated during the rainy season. The stems of the
harvested plants are immersed in water for a few days. Jute fibres are composed primarily of the plant
materials cellulose and lignin. It falls into the bast fibre category (fibre collected from the phloem of the
plant).

271. Which of these factors is/are usually touted as major hindrances to agricultural productivity growth in
India?
1. Cropping intensity being less than cent per cent
2. Small size of farm holdings
3. Rain fed irrigation

Direct PDF - https://telegram.me/UPSC_PrelimsTest


https://telegram.me/UPSC_PDF

OGP 2019 GEOGRAPHY MODULE MCQS

Select the correct answer using the codes below.


a) 1 and 2 only
b) 2 and 3 only
c) 1 only
d) 2 only

Solution: B
Justification: Statement 1: Around 51% of India s geographical area is already under cultivation as compared
to 11% of the world average. The present cropping intensity of 136% has registered an increase of only 25%
since independence.
Statement 2: India has the second largest amount of arable land of any country after the U.S. However, the
2001 census found that 80% of farm holdings were less than 2 hectares in size, with 62% averaging less than
half a hectare. This prevents farmers from earning large income and does not give them an incentive to invest
in the farm for greater productivity.
Statement 3: Further, rainfed drylands constitute 65% of the total net sown area. This increases fluctuations
in crop yield depending on monsoon and Rabi seasons rainfall

272. In which of the following regions extensive commercial grain cultivation is practised?
1. Interior parts of Southeast Asia
2. Pampas of Argentina
3. European Steppes
4. Amazon Basin

Select the correct answer using the codes below.


a) 1 and 2 only
b) 2 and 3 only
c) 1 and 3 only
d) 2, 3 and 4 only

Solution: B
Justification: Statement 1 and 4: Gathering is practised in: (i) high latitude zones which include northern
Canada, northern Eurasia and southern Chile; (ii) Low latitude zones such as the Amazon Basin, tropical
Africa, Northern fringe of Australia and the interior parts of Southeast Asia.
Statement 2 and 3: Extensive commercial grain cultivation is best developed in Eurasian steppes, the
Canadian and American Prairies, the Pampas of Argentina, the Velds of South Africa, the Australian Downs
and the Canterbury Plains of New Zealand

Direct PDF - https://telegram.me/UPSC_PrelimsTest


https://telegram.me/UPSC_PDF

OGP 2019 GEOGRAPHY MODULE MCQS

273. In India, majority of growing stock of Bamboo is in the


a) North-eastern region
b) Central India
c) Western India
d) Southern India

Solution: A
Justification:

28% of area and 66% of growing stock of bamboo in NE region. 20% of area and 12% of growing stock in MP
& Chhattisgarh. Image below shows the regional stock of Bamboo in India. It is widely seen as a wood
substitute in India given that we import a lot of wood. Largest use is in scaffolding, second largest in paper,
third largest handicrafts

274. Among the following, the state with the highest cropping intensity is?
a) Andhra Pradesh
b) Maharashtra

Direct PDF - https://telegram.me/UPSC_PrelimsTest


https://telegram.me/UPSC_PDF

OGP 2019 GEOGRAPHY MODULE MCQS

c) Madhya Pradesh
d) Tamil Nadu

Solution: C
Justification: Cropping intensity shows how effectively the net sown area was utilized within a cropping
season. If the net sown area was left empty throughout the season, cropping intensity would be zero. If it was
occupied fully once, it would be hundred per cent. If there are more than one cropping seasons and the same
land was sown multiple times, cropping intensity would be more than hundred per cent.
Andhra Pradesh Cropping Intensity: 122.79 % Madhya Pradesh: 150.66%
Maharashtra: 126.12 %
Tamil Nadu: 113.11%
Large cropping intensity does not mean that net sown area will be large as well.

275. Tea is produced in which of the following regions of India?


1. Terai region bordering Nepal
2. Kerala and Karnataka
3. Assam dooars
4. Cachar Hills
5. Odisha
Select the correct answer using the codes below.
a) 1, 3 and 4 only
b) 1, 2 and 3 only
c) 2, 4 and 5 only
d) 1, 2, 3, 4 and 5

Solution: D
Justification: Statement 1: Assam is the single largest tea producer in India. Tea requires cool climate and well
distributed high rainfall throughout the year for the growth of its tender leaves.

Direct PDF - https://telegram.me/UPSC_PrelimsTest


https://telegram.me/UPSC_PDF

OGP 2019 GEOGRAPHY MODULE MCQS

276. Which of the following are NOT Rabi crops?


1. Cotton
2. Gram
3. Pea
4. Mustard

Choose the correct answer using the codes below:


a) 1 only
b) 2 and 3 only
c) 3 and 4 only
d) 2 and 4 only

Solution: A
Justification: The crops which are sown in the rainy season are called kharif crops. The rainy season in India is
generally from June to September. Paddy, maize, soyabean, groundnut, cotton, etc., are kharif crops. The
crops grown in the winter season are called rabi crops. Their time period is generally from October to March.
Examples of rabi crops are wheat, gram, pea, mustard and linseed.

277. Which one among the following states is the major producer of cotton, groundnut and tobacco in India?
a) Andhra Pradesh

Direct PDF - https://telegram.me/UPSC_PrelimsTest


https://telegram.me/UPSC_PDF

OGP 2019 GEOGRAPHY MODULE MCQS

b) Madhya Pradesh
c) TamilNadu
d) Gujarat

Solution: D
Justification: Gujarat is the main producer of tobacco, cotton, and groundnuts in India. Other major crops
produced are rice, wheat, jowar, bajra, maize, tur, and gram. More than 60% of the population of Gujarat
lives in rural areas and depends for its livelihood on agriculture and the rural non-farm sector that is
interlinked with agriculture.

Direct PDF - https://telegram.me/UPSC_PrelimsTest


https://telegram.me/UPSC_PDF

OGP 2019 GEOGRAPHY MODULE MCQS

278. Mixed farming is the practice of


a) Growing one or more vegetable crops on a large scale for shipment to distant markets
b) Rotating crops at frequent intervals
c) Growing a variety of crops in the same field in the same season
d) Combining crops and livestock farming

Solution: D
Justification: Option (a): It is truck farming. Option (b): is crop rotation.
Option (c): It is similar to mixed cropping. Mixed farming system mixes arable farming with the raising of
livestock at the same time, e.g. poultry, bee-keeping etc. Combining crops and livestock also has the potential
to maintain ecosystem function and health and help prevent agricultural systems from becoming too brittle,
or over connected, by promoting greater biodiversity, and therefore increased capability to absorb shocks to
the natural resource base.

279. The main objectives of tillage in a crop farm are to


1. Evacuate air molecules from the soil for improved water seepage
2. Prepare a good seed bed which helps the germination of seeds
3. Retain the hard pan and reduce soil depth for better absorption of nutrients
Select the correct answer using the codes below.
a) 2 only
b) 2 and 3 only
c) 1 and 3 only
d) 1, 2 and 3

Direct PDF - https://telegram.me/UPSC_PrelimsTest


https://telegram.me/UPSC_PDF

OGP 2019 GEOGRAPHY MODULE MCQS

Solution: A
Justification: Tillage is the mechanical manipulation of soil with tools and implements for obtaining
conditions ideal for seed germination, seedling establishment and growth of crops. Tilth is the physical
condition of soil obtained out of tillage (or) it is the result of tillage. The tilth may be a coarse tilth, fine tilth or
moderate tilth.
Objectives of tillage are:
 To prepare a good seed bed which helps the germination of seeds.
 To create conditions in the soil suited for better growth of crops.
 To control the weeds effectively.
 To make the soil capable for absorbing more rain water.
 To mix up the manure and fertilizers uniformly in the soil.
 To aerate the soil. So, S1 is wrong because without aerating the soil, plants may find it difficult to
exchange gases.
 To provide adequate seed-soil contact to permit water flow to seed and seedling roots.
 To remove the hard pan and to increase the soil depth.
 To achieve these objectives, the soil is disturbed / opened up and turned over.

280. Which of the following crops require moderate temperature and rainfall during growing season and
bright sunshine at the time of harvest?
a) Rice and maize
b) Wheat and maize
c) Wheat and Cotton
d) Jute and Cotton

Solution: B
Justification: Option A: For rice, high rainfall, temperature and clayey soil is required which can hold water.
Option D: For Cotton, light rainfall is needed. For Jute high rainfall and temperature is needed.
Option B: Wheat requires moderate temperature and rainfall during growing season and bright sunshine at
the time of harvest. It thrives best in well drained loamy soil. Wheat is grown extensively in USA, Canada,
Argentina, Russia, Ukraine, Australia and India. In India it is grown in winter. Maize requires moderate
temperature, rainfall and lots of sunshine. It needs well-drained fertile soils. Maize is grown in North America,
Brazil, China, Russia, Canada, India, and Mexico.

281. Millet crop is favoured over other cereal crops due to


1. Longer growing season that increases the ability to manage moisture stresses in soil
2. Being grown for both fodder and food crop at a considerably lower cost than many other cereals
3. Good water-use efficiency and ability to grow in high temperature conditions

Select the correct answer using the codes below.


a) 1 and 2 only
b) 2 and 3 only
c) 3 only
d) 1 and 3 only

Solution: B
Justification: Statement 1 and 3: Millets are important crops in the semiarid tropics of Asia and Africa
(especially in India, Mali, Nigeria, and Niger), with 97% of millet production in developing countries. The crop
is favoured due to its productivity and short growing season under dry, high-temperature conditions. Millet is
a C4 plant which means it has good water-use efficiency and utilizes high temperature and is therefore a
summer crop. A C4 plant uses a different enzyme in photosynthesis from C3 plants and this is why it improves
water efficiency.
Statement 2: In addition to being used for seed, millet is also used as a grazing forage crop. Instead of letting
the plant reach maturity it can be grazed by stock and is commonly used for sheep and cattle. In southern

Direct PDF - https://telegram.me/UPSC_PrelimsTest


https://telegram.me/UPSC_PDF

OGP 2019 GEOGRAPHY MODULE MCQS

Australia millet is used as a summer quality pasture, utilizing warm temperatures and summer storms. Millet
is frost sensitive and is sown after the frost period, once soil temperature has stabilised at 14 °C or more. It is
sown at a shallow depth. Millets are indigenous to many parts of the world. The most widely grown millet is
pearl millet, which is an important crop in India and parts of Africa. Finger millet, proso millet, and foxtail
millet are also important crop species.

282. In India, Jute is largely grown in


a) Eastern and North-eastern regions
b) Western region
c) Northern region
d) South-eastern and South-western regions

Solution: A
Justification: Jute cultivation is mainly concentrated in the eastern and north eastern India while that of
mesta (added to jute) cultivation is spread almost throughout the country. In trade and industry, jute and
mesta crop together known as raw jute as their uses are almost same. The crop can be grown in low, medium
and high land situation, both moisture stress and water stagnating condition. White Jute can be grown
comparatively in low land situation while that of Tossa Jute prefers medium and high land situation. Mesta is
grown in almost all over the country. It being a hardy crop and can tolerate moisture stress; its cultivation is
spread in different agro-climatic situations unlike jute which is concentrated only in eastern and north
eastern States. In a major part of Jute area, particularly in North Bengal, Bihar and North-Eastern States, the
soil is acidic in nature. However, the Jute crop is growing in these areas in existing situation adjusting the crop
sequence.

Direct PDF - https://telegram.me/UPSC_PrelimsTest


https://telegram.me/UPSC_PDF

OGP 2019 GEOGRAPHY MODULE MCQS

283. India is the country to grow all commercially cultivated natural species of
1. Cotton
2. Silk
3. Linen (flax)

Select the correct answer using the codes below.


a) 1 only
b) 2 and 3 only
c) 1 and 2 only
d) 2 only

Solution: C
Justification: Statement 1: All Four species of cultivated Cotton - Gossypium arboreum and herbaceum
(Asian Cotton), G.barbadense (Egyptian cotton) and G. hirsutum (American Upland Cotton) – are all grown in
India.
Gossypium hirsutum represents 88% of the hybrid cotton production in India and all the current BT cotton
hybrids are G.hirsutuim. Most of the country’s cotton production comes from ten cotton growing States,
which can be grouped into three regions viz., Northern Zone comprising Punjab, Haryana and Rajasthan,
Central zone comprising of Gujarat, Maharashtra, Madhya Pradesh & Orissa and Southern Zone comprising of
Andhra Pradesh, Karnataka and Tamil Nadu.

Direct PDF - https://telegram.me/UPSC_PrelimsTest


https://telegram.me/UPSC_PDF

OGP 2019 GEOGRAPHY MODULE MCQS

Statement 2: There are five major types of silk of commercial importance, obtained from different species of
silkworms which in turn feed on a number of food plants. These are:
 Mulberry
 Oak Tasar & Tropical Tasar
 Muga
 Eri
India has the unique distinction of producing all these commercial varieties of silk.
Statement 3: linen fabric or weave is produced of fibre of the flax plant. Its softness, natural origins, durability
and strength, as well as its antifungal and antibacterial properties are the reasons linen fabric is desired.
Vedic literature reveals the usage of “umah” and “ksauma”, both types of linen, to create woven textiles.
Fibre flax (the plant from which linen is derived) is known to have been cultivated in India up to 5000 years
ago, pre dating any evidence of cotton production. The two varieties of Linen are – Fibre flax (mainly grown in
Europe), and seed flax (grown worldwide).

284. Consider the following map of India

The shaded area in the map given above is the major producer of which one of the following?
a) Mustard
b) Cotton
c) Groundnut
d) Rice

Solution: C
Justification: Major Growing States are Gujarat, Andhra Pradesh, Tamil Nadu, Karnataka, Maharashtra,
Rajasthan, Madhya Pradesh, Orissa, and Uttar Pradesh. India is the second largest producer of groundnuts in
the world. Groundnut is the major oil seed crop in India and it plays a major role in bridging the vegetable oil
deficit in the country. Groundnuts in India are available throughout the year due to a two-crop cycle
harvested in March and October. Ground Nuts are important protein crops in India grown mostly under rain-
fed conditions.

285. A system of agriculture where monoculture is practiced on a large area is

a) Subsistence Agriculture
b) Plantation Agriculture
c) Slash and Burn Agriculture
d) Transhumance Agriculture

Solution: B
Justification: Plantation agriculture is a form of commercial farming where crops are grown for profit. Large
land areas are needed for this type of agriculture. Countries that have plantation agriculture usually

Direct PDF - https://telegram.me/UPSC_PrelimsTest


https://telegram.me/UPSC_PDF

OGP 2019 GEOGRAPHY MODULE MCQS

experience high annual temperatures and receive high annual rainfall. Plantations are mainly found in
countries that have a tropical climate.

286. Which of these conditions is important for the proper growth of cotton?
1. Frost-free days and bright sunshine
2. Presence of red and yellow soils
3. Minimum rainfall of 150-200 cm
4. Very low temperature conditions

Select the correct answer using the codes below.


a) 1 only
b) 2, 3 and 4 only
c) 1 and 3 only
d) 2 and 4 only

Solution: A
Justification: Cotton requires high temperature, light rainfall, two hundred and ten frost-free days and bright
sunshine for its growth. It grows best on black and alluvial soils. China, USA, India, Pakistan, Brazil and Egypt
are the leading producers of cotton. In India, it is extensively grown in Punjab, Gujarat, Maharashtra, Madhya
Pradesh, Karnataka, Haryana, Tamil Nadu, and Rajasthan.

287. Why Millets are often considered a solution for food security in many developing economies?
1. They can be grown on less fertile and sandy soils as compared to other grains.
2. Their deep root systems and short life cycles make it possible for millets to grow in places even where the
mean annual precipitation is low.

Which of the above is/are correct?


a) 1 only
b) 2 only
c) Both 1 and 2
d) None

Solution: C
Justification: Statement 1: It is a hardy crop (coarse grains) that needs low rainfall and high to moderate
temperature and adequate rainfall. Jowar, bajra and ragi are millets that are grown in India.
Statement 2: It can be as low as 300mm or 30 cm which is what the driest regions of India (e.g. western
region) experience annually.
Maize requires moderate temperature, rainfall and lots of sunshine. It needs well- drained fertile soils. Maize
is grown in North America, Brazil, China, Russia, Canada, India, and Mexico.

288. Which of these regions of India is most suitable for growing rice for all three cropping seasons – kharif,
rabi and zaid?

a) North-western India
b) Eastern India
c) Southern India
d) Western India

Solution: C
Justification: Climate with large number of days with sunshine and fertile soils permit growing of more than
one crop annually on the same plot. Southern India, being nearer to the equator than other regions of India,
has more even temperature conditions than the rest of India. While rainfall is not evenly distributed

Direct PDF - https://telegram.me/UPSC_PrelimsTest


https://telegram.me/UPSC_PDF

OGP 2019 GEOGRAPHY MODULE MCQS

throughout the year, irrigation structures allow sowing of crops in multiple seasons. In fact, rice is sown in all
three seasons here. It is both a kharif and rabi crop despite the huge water demands of rice.

289. Among the following, which continent has the largest proportion of land as arable land?

a) Europe
b) Australia
c) South America
d) Africa

Solution: A
Justification: Europe may not be leading in terms of the absolute area, but leads in terms of arable area as a
percentage of total land. This does not mean that all arable area is actually being cultivated.

290. What are the agro-climatic conditions that differentiate the production of wheat and rice in India?
1. Wheat thrives well in soils that have the highest water retention capacity, whereas rice requires well
drained soils.
2. Wheat requires moderate temperature and high rainfall, whereas rice requires high temperature and
comparatively lower rainfall.

Which of the above is/are correct?


a) 1 only
b) 2 only
c) Both 1 and 2
d) None

Solution: D
Justification: Statement 1: Rice needs high temperature, high humidity and rainfall. It grows best in alluvial
clayey soil, which can retain water. China leads in the production of rice followed by India,
Statement 2: Wheat requires moderate temperature and rainfall during growing season and bright sunshine
at the time of harvest. It thrives best in well drained loamy soil. Wheat is grown extensively in USA, Canada,
Argentina, Russia, Ukraine, Australia and India. In India, it is grown in winter.

291. India was the top producer of tea for nearly a century, but recently China has overtaken India as the top
tea producer of green tea mainly due to

a) Substantial reduction of coffee production in China

Direct PDF - https://telegram.me/UPSC_PrelimsTest


https://telegram.me/UPSC_PDF

OGP 2019 GEOGRAPHY MODULE MCQS

b) Increased land availability and labour productivity in China


c) China becoming the largest tea consumer in the world recently
d) Sudden climate events in India have disrupted tea production since last decade

Solution: B
Justification: China is the world’s largest tea-producing country with an annual output of 1.4- 1.5 million
tonne (MT) of green tea. India is the leading producer of black tea in the world with a 25% share of the total
production. India consumes 75-80% of its own production.
Option C: India is the world's largest tea-drinking nation. However, the per capita consumption of tea in India
remains a modest 750 grams per person every year due to the large population base and high poverty levels.
Option B: Apart from increased land, labour productivity is also high in China and the wage burden on the tea
sector is lower than in India.

292. With reference to Common Property Land Resources (CPLR) in India, consider the following statements.
1. CPLRs constitute over fifty per cent of country’s land area.
2. CPLRs can be managed solely by the village panchayat.

Which of the above is/are correct?


a) 1 only
b) 2 only
c) Both 1 and 2
d) None

Solution: D
Justification: Statement 2: CPRs are resources accessible to and collectively owned\held\managed by an
identifiable community and on which no individual has exclusive property rights are called common property
resources.
Statement 1: The Percentage of Common Property Land Resources in total geographical area is 15 %.

293. Land use is determined by which of these factors?


1. Topography
2. Availability of water
3. Residing population
4. Climate

Select the correct answer using the codes below.


a) 1 and 2 only
b) 3 and 4 only
c) 2, 3 and 4 only
d) D. 1, 2, 3 and 4

Solution: D
Justification: Statement 1: For e.g. plains are more suitable for establishing industries than hilly areas.
Statement 2: Areas that do not have water may not have industries or habitations.
Statement 3: Large population base will have a different habitation pattern and more intensive land use.
Statement 4: Vegetation and living patterns depend on climate.

294. As per the various existing State Land Record databases, which of these are local units of land
measurement in India?
1. Guntas
2. Sarsaai
3. Biswansi
4. Dugurum

Direct PDF - https://telegram.me/UPSC_PrelimsTest


https://telegram.me/UPSC_PDF

OGP 2019 GEOGRAPHY MODULE MCQS

5. Marla

Select the correct answer using the codes below.


a) 3 and 4 only
b) 1, 2, 3 and 5 only
c) 2, 4 and 5 only
d) D. 1, 2, 3, 4 and 5

Solution: B
Justification: Different traditional units like Kanal, Marla, Cents, Guntas, Bigha, Biswa, Biswansi, and Sarsaai
are used for recording the area of a land parcel. Area units are different within the state and sometimes same
unit may have different conversion factors when used in different states. For example, Bigha is used in
practice in both Assam and also in some states like Delhi, Rajasthan, and Haryana etc. Both the Bigha units
have different conversion factors to hectares. At present, it is represented as a numeric data type up to three
decimal places. It is now recommended to have the area only in Metric Units.

295. Consider the following with reference to rubber production in India.


1. Kerala is the largest producer of natural rubber in India.
2. The government has deregulated synthetic rubber industry in India.
3. As of date, no dedicated scheme or policy has been enacted to develop rubber sector in India.

Select the correct answer using the codes below.


a) 1 only
b) 2 and 3 only
c) 1 and 2 only
d) 1, 2 and 3

Solution: C
Justification: Statement 1: India is the third largest natural rubber producing country of the world, next to
Thailand and Indonesia, producing about 9 per cent of the global output. Kerala is the largest producer,
followed by TN.
Statement 2: So far as artificial/synthetic rubber is concerned, Chemical & Petrochemical Industry is
delicensed and decontrolled and Government acts as facilitator in this sector.
Statement 3: In order to increase production of Natural Rubber in the country the Government through the
Rubber Board is implementing the scheme “Sustainable and Inclusive Development of Natural Rubber Sector”
wherein support is provided for plantation development and extension, strengthening research, technology
upgradation and market development, Human resource development etc.

WATER RESOURCES

296. In India, which among the following are artificial lakes?


1. Bhimtal
2. Barapani
3. Ghoda Katora
4. Hussain Sagar
Select the correct answer using the codes below.
a) 1 and 2 only
b) 3 and 4 only
c) 1, 2 and 3 only
d) 2, 3 and 4 only

Solution: B

Direct PDF - https://telegram.me/UPSC_PrelimsTest


https://telegram.me/UPSC_PDF

OGP 2019 GEOGRAPHY MODULE MCQS

Justification: Statement 1 and 2: These are natural lakes. Natural lakes are generally found in mountainous
areas, rift zones, and areas with ongoing glaciation.
Statement 3: This is located in Rajgir, a Buddhist site and a tourist destination.
Statement 4: Hussain Sagar is a heart shaped lake in Hyderabad built by Ibrahim Quli Qutub Shah in 1563,
during the rule of Ibrahim Quli Qutub Shah. It is spread across an area of 5.7 square kilometers and is fed by
River Musi. Artificial Lakes are constructed for industrial or agricultural use, for hydro-electric power
generation or domestic water supply, or for aesthetic or recreational purposes. Guru Gobind Sagar is an
example of an artificial lake. It supports the Bhakra Nangal Hydel Project.

297. The sector that uses over eighty five per cent of groundwater resources in India is
a) Agriculture
b) Household sector
c) Industrial sector
d) Energy sector

Solution: A
Justification: As much as 55% of India’s total water supply comes from groundwater resources. Irrigation
accounts for nearly 90% of total groundwater water used. Its share is much higher in agriculture-heavy states.
Surface water is the major source of water for the industries (41%) followed by groundwater (35%) and
municipal water (24%). The use of municipal water is limited to industries located in urban/ peri-urban areas.
Sectors like pharmaceuticals, power, food processing and agriculture feel the brunt of poor water quality.
High costs for obtaining water are hindering the business interest of smaller industries and the ones which
are located in the drier regions of the country.

298. The area of the country that has the most irrigated proportion among the following is
a) Northern region of Gangetic Plains
b) Western India
c) North Eastern India
d) East India especially West Bengal

Solution: A
Justification: The diagram shows the findings of the ICRISAT data. The Indo-Gangetic plain, and parts of
Gujarat and Madhya Pradesh are well irrigated. But, parts of Karnataka, Maharashtra, Madhya Pradesh,
Rajasthan, Chhattisgarh and Jharkhand are still extremely vulnerable to climate change on account of not
being well irrigated.

Direct PDF - https://telegram.me/UPSC_PrelimsTest


https://telegram.me/UPSC_PDF

OGP 2019 GEOGRAPHY MODULE MCQS

299. Consider the following statements.


Assertion (A): Irrigation intensity reduces consistently as one moves from North to South in India.
Reason (R): Deccan region is known for its hard rock structure as compared to the plains of India.
In the context of the above, which of these is correct?
a) A is correct, and R is an appropriate explanation of A.
b) A is correct, but R is not an appropriate explanation of A.
c) A is incorrect, but R is correct.
d) A is correct, but R is incorrect.

Solution: C
Justification: Hard rock structure does make setting up irrigation structures more difficult, but irrigation
intensity does not reduce in the fashion as explained by statement 1. It is a bit complex and depends on a
number of factors such as availability of river catchments, rock structure, dispersal of agricultural land, state
investment in irrigation etc. The figure below shows the irrigation intensity in India.

Direct PDF - https://telegram.me/UPSC_PrelimsTest


https://telegram.me/UPSC_PDF

OGP 2019 GEOGRAPHY MODULE MCQS

300. Saltwater intrusion is a common problem in


a) Hilly areas
b) Coastal areas
c) Interior of Continents
d) Arid regions

Solution: B
Justification: It is the movement of saline water into freshwater aquifers. The movement happens because
saltwater has a higher mineral content than freshwater, it is denser and has a higher water pressure. As a
result, saltwater can push inland beneath the freshwater, which can lead to contamination of drinking water
sources and other consequences.

301. India’s offshore and deep sea fish catch is very poor considering the marine potential. Why?
1. The Eastern coast, which contributes to majority of the catch, lies in a sedimentary basin zone.
2. UN Commission on the Seas (UNCS) has stated in its recent report that the marine potential of Indian
Ocean is near exhaustion.
Which of the above is/are correct?
a) 1 only
b) 2 only
c) Both 1 and 2
d) None

Solution: D
Justification: Statement 1: Western coasts contribute 75% to the total fish catch, and eastern coast only 25%.
So, 1 is wrong.
Statement 2: There is no such report. Indian Ocean is the least exploited of all oceans. The marine potential is
around 20-25 million tonnes at present, only 11-12% is caught at present. The actual reasons are as here:
 Only 11% of fishing grounds are more than 200 m deep.

Direct PDF - https://telegram.me/UPSC_PrelimsTest


https://telegram.me/UPSC_PDF

OGP 2019 GEOGRAPHY MODULE MCQS

 India has tropical climate in which fish cannot be preserved for a long time. Heavy expenditure on
refrigeration and deep freezing increases the market price of the fish.
 Indian coast does not have many gulfs, bays, estuaries and backwaters as is the case with Norway. As
such, it lacks good fishing grounds
 Marine fishing in India is a seasonal phenomenon. Strong winds during the monsoon season accompanied
by tropical cyclones often hinder the fishing operation.
 Majority of Indians are vegetarians and do not eat fish.
 About 60 per cent of the fishermen still use small non-mechanised boats. They normally do nut venture
beyond 10 km. from the coast and in water more than 18 metres deep. They have to come back to the
coast at night.
 Lack of landing, freezing, canning, transport and organised markets are other handicaps faced by fishing in
India

302. Match the following regions and their traditional water harvesting systems.
1. Virdas: Eastern Coastal Plains
2. Phad: Deccan Plateau
3. Dongs: Brahmaputra Valley
Select the correct answer using the codes below.
a) 1 and 2 only
b) 3 only
c) 1 and 3 only
d) 2 and 3 only

Solution: D
Justification: In respect of availability and non-availability of water, India can be divided into 15 Ecological
regions, ranging from dry, cold desert or Ladakh to the dry hot desert of Rajasthan, from the sub-temperate
mountain of the Himalayas to the tropical high mountain of Nilgiri.

Direct PDF - https://telegram.me/UPSC_PrelimsTest


https://telegram.me/UPSC_PDF

OGP 2019 GEOGRAPHY MODULE MCQS

ZING SYSTEM: Melting glaciers and snows are the only sources of water for the people residing in Ladakh
region. The people of this region make intelligent use of their limited resources and make agriculture possible
in this dry and barren land. The snow and ice melt slowly through the day and water is available in the
streams only in the evening, when it is too late for irrigation. The water in the streams is hence led through
channels to storage tanks and used the next day. These Storage tanks are called as zing.
APTANI SYSTEM: This system is practiced in Arunachal Pradesh by Aptani tribes. Under this system the stream
water is blocked by constructing a wall 2 to 4 m high and 1 m thick near forested hill slopes. This water is
taken to the agricultural fields through channels. The valleys are terraced into plots separated by 0.6 m high
earthen darns with inlet and outlet channels (to the next plot) that help to flood or drain the plots as when
required.
ZABO SYSTEM: This traditional system is practiced in Nagaland. The term labo means 'impounding run-off'.
labo is practiced in Nagaland. When rain falls on terraced hill slopes, the run-off collects in ponds in the
middle terrace. The runoff then passes through slopes where there are cattle yards, and finally reaches the
paddy fields at the foot of the hills. Thus it is through this system that only the irrigation of paddy fields is
brought about, but the fertility of the crop- field is also improved.
BAMBOO DRIP IRRIGATION TRICKLE DRIP IRRIGATION: Under this system the rapidly flowing water front
streams and springs is captured by bamboo pipes and transported over hundreds of metres to drip irrigate
black pepper cultivation in Meghalaya. Many bamboo pipes of varying diameters and lengths are laid to
manipulate and control the flow of water through this system of water conservation.

303. The North-eastern region of the country holds nearly one-third of surface water resources of India and
yet faces water scarcity in many regions because of
1. Hilly terrain that prevents water retention
2. Many springs and streams running dry in non-rainy season
3. Lack of required infrastructure to harness water resources
Select the correct answer using the codes below.
a) 1 only
b) 1 and 3 only
c) 1, 2 and 3
d) 2 and 3 only

Solution: C
Justification: The NE region has an abundance of natural resources.
 It has 34% of total surface water in India.
 It gets high rain falls around 250 cm on an average.
 One of the major rivers of Asia, Brahmputra flows through the region.
However, the region faces a contradictory situation as monsoon months often see devastating floods. But
during non-rainy season many areas face water scarcity. Many areas have hilly terrains where there isn’t

Direct PDF - https://telegram.me/UPSC_PrelimsTest


https://telegram.me/UPSC_PDF

OGP 2019 GEOGRAPHY MODULE MCQS

much water retention. Rain water runoff is quick and many of springs and streams dry off in non-rainy
season. The region needs proper management of its rich water resources and also development of required
infrastructure. At national level, main drinking water source is tap water (44% of total households), followed
by hand pump/tube well water (42% of total households). In North-East, only 21% of households have access
to tap water. Main drinking water source in the region is hand pump/tube well (45% of total households).
But, the absence of required infrastructure is somehow compensated by the proximity of the water
structures from the households (see chart below).

304. In the technique of drip irrigation


a) Water is applied to the crops using a sprinkler
b) Plants are watered by narrow tubings which deliver water directly at the base of the plant
c) Canal water is directly supplied to crops
d) Plants are dipped in high water for long

Solution: B
Justification: Drip irrigation is a form of irrigation that saves water and fertilizer by allowing water to drip
slowly to the roots of many different plants, either onto the soil surface or directly onto the root zone,
through a network of valves, pipes, tubing, and emitters. It is done through narrow tubes that deliver water
directly to the base of the plant. Water is applied close to plants so that only part of the soil in which the
roots grow is wetted unlike surface and sprinkler irrigation, which involves wetting the whole soil profile.
With drip irrigation water, applications are more frequent (usually every 1-3 days) than with other methods
and this provides a very favourable high moisture level in the soil in which plants can flourish. Drip irrigation
is most suitable for row crops (vegetables, soft fruit), tree and vine crops where one or more emitters can be
provided for each plant. Generally only high value crops are considered because of the high capital costs of
installing a drip system.

305. What is the impact of SAUNI Yojna?


1. The project will make the longest conventional open canal system in India which would increase
groundwater recharge in nearby regions.
2. The project requires a very large land area to be acquired which would result in large scale development
displacement.

Which of the above is/are correct?


a) 1 only

Direct PDF - https://telegram.me/UPSC_PrelimsTest


https://telegram.me/UPSC_PDF

OGP 2019 GEOGRAPHY MODULE MCQS

b) 2 only
c) Both 1 and 2
d) None

Solution: D
Justification: Parts of Saurashtra are prone to drought, and irrigation water is also a politically sensitive issue
in this region. SAUNI stands for Saurashtra Narmada Avtaran Irrigation, a project launched in 2012. SAUNI
envisages filling several major dams of the arid Saurashtra region by diverting floodwaters overflowing from
the Sardar Sarovar Dam across the Narmada in south Gujarat. Unlike traditional irrigation projects, SAUNI is
technically a ‘linking’ project, where the water will be filled in irrigation dams that are already equipped with
canal network. This network of canals will then help channel water into farms. Also, SAUNI project involves
making pipe canals instead of the conventional open canals. The pipelines in the SAUNI project are
underground, which means that no land needs to be acquired.

306. In India, which of these regions have the most developed canal system of irrigation?

a) Central India
b) Southern Deccan plateau
c) Northern plains
d) North-eastern India

Solution: C
Justification: This is the rough irrigation map of India.

Direct PDF - https://telegram.me/UPSC_PrelimsTest


https://telegram.me/UPSC_PDF

OGP 2019 GEOGRAPHY MODULE MCQS

You can clearly note that central India lacks good irrigation facilities. Not all regions in India have high levels
of irrigation. Apart from the riverine plains, coastal regions in our country are well-irrigated. The eastern and
northern belt have good canal irrigation facilities. In contrast, plateau regions such as the Deccan plateau
have low levels of irrigation. Of the total cultivated area in the country a little less than 40 per cent is irrigated
even today. In these areas, farming is largely dependent on rainfall.

MINERAL AND ENERGY RESOURCES

307. In India or in areas with Indian jurisdiction or where India’s mineral extraction rights prevail, which of the
following occupy the largest area in geographical terms?
a) Exclusive Economic Zone
b) Continental Shelf
c) Brackish waters and Estuaries
d) Inland Reservoirs

Solution: A
Justification: 1 hectare is equal to 0.01 sq. km. So, in this table, the largest is EEZ followed by continental shelf
and then Reservoirs. The resource base matters for resource extraction like fisheries (figures quoted in table
below).

308. Consider the following about Central Asian resources.


1. Central Asia is a major producer of Uranium, gas and oil.
2. No Central Asian country has Gold reserves.

Direct PDF - https://telegram.me/UPSC_PrelimsTest


https://telegram.me/UPSC_PDF

OGP 2019 GEOGRAPHY MODULE MCQS

3. Most Central Asian countries are major importers of cotton and Aluminium.
Select the correct answer using the codes below.
a) 1 only
b) 1 and 3 only
c) 2 and 3 only
d) 1, 2 and 3

Solution: A
Justification: Statement 1: Kazakhstan and Turkmenistan have abundant oil and natural gas reserves and
Uzbekistan’s own reserves make it more or less self-sufficient.
Statement 2: Kyrgyzstan, Tajikistan and Uzbekistan all have gold reserves and Kazakhstan has the world’s
largest uranium reserves.
Statement 3: Fluctuating global demand for cotton, aluminium and other metals (except gold) in recent years
has hit Tajikistan hardest, since aluminium and raw cotton are its chief exports − the Tajik Aluminium
Company is the country’s primary industrial asset. In fact in 2014, it announced the government’s intention to
reduce the acreage of land cultivated by cotton to make way for other crops. Uzbekistan and Turkmenistan
are major cotton exporters themselves,

309. The Heavy Mineral Sand deposits in Kerala contain an assemblage of which of these minerals
1. Ilmenite
2. Zircon
3. Monazite
4. Rutile
Select the correct answer using the codes below.
a) 1 and 2 only
b) 2, 3 and 4 only
c) 1, 3 and 4 only
d) 1, 2, 3 and 4

Solution: D
Justification: Kerala State is endowed with a number of occurrences/deposits of minerals such as Heavy
Mineral Sands ( Ilmenite, Rutile, Zircon, Monazite, Sillimanite) ,Gold, Iron ore, Bauxite, Graphite, China Clay,
Fire Clay, Tile and Brick Clay, Silica Sand, Lignite, Limestone, Limeshell, Dimension Stone (Granite),
Gemstones, Magnesite, Steatite etc. However, mining activities on large scale are confined mainly to a few
minerals - Heavy Mineral Sands, China Clay and to a lesser extent Limestone/Limeshell, Silica Sand and
Granite. In fact, Heavy mineral sand and China Clay contribute more than 90% of the total value of mineral
production in the State

310. Limestone based topography supports very few minerals, one of the most important of them is
a) Coal
b) Lead
c) Iron ore
d) Copper

Solution: B
Justification: Limestone is a sedimentary rock consisting primarily of calcium carbonate in the form of the
mineral calcite. Aragoniteis also found which is a different crystal form of calcium carbonate (CaCO3). The
only mineral found of importance is Lead. The process of chemical weathering of bedrock in which the
combination of water and acid slowly removes mineral compounds from solid bedrock and carries them away
in liquid solution makes it difficult for limestone to host minerals.

311. What in the continental shelves becomes the source of fossil fuels?
a) Disconnection with the sea

Direct PDF - https://telegram.me/UPSC_PrelimsTest


https://telegram.me/UPSC_PDF

OGP 2019 GEOGRAPHY MODULE MCQS

b) Frequent volcanic eruptions


c) Massive sedimentary deposits received over a long time
d) Tectonic movements

Solution: C
Justification: A continental shelf is the edge of a continent that lies under the ocean. It extends from the
coastline of a continent to a drop-off point called the shelf break. From the break, the shelf descends toward
the deep ocean floor in what is called the continental slope. Even though they are underwater, continental
shelves are part of the continent. The continental shelves are covered with variable thicknesses of sediments
brought down by rivers, glaciers, wind, from the land and distributed by waves and currents. These become
the source of fossil fuels.

312. Which of the following regions have been known traditionally for yielding diamonds in India?
a) Northern and Western India
b) Central and South-eastern India
c) Central and Western India
d) North-eastern and South-western India

Solution: B
Justification: Diamond mining as an industry appears to have originated between 800 and 600 B.C. in India
Kolleru (in former Undivided AP) is thought to have produced many large diamonds that are or have been a
part of crown jewels. The mine was established in the 16th century and operated until the 19th century. As of
2017, there was one industrial-scale diamond mine in India, the Majhgawan mine, near the town of Panna,
Madhya Pradesh. Bunder project in MP and Majhgawan mine were also known for diamond extraction.

313. Coalbed Methane in India can be found in


1. Jharkhand
2. Madhya Pradesh
3. West Bengal
Select the correct answer using the codes below.
a) 1 and 2 only
b) 1 only
c) 2 and 3 only
d) 1, 2 and 3

Solution: D
Justification: India has the fifth-largest coal reserves in the world, estimated at more than 300 billion tonnes
by the Ministry of Coal as in 2014. This means significant potential for CBM exploitation. The exploration of
CBM in India is spread over a sedimentary area of 26,000 square kilometres and is already ongoing, mainly
regions such as Jharkhand, West Bengal and Madhya Pradesh.

314. India primarily imports LNG from which of these countries?


1. USA
2. Mongolia
3. Qatar
4. Australia
Select the correct answer using the codes below.
a) 2 and 3 only
b) 1, 2 and 3 only
c) 1, 3 and 4 only
d) 1 and 4 only

Solution: C

Direct PDF - https://telegram.me/UPSC_PrelimsTest


https://telegram.me/UPSC_PDF

OGP 2019 GEOGRAPHY MODULE MCQS

Justification: Statement 2: Mongolia has no proven natural gas reserves

India primarily imports LNG from Qatar and Australia under long-term contracts. In recent times, it has been
trying to diversify its natural gas sources. Moreover, after former US President Barack Obama lifted US’s 40
year-old oil export ban, Indian oil and gas companies in response bought shipments of US crude to check for
cost competitiveness and its compatibility with Indian refinery configuration. India is also trying to increase its
use of natural gas in the energy mix from current 6% to 15% within five years inorder reduce dependence on
oil and to secure energy supplies and reduce emissions by using more natural gas. GAIL has signed two 20-
year Sale Purchase Agreement (SPA) potentially worth US $32 billion with US’ natural gas exporters.

315. Polymetallic nodules (manganese nodules) are rock concretions on the sea bottom formed of concentric
layers of metal hydroxides around a core. It mainly contains
1. Nickel
2. Selenium
3. Cobalt
4. Bromium
Select the correct answer using the codes below.
a) 1 and 3 only
b) 2, 3 and 4 only
c) 1, 2 and 3 only
d) 1, 2, 3 and 4

Solution: A
Justification: Mainly iron and manganese hydroxide, it also contains nickel and copper. Government of India
is pushing mining of these nodules from sea beds as they are commercially attractive. Nodule mining could
affect tens of thousands of square kilometers of deep sea ecosystems. Nodule regrowth takes decades to
millions of years and that would make such mining an unsustainable and non-renewable practice.

316. Choose the correct arrangement of the following minerals in decreasing order of their estimated value of
production in India:
a) Coal> Natural gas> Iron Ore> Limestone
b) Natural gas > Coal> Iron Ore> Limestone
c) Iron Ore > Limestone > Coal> Natural gas
d) Iron Ore > Coal > Limestone > Natural gas

Solution: A
Justification: These are the figures for the month of August 2017, but annual trends remain more or less
similar (as shown in the figure below). The total estimated value of mineral production (excluding atomic &
minor minerals) in the country during August 2017 was Rs. 18015 crore. The contribution of Coal was the

Direct PDF - https://telegram.me/UPSC_PrelimsTest


https://telegram.me/UPSC_PDF

OGP 2019 GEOGRAPHY MODULE MCQS

highest at Rs. 6158 crore (34%). Next in the order of importance were: Petroleum (crude) Rs. 5489 crore,
Natural gas (utilized) Rs. 2225 crore, Iron ore Rs. 1921 crore, Lignite Rs. 615 crore and Limestone Rs. 562
crore. These six minerals together contributed about 94% of the total value of mineral production in August
2017.

317. Which of these minerals is famously found at North Cachar Hills?


a) Uranium
b) Tertiary Coal
c) Graphite
d) Mica

Solution: B
Justification: North Cachar Hills is one of the autonomous district councils in Assam under the Sixth Schedule
of the constitution. It was recently in news due to a large scam. Tertiary coal is young and inferior coal
deposit. Assam coal is friable in nature and has a high sulphur content. It is mainly utilised by local railways,
steamers, and hydro power stations.

318. Gold and diamonds are most likely to be found in which of these regions of Africa?
a) West Africa
b) Southern Africa
c) East Africa
d) North-western Africa

Solution: B
Justification:

Direct PDF - https://telegram.me/UPSC_PrelimsTest


https://telegram.me/UPSC_PDF

OGP 2019 GEOGRAPHY MODULE MCQS

319. Most of the salt produced in India comes from


a) Rock-salt deposits
b) Sea Water
c) Brine springs
d) Salt lakes

Solution: B
Justification: There are three main sources of salt in India: (i) sea water, along the coasts of the Peninsula, (ii)
brine springs, wells and salt lakes of the arid tracts of Rajasthan, and (iii) rock-salt deposits of Gujarat and
Himachal Pradesh. About 75 per cent of the total salt produced in India is manufactured in marine salt works
by the solar evaporation of sea water. Dry and sunny climate is essential for salt production and the western
arid region of Gujarat along with 1,600 km long coastline provides a conducive environment, making Gujarat

Direct PDF - https://telegram.me/UPSC_PrelimsTest


https://telegram.me/UPSC_PDF

OGP 2019 GEOGRAPHY MODULE MCQS

the largest producer of salt in the country. The state contributes nearly 70% of the total salt produced in the
country. Unseasonal rainfall and an extended winter could cut down the production of salt.

320. As per the estimates of Ministry of Coal, which of these states has the largest geological coal reserve?
a) Assam
b) Odisha
c) Maharashtra
d) West Bengal

Solution: B
Justification: The Coal resources of India are available in older Gondwana Formations of peninsular India and
younger Tertiary formations of north-eastern region. As a result of exploration carried out up to the
maximum depth of 1200m by the GSI, CMPDI, SCCL and MECL etc, a cumulative total of 301.56 Billion tonnes
of Geological Resources of Coal have so far been estimated in the country as on 1.4.2014. The details of state-
wise geological resources of Coal are given as under:

Meghalaya and Nagaland have some coal reserves as well.

321. Most of the lignite reserves in India occur in


a) Karnataka
b) Jharkhand
c) Odisha
d) Tamil Nadu

Solution: D
Justification: Indian coal offers a unique eco-friendly fuel source to domestic energy market for the next
century and beyond. Hard coal deposit spread over 27 major coalfields, are mainly confined to eastern and
south central parts of the country. The lignite reserves stand at a level around 36 billion tonnes, of which 90
% occur in the southern State of Tamil Nadu. Coal accounts for 55% of the country's energy need. Commercial
primary energy consumption in India has grown by about 700% in the last four decades. Coal accounts for a
large part of this increase.

Direct PDF - https://telegram.me/UPSC_PrelimsTest


https://telegram.me/UPSC_PDF

OGP 2019 GEOGRAPHY MODULE MCQS

322. Despite having large reserves of coal, why India imports millions of tonnes of coal every year?
1. Availability of coking coal is limited in India.
2. Indian coal has very high calorific value that is unsuitable for the power plants in India.
3. Fumes emitted by Indian coal contain Mercury.

Select the correct answer using the codes below.


a) 1 only
b) 2 and 3 only
c) 1 and 2 only
d) 3 only

Solution: A
Justification: Statement 1: Indian coal quality is poor and the supplies are less given so many problems like
mining licenses issue, problems of fuel supply agreements; technology deficit for sustainable coal mining etc.
Coking coal needs to be of higher quality and hence needs to be imported.
Statement 2: This is incorrect, there are better coal grades available abroad that yield high calorific value.
Higher the calorific value, better it is for the power plants.
Statement 3: All Coal emissions invariably contain mercury, so this can’t be a reason for importing coal from
abroad.

323. Which of these are examples of trans-national resources over which more than one nation can have joint
control?
1. Marine animals
2. Manganese nodules
3. Crude Oil and natural Gas

Select the correct answer using the codes below.


a) 1 and 2 only
b) 2 only
c) 3 only
d) 1, 2 and 3

Solution: D
Justification: There are international institutions which regulate some resources. The oceanic resources
beyond 200 km of the Exclusive Economic Zone belong to open ocean and no individual country can utilise
these without the concurrence of international institutions. Anything useful found in the sea or seabed can be
termed as a resource like gas, oil etc.

324. Consider the following minerals and regions associated with their deposits.
1. Bauxite: Madhya Pradesh
2. Mica: Andhra Pradesh
3. Copper: Jharkhand
4. Manganese: Chhattisgarh

Select the correct matches using the codes below.


a) 1 and 2 only
b) 3 and 4 only
c) 1, 2 and 3 only
d) D. 1, 2, 3 and 4

Solution: D
Justification: Statement 1: Major bauxite producing areas are Jharkhand, Orissa, Chhattisgarh, Madhya
Pradesh, Gujarat, Maharashtra and Tamil Nadu.

Direct PDF - https://telegram.me/UPSC_PrelimsTest


https://telegram.me/UPSC_PDF

OGP 2019 GEOGRAPHY MODULE MCQS

Statement 2: Mica deposits mainly occur in Jharkhand, Bihar, Andhra Pradesh and Rajasthan. India is the
largest producer and exporter of mica in the world.
Statement 3: Copper is mainly produced in Rajasthan, Madhya Pradesh, Jharkhand, Karnataka and Andhra
Pradesh.
Statement 4: India’s manganese deposits lie in Maharashtra, Madhya Pradesh, Chhattisgarh, Orissa,
Karnataka and Andhra Pradesh.

325. Among the following, which ones are ferrous minerals?


1. Mica
2. Chromite
3. Gypsum
4. Lead

Select the correct answer using the codes below.


a) 1, 2 and 3 only
b) 4 only
c) 2 only
d) 1 and 4 only

Solution: C
Justification: Metallic minerals may be ferrous or non-ferrous. Ferrous minerals like iron ore, manganese and
chromite contain iron. A non-ferrous mineral does not contain iron but may contain some other metal such as
gold, silver, copper or lead. The mineral fuels like coal and petroleum are also non-metallic minerals. The
non-metallic minerals do not contain metals. Limestone, mica and gypsum are examples of such minerals.

326. India has deposits of high grade iron ore which are mainly found in
1. Jharkhand
2. Orissa
3. Chhattisgarh
4. Madhya Pradesh
5. Goa

Select the correct answer using the codes below.


a) 1, 2 and 3 only
b) 4 and 5 only
c) 2, 3 and 4 only
d) 1, 2, 3, 4 and 5

Solution: D
Justification: It is also found in Maharashtra and Karnataka. India is ranked 4th in the production of iron ore.
Mining iron ore is a high volume low margin business, as the value of iron is significantly lower than base
metals. It is highly capital intensive, and requires significant investment in infrastructure such as rail in order
to transport the ore from the mine to a freight ship.

327. Why Gold mining in Kolar mines is expensive or economically less viable?
1. Gold is combined with uranium in these mines.
2. These are some of the deepest mines on earth.
3. Mining follows expulsion of methane hydrates from the mine bed.

Select the correct answer using the codes below.


a) 1 only
b) 2 only
c) 2 and 3 only

Direct PDF - https://telegram.me/UPSC_PrelimsTest


https://telegram.me/UPSC_PDF

OGP 2019 GEOGRAPHY MODULE MCQS

d) None of the above

Solution: B
Justification: Statement 2: Even though these mines produce Gold, the extraction potential cannot be
reached due to the depth of the mine. India currently produces hardly 0.4% of its gold consumption despite
having 9% of global gold reserves under the country’s land mass.
Statement 3: Methane Hydrates are found mostly under ocean beds. So, 3 is wrong.
The Indian government has asked GSI to explore additional reserves of gold and diamonds in Andhra Pradesh,
Karnataka, Madhya Pradesh, West Bengal, Rajasthan, Bihar and Chhattisgarh. The government is also
formulating a new mining policy to boost exploration of gold and diamond deposits. The new mining policy is
expected to open up the mining sector for foreign investment.

MANUFACTURING INDUSTRIES

PLANNING AND SUSTAINABLE DEVELOPMENT IN INDIA

328. India as a “Knowledge Economy” implies


a) Only education can result in our economic growth.
b) India has become a large export destination for published materials.
c) Physical infrastructure must be given for expanding knowledge-based education in Universities.
d) India is advancing in the use of knowledge-intensive sectors like Information Technology.

Solution: D
Justification: A knowledge economy relies more on consumption and production on intellectual capital rather
than only physical goods and services. For e.g. the Indian software industry has been showing an impressive
record over the past decade. Entrepreneurs, bureaucrats and politicians are now advancing views about how
India can transform itself into a knowledge-based economy by using information technology (IT). There have
been some instances of villagers using e-mail which are cited as examples of such transformation. Likewise, e-
governance is being projected as the way of the future. The value of IT depends greatly on the existing level
of economic development.

TRANSPORTATION AND COMMUNICATION


ROADS

329. Why Highways in Russia are not as important as railways?


a) Anomalous Magnetic activity that supports grounding of metallic rails rather than roads
b) Multitude of subsidence zones and weak soil
c) Criss-crossing of rivers across the major transportation zones
d) Vast geographical expanse

Solution: D
Justification: In Russia, a dense highway network is developed in the industrialised region west of the Urals
with Moscow as the hub. The important Moscow-Vladivostok Highway serves the region to the east. Due to
the vast geographical area, highways in Russia are not as important as railways.

330. The construction of Silkyara Bend-Barkot Tunnel has often been in news. Completion of work on this
tunnel will provide
a) All weather connectivity to all chaar-dhaams
b) Leh-Zanskar access in Winter season

Direct PDF - https://telegram.me/UPSC_PrelimsTest


https://telegram.me/UPSC_PDF

OGP 2019 GEOGRAPHY MODULE MCQS

c) Indian troops access to Xinjiang region that is dominated by China


d) None of the above

Solution: D
Justification: Cabinet Committee on Economic Affairs has approved Silkyara Bend-Barkot Tunnel in
Uttarkhand as part of ‘Chardham Mahamarg Pariyojana’. The construction of this tunnel will provide all
weather connectivity to Yamunotri (and not all chardhams), one of the dham on Chardham Yatra,
encouraging regional socio-economic development, trade and tourism within the country. It will reduce the
travel distance from Dharasu to Yamunotri by about 20 km and travel time by about an hour. The project will
be falling along NH-134 (old NH-94) in the State of Uttarakhand. The Chardham Mahamarg Vikas Pariyojna, or
the Chardham highway development project, is an ambitious initiative to improve connectivity to the Char
Dham pilgrimage centres in the Himalayas.

331. As shown by the Economic Survey 2017-18, the density of National highways in India is likely to be higher
in the
a) Relatively developed States
b) Coastal states
c) States with International Border
d) Central Indian region due to highway convergence

Solution: A
Justification: The relatively developed States like Maharashtra, Karnataka, Kerala and Goa have higher
density of National Highways and State Highways followed by Gujarat, Tamil Nadu, Bihar, Haryana and a
number of hilly States (MAP 1)

Direct PDF - https://telegram.me/UPSC_PrelimsTest


https://telegram.me/UPSC_PDF

OGP 2019 GEOGRAPHY MODULE MCQS

WATERWAYS

332. Consider the following statements.


If the National Waterway – 1 is fully developed
Assertion (A): The cargo from the Gangetic plain states that takes a larger circuitous land route to reach the
Maharashtra ports can go to the much-closer Kolkata port.
Reason (R): The NW 1 will link up with the Eastern Dedicated Rail Freight Corridor, as well as with the area’s
existing network of highways.
In the context of the above, which of these is correct?
a) A is correct, and R is an appropriate explanation of A.
b) A is correct, but R is not an appropriate explanation of A.
c) A is correct, but R is incorrect.
d) Both A and R are incorrect.

Solution: A
Justification: The waterway’s stretch between Kolkata and Delhi passes through one of India’s most densely
populated areas. A sizeable forty percent of all India’s traded goods either originate from this resource-rich
region or are destined for its teeming markets. While the region is estimated to generate about 370 million
tonnes of freight annually, only a tiny fraction of this - about 5 million tonnes - currently travels by water.

Direct PDF - https://telegram.me/UPSC_PrelimsTest


https://telegram.me/UPSC_PDF

OGP 2019 GEOGRAPHY MODULE MCQS

Currently, cargo from the Gangetic states of Bihar and Uttar Pradesh takes circuitous land routes to reach the
sea ports of Mumbai in Maharashtra and Kandla in Gujarat, rather than going to the much-closer port at
Kolkata. The development of NW1 will help these states direct some of their freight to the Kolkata-Haldia
complex, making the movement of freight more reliable and reducing logistics costs significantly. The World
Bank is financing the development of the Ganga waterway with a loan of $ 375 million. Once operational, the
waterway will form part of the larger multi-modal transport network being planned along the river.

333. Why large vessels do not navigate in the Indian Ocean through Palk Strait?
a) It is a religious site.
b) It is too shallow for ships to cross.
c) It has disputed islands.
d) Sri Lanka government does not allow International vessels in the Strait.

Solution: B
Justification: A ship going from Chennai to Kochi goes around Sri Lanka rather than crossing through the Palk
Strait, that divides India and Sri Lanka. The shallow waters and reefs of the strait make it difficult for large
ships to pass through, although fishing boats and small craft carrying coastal trade have navigated the strait
for centuries. Large ships must travel around Sri Lanka. Construction of a shipping canal through the strait
was first proposed to the British government of India in 1860, and a number of commissions have studied the
proposal up to the present day. The most recent study of the Sethusamudram Shipping Canal Project, as it is
now called, was an environmental impact assessment and a technical feasibility study commissioned by the
Tamil Nadu government in 2004.

334. National Waterway-4, which has been seen frequently in news, covers canals and waterways built over
1. Godavari river
2. Krishna river
3. Mahanadi river
4. Ghaghra river

Select the correct answer using the codes below.


a) 3 and 4 only
b) 1 and 2 only
c) 1, 2 and 3 only
d) D. 1, 2, 3 and 4

Solution: B
Justification: Statement 1: Kakinada-Puducherry canal stretch along with Godavari River stretch between
(Bhadrachalam and Rajhamundry) and Krishna River stretch between (Wazirabad and Vijaywada) is termed as
NW-4. Total length of NW-4 is 1095 km.
A network of irrigation cum navigation canal linking Chennai and Ennore Ports in Tamil Nadu with the
Kakinada Port, and Machlipatnam Ports in Andhra Pradesh runs through a long distance.

Direct PDF - https://telegram.me/UPSC_PrelimsTest


https://telegram.me/UPSC_PDF

OGP 2019 GEOGRAPHY MODULE MCQS

335. From the point of view of commerce, the Atlantic Ocean is the busiest Ocean in the world. This is because
1. It is the largest ocean in the world.
2. It connects major continents.
3. Its coastline is indented that favours formation of natural harbours and ports.

Select the correct answer using the codes below.


a) 1 and 2 only
b) 3 only
c) 2 and 3 only
d) 1, 2 and 3 only
Solution: C
Justification: Statement 1: Pacific is the largest, Atlantic is second largest.
Statement 2: It connects the South America to North America (Panama Canal), Africa to Europe and Europe to
Asia (through Gibraltar Strait connecting Mediterranean Sea).
Statement 3: The coastline of Atlantic Ocean is highly indented. This irregular and indented coastline provides
ideal location for natural harbours and ports that favours commerce related activities.

Direct PDF - https://telegram.me/UPSC_PrelimsTest


https://telegram.me/UPSC_PDF

OGP 2019 GEOGRAPHY MODULE MCQS

INTERNATIONAL TRADE

LOCATION BASED QUESTIONS (MAPPING)


INDIA BASED

336. Which of the following regions of India is known as the Tiger Capital of the country?
a) Gir
b) Nagpur
c) Bandipur
d) Nagarhole

Solution: B
Justification: The central government is promoting Nagpur in Maharashtra as the tiger capital. As per an
official statement, Maharashtra has done exceptionally well in the growth of tigers- which has prompted the
government to promote Nagpur, which is the central most city of the country. Maharashtra has a tiger
population of 169 according to latest census. Nagpur and its nearby reserves in Maharashtra have 148 of
these. In addition, the city was close to many tiger reserves in Madhya Pradesh like Kanha, Pench, and
Satpuda. There are many notified tiger parks and reserves in its close proximity. It is promoted as the tiger
capital of the world given the city's closeness with several tiger reserves and its infrastructure that can
support a large tourist industry

337. Why are estuaries considered favourable for situating ports?


1. Estuaries have an open connection to the Sea or Ocean.
2. Estuaries have considerable depth of water for anchoring and stationing ships.
Which of the above is/are correct?
a) 1 only
b) 2 only
c) Both 1 and 2
d) None

Solution: C
Justification: An estuary is a partially enclosed coastal body of brackish water with one or more rivers or
streams flowing into it, and with a free connection to the open sea. They are an important part of the
shipping industry because there are many industrial ports located in estuaries due its depth of water and
connection with open sea. As against estuaries, deltas are not preferred for coastal shipping because deltas
do not offer the depth of water required for shipping and may not offer the required connection to the open
sea.

338. Consider the following statements.


1. It is India's largest inland salt lake.
2. It is geographically a land locked river basin.
3. It has been designated as a Ramsar site.

The above refer to?


a) Bhimtal Lake
b) Bhitarkanika Lake
c) Sambhar Lake
d) Shakambari Lake

Solution: C

Direct PDF - https://telegram.me/UPSC_PrelimsTest


https://telegram.me/UPSC_PDF

OGP 2019 GEOGRAPHY MODULE MCQS

Justification: Sambhar has been designated as a Ramsar site (recognized wetland of international
importance) because the wetland is a key wintering area for tens of thousands of flamingos and other birds
that migrate from northern Asia. The specialized algae and bacteria growing in the lake provide striking water
colours and support the lake ecology that, in turn, sustains the migrating waterfowl. It is not part of the
Ganga river basin area and is geographically a separate land locked river basin. It is the source of most of
Rajasthan's salt production.

339. Consider the following matches of Buddhist Monasteries with their locations.
1. Tabo Monastery : Located in a Valley of Northern India
2. Kardang Monastery : Most populated monastery in Southern India
3. Rumtek Monastery : Largest in Himachal Pradesh

Select the correct answer using the codes below.


a) 1 and 2 only
b) 3 only
c) 1 only
d) 2 only

Solution: C
Justification: Statement 1: It is located in the Tabo village of Spiti Valley, Himachal Pradesh. It was founded in
996 CE in the Tibetan year of the Fire Ape. Tabo is noted for being the oldest continuously operating Buddhist
enclave in both India and the Himalayas. A large number of frescoes displayed on its walls depict tales from
the Buddhist pantheon.
Statement 2: Kardang Monastery or Gompa is a famous Drukpa Lineage monastery, and is the most
important monastery the Lahaul valley, India. The monastery is a huge white building bedecked with prayer
flags.
Statement 3: The monastery is currently the largest in Sikkim. It is home to the community of monks and
where they perform the rituals and practices of the Karma Kagyu lineage. A golden stupa contains the relics
of the 16th Karmapa. Opposite that building is a college, Karma Shri Nalanda Institute for Higher Buddhist
Studies.

340. The Union Cabinet has approved the construction of Asia's longest bi-directional Zojila Pass tunnel. This
will facilitate all-weather connectivity between
a) Srinagar and Leh
b) Zanskar and Nathu La
c) Ladakh and Xinjiang
d) Siachin Glacier and Far eastern side of the Indo-China border

Solution: A
Justification: The tunnel is being built at an estimated cost of Rs 6,089 crore (please ignore the figure
mentioned in the image below), which will reduce the travel time between Srinagar and Leh to 15 minutes
from the current 3.5 hours. Zojila pass is situated at an altitude of 11,578 feet on Srinagar-Kargil-Leh National
Highway which remains closed during winters (December to April) due to heavy snowfall and avalanches
cutting off Leh-Ladakh region from Kashmir.

Direct PDF - https://telegram.me/UPSC_PrelimsTest


https://telegram.me/UPSC_PDF

OGP 2019 GEOGRAPHY MODULE MCQS

341. The country that has the longest coastline on the Indian Ocean is
a) Indonesia
b) Myanmar
c) India
d) Malaysia

Solution: C
Justification: The Indian landmass has a central location between the East and the West Asia. India is a
southward extension of the Asian Continent. No other country has a long coastline on the Indian Ocean as
India has and indeed, it is India’s eminent position in the Indian Ocean which justifies the naming of an Ocean
after it.

342. With reference to the Suez Canal, consider the following statements.
1. It is a naturally made sea-level waterway in Egypt.
2. Transit through the Suez Canal is allowed only to vessels of member nations of the Suez Canal Authority
(SCA).
3. During war times, every vessel of commerce or of war can use the canal without distinction.
Select the correct answer using the codes below.
a) 1 and 2 only
b) 2 and 3 only
c) 3 only
d) 1 only

Solution: C
Justification: Statement 1: The Suez Canal is an artificial waterway in Egypt, connecting the Mediterranean
Sea to the Red Sea through the Isthmus of Suez. It is a sea level Canal.
Statement 2: Transit through the Suez Canal is allowed to all vessels of the world subject to comply with the
conditions stated in the present Rules of Navigation.

Direct PDF - https://telegram.me/UPSC_PrelimsTest


https://telegram.me/UPSC_PDF

OGP 2019 GEOGRAPHY MODULE MCQS

Statement 3: The canal is owned and maintained by the Suez Canal Authority (SCA) of Egypt. Under the
Convention of Constantinople, it may be used "in time of war as in time of peace, by every vessel of
commerce or of war, without distinction of flag". Since the opening of the Suez Canal in 1869, India’s distance
from Europe has been reduced by 7,000 km.

343. A National Park with Shola forests can be most likely found in which of the following parts of India?
a) Eastern Ghats
b) Wetlands near Gulf of Mannar
c) Lakshadweep Islands
d) Western Ghats

Solution: D
Justification: Sholas are the local name for patches of stunted tropical montane forest found in valleys amid
rolling grassland in the higher montane regions of South India. These patches of shola forest are found mainly
in the valleys and are usually separated from one another by undulating montane grassland. The shola and
grassland together form the shola-grassland complex or mosaic. Shola forests are found in the higher altitude
hill regions of the Nilgiris, Kanyakumari district, the Western Ghats and associated ranges in the states of
Karnataka, Kerala and Tamil Nadu. Although generally said to occur above 2000 meters above sea level, shola
forests can be found at 1600 meters elevation in many hill ranges (e.g. Biligiriranga Hills)

344. This Protected Area is very important because it links the eastern and western populations of many wild
animals through the Vindhyan ranges that run from north-east to south-west. It can be
a) Panna Tiger Reserve
b) Jim Corbett National Park
c) Gir Forest National Park
d) Sariska Wildlife Sanctuary

Solution: A
Justification: Situated in the Vindhyan mountain range in the northern part of Madhya Pradesh, Panna Tiger
Reserve is spread over the Panna and Chhatarpur districts. The terrain here consists of extensive plateaus and
gorges. This reserve contains the last remaining tiger habitat of North Madhya Pradesh. Apart from the tiger,
it is home to other animals like the leopard, nilgai, chinkara, chousinga, chital, rusty spotted cat, porcupine,
and sambhar. Gharials (long snouted crocodiles) and muggars (marsh crocodiles) can be found in River Ken.
Poaching is of significant concern as the park lost almost all its tigers in 2009 due to the menace. The reserve
is also dotted with two thousand year-old rock paintings.

345. The state of Sikkim borders which of the following regions?


1. Nepal
2. Bihar
3. Bhutan
4. Assam
Select the correct answer using the codes below.
a) 1, 2 and 3 only
b) 1 and 3 only
c) 1, 3 and 4 only
d) 2 and 4 only

Solution: B
Justification:

Direct PDF - https://telegram.me/UPSC_PrelimsTest


https://telegram.me/UPSC_PDF

OGP 2019 GEOGRAPHY MODULE MCQS

346. The Indo-Bangladesh Border covers which of the states of India?


1. Assam
2. Nagaland
3. Mizoram
4. Tripura
5. Arunachal Pradesh

Select the correct answer using the codes below.


a) 1, 2, 3 and 4 only
b) 1, 3 and 4 only
c) 3, 4 and 5 only
d) 1 and 5 only

Solution: B
Justification: 5 states of India including Assam, Meghalaya, Mizoram, Tripura and West Bengal cover the
border which is 4096 km long.

347. These are some of the notable lakes in India. Match them with the state they are located in.
1. Pulicat Lake: Kerala
2. Haflong Lake: Arunachal Pradesh
3. Sardar Sarovar Dam: Haryana
4. Chandra Tal Lake: Himachal Pradesh

Select the correct answer using the codes below.

Direct PDF - https://telegram.me/UPSC_PrelimsTest


https://telegram.me/UPSC_PDF

OGP 2019 GEOGRAPHY MODULE MCQS

a) 1 and 3 only
b) 2 and 4 only
c) 4 only
d) 1, 3 and 4 only

Solution: C
Justification: Statement 1: The second largest brackish water lake or lagoon in India, after Chilika Lake, it
straddles the border of Andhra Pradesh and Tamil Nadu states with over 96% of it in Andhra Pradesh and 4%
in Tamil Nadu situated on the Coromandal Coast in South India. The lake encompasses the Pulicat Lake Bird
Sanctuary.
Statement 2: Located in Assam, it is a major tourist attraction and is under protection of Dima Hasao
Tourist/forest Department and Dima Hasao Autonomous Council.
Statement 3: It is a gravity dam on the Narmada river in Gujarat. Four Indian states, Gujarat, Madhya Pradesh,
Maharastra and Rajasthan, receive water and electricity supplied from the dam.
Statement 4: Situated in the Spiti part of the Lahul and Spiti district of Himachal Pradesh (India), the name of
the lake originates from its crescent shape. It is situated at an altitude of about 14,100 ft in the Himalayas.

348. The island territories of the Andaman and Nicobar Islands provide India with a strategic presence at the
entrance to the
a) Isthmus of Oman
b) Strait of Malacca
c) Sea lines of Persia
d) Straits of Irrawaddy

Solution: B
Justification: India’s location at the base of continental Asia and at the top of the Indian Ocean provides it a
vantage point with respect to both, Central Asia and the Indian Ocean Region. The island territories of the
Andaman and Nicobar Islands, located 1,300 km away from the mainland in the Bay of Bengal, provide India
with a strategic presence at the entrance to the Strait of Malacca, through which more than 60,000 shipping
vessels transit each year

Direct PDF - https://telegram.me/UPSC_PrelimsTest


https://telegram.me/UPSC_PDF

OGP 2019 GEOGRAPHY MODULE MCQS

349. Enclosed by the Aravali Hills in the North, it is an important artificial lake and a large bird site with the
famous Jal Mahal being situated in the middle of the lake. It is
a) Man Sagar Lake
b) Bhojtal Lake
c) Pichola Lake
d) Kanwar Lake

Solution: A
Justification: It has a water spread area of 300 acres and is enclosed by the Aravalli hills on the north, west
and eastern sides, while the southern side consists of plains that are intensely inhabited. Throughout the
year, more than 180 species of birds have been recorded at the Birding Fair site, making it as a great
attraction to bird watchers. As the lake is being restored and islands have been created for bird nesting, more
species in bigger number are expected to settle here. The 21st Birding festival is being held at the lake.

350. What is the importance of Sela Pass, which was recently in news?
1. It is carries the main road connecting Chicken’s neck Corridor of North-east with the rest of India.
2. It is considered to be the most sacred site of Mahayana Buddhists.

Which of the above is/are correct?


a) 1 only
b) 2 only

Direct PDF - https://telegram.me/UPSC_PrelimsTest


https://telegram.me/UPSC_PDF

OGP 2019 GEOGRAPHY MODULE MCQS

c) Both 1 and 2
d) None

Solution: D
Justification: The Sela Pass is a high-altitude mountain pass located on the border between the Tawang and
West Kameng Districts of Arunachal Pradesh. It connects the Tibetan Buddhist town of Tawang to Dirang and
Guwahati. The pass carries the main road connecting Tawang with the rest of India. While Sela Pass does get
heavy snowfall in winters, it is usually open throughout the year unless landslides or snow require the pass to
be shut down temporarily. The Finance Minister announced the government’s plan to build a tunnel through
the Sela Pass located at an elevation of 13,700 ft which will ensure faster movement of troops in Tawang, a
strategically- located town in Arunachal Pradesh bordering China. Sela Lake, near the summit of the pass, is
one of approximately 101 lakes in the area that are sacred in Tibetan Buddhism.

351. The states with the shortest and longest coastline are
a) Odisha and Maharashtra
b) Goa and Tamilnadu
c) Goa and Gujarat
d) Odisha and Andhra Pradesh

Solution: C
Justification: Total length of Indian coastline is 7517 Km. Out of this total length the coastal length of
mainland India is 5423 Km and the coastal length of all Islands (Andaman & Nicobar, Kavaratti islands) is 2094
Km. Gujarat’s coastline is 1600 Km, followed by Tamilnadu, AP and Maharashtra. Goa has the shortest
coastline among all states.

352. Northern Circars and Coromandel Coast in India are located


a) Along the Eastern Ghats
b) Along the Western and Eastern Ghats respectively
c) Along the Eastern and Western Ghats respectively
d) Along the Western Ghats

Solution: A
Justification: The plain along the Eastern coast of India is called the Eastern Coastal Plain. This plain stretches
from the Delta of the Ganga to Kanya Kumari. This plain includes some parts of Orissa, Andhra Pradesh and
Tamil Nadu. This plain is much wider than the Western Coastal Plain. Its width varies from 100 to 150 km. This
plain can also be divided into two parts. The Northern part that lies in the North of river Krishna is called the
Northern Circars. The Southern part is called The Coromandal Coast. The Northern Circars plain consists of
the deltas of Mahanadi, Godavari and Krishna. These rivers have broken the Eastern Ghats at many places
and made this plain more wide.

Direct PDF - https://telegram.me/UPSC_PrelimsTest


https://telegram.me/UPSC_PDF

OGP 2019 GEOGRAPHY MODULE MCQS

353. Consider the following hills and their locations.


1. Shevroy Hills: Eastern Ghats
2. Cardamom Hills: Western Ghats
3. Anaimalai Hills: Garhjat Range

Select the correct answer using the codes below.


a) 1 and 2 only
b) 2 only
c) 1 and 3 only
d) 1, 2 and 3

Solution: A
Justification: Statement 1: The Servarayans form part of the southern ranges of the Eastern Ghats System. It
also represents the highest peak in southern part of the Eastern Ghats, with the Solaikaradu peak.
Statement 2: They conjoin the Anaimalai Hills to the northwest, the Palni Hills to the northeast and the
Agasthyamalai Hills to the south as far as the Aryankavu. The crest of the hills forms the boundary between
Kerala and Tamil Nadu. Anamudi in Eravikulam National Park, is the highest peak in Western Ghats and also
the highest point in India south of the Himalayas
Statement 3: They form a southern portion of the Western Ghats. Anamala / Anaimalai Hills are south of
where the Western Ghats are broken by the Palakkad Gap, which in turn is south of the Nilgiri Hills. They
border the state of Kerala on the Southwest and the Cardamom Hills to the southeast.

354. Deterioration of relations between which of these countries will have repercussions on the Walong tri-
junction?
a) India, Myanmar and Bangladesh

Direct PDF - https://telegram.me/UPSC_PrelimsTest


https://telegram.me/UPSC_PDF

OGP 2019 GEOGRAPHY MODULE MCQS

b) India, China and Pakistan


c) India, Pakistan and Afghanistan
d) India, China and Myanmar

Solution: D
Justification: The Walong tri-junction is located around 50 km from Walong, India’s easternmost town in
Arunachal Pradesh, near the Tibet region. It is situated on the bank of Lohit river. Indian troops have
increased their patrolling at a tri-junction of India, China and Myanmar to prevent a repeat of a Doklam-like
standoff. As per an official, Chinese troops did not enter the tri-junction too frequently, but had developed
road infrastructure near the area, which could be advantageous for the mobilisation of troops.

355. Part of the Eastern Ghats, these hills were declared as a Biosphere Reserve in 2010. Tirupati, a major
Hindu pilgrimage town, is located in these hills. It is
a) Anantagiri Hills
b) Seshachalam Hills
c) Agasthyamalai Biosphere Reserve
d) Simlipal Biosphere Reserve

Solution: B
Justification: The ranges were formed during the Precambrian era (3.8 billion to 540 million years ago).
Minerals contained in these hills include sandstone and shale interbedded with limestone. The ranges are
bounded by the Rayalaseema uplands to the west and northwest, and the Nandyal Valley to the north. The
Srivenkateshwara National Park is also located in these ranges. The famous Natural Arch, Tirumala Hills is also
a part of Seshachalam Hills. In 2010 it was designated as a Biosphere Reserve. It has large reserves of Red
Sandal wood which is used in medicines, soaps, spiritual rituals.

356. This river is a major tributary of Brahmaputra river and splits into two separate rivers, the Beki and
Bholkaduba. The river serves as an international border dividing India and Bhutan. The protected area named
after this river is
a) Nameri National Park
b) Kolong Wildlife Sanctuary
c) Singalila National Park
d) Manas National Park

Solution: D
Justification: Manas National Park is a UNESCO Natural World Heritage site, a Project Tiger reserve, an
elephant reserve and a biosphere reserve in Assam, India. Located in the Himalayan foothills, it is contiguous
with the Royal Manas National Park in Bhutan. The Manas river flows thorough the west of the park and is
the main river within it. It is a major tributary of Brahmaputra River and splits into two separate rivers, the
Beki and Bholkaduba as it reaches the plains. The Manas River also serves as an international border dividing
India and Bhutan. The bedrock of the savannah area in the north of the park is made up of limestone and
sandstone, whereas the grasslands in the south of the park stand on deep deposits of fine alluvium. The
combination of Sub-Himalayan Bhabar Terai formation along with the riverine succession continuing up to
Sub-Himalayan mountain forest make it one of the richest areas of biodiversity in the world. The park is well
known for species of rare and endangered wildlife that are not found anywhere else in the world like the
Assam roofed turtle, hispid hare, golden langur and pygmy hog. Assam Spring Festival is being held at the
Manas National Park where visitors will get to experience the state’s local food, music, handloom and
handicrafts.

357. Madhya Pradesh shares land boundary with which of these states?
1. Rajasthan
2. Andhra Pradesh
3. Karnataka

Direct PDF - https://telegram.me/UPSC_PrelimsTest


https://telegram.me/UPSC_PDF

OGP 2019 GEOGRAPHY MODULE MCQS

4. Jharkhand
5. Bihar

Select the correct answer using the codes below.


a) 1, 4 and 5 only
b) 1 only
c) 1, 2 and 4 only
d) 2, 3 and 5 only

Solution: B
Justification: It borders Chhattisgarh, MH, GJ, RJ and UP only.

358. Which of these regions fall between Gulf of Khambat and Gulf of Kutch?
1. Kuchchh
2. Dadra and Nagar Haveli
3. Porbandar
4. Diu Daman

Select the correct answer using the codes below.


a) 1 and 4 only
b) 2 and 3 only
c) 3 only

Direct PDF - https://telegram.me/UPSC_PrelimsTest


https://telegram.me/UPSC_PDF

OGP 2019 GEOGRAPHY MODULE MCQS

d) None of the above

Solution: C
Justification: If you eliminate the position of these two Union Territories from the options, you can easily
arrive at the answer.

359. Willingdon Islands is the largest artificial island in India, which forms part of the city of
a) Ahmedabad
b) Kochi
c) Vishakhapatnam
d) Bengaluru

Solution: B
Justification: Much of the present Willingdon Island was claimed from the Lake of Kochi, filling in dredged soil
around a previously existing, but tiny, natural island. It is a major port in India, and a landmark in the city of
Kochi. Willingdon Island is connected to the mainland by Venduruthy Bridge, which has road and railway
links. The headquarters of the Southern Naval Command of the Indian Navy is located on the island. Cochin
shipyard is also located near this island. It is also a major tourist centre. The idea of developing a new port in
Kochi was first felt by Sir Robert Bristow, who was appointed by Lord Willingdon, then-the Governor of
Madras Presidency, to create a new modern port on the West coast of India at Kochi.

360. India and Pakistan have been engaged in a territorial dispute on Sir Creek. What is the importance of the
Sir Creek region?
1. It is considered to be among the important fishing grounds in Asia.

Direct PDF - https://telegram.me/UPSC_PrelimsTest


https://telegram.me/UPSC_PDF

OGP 2019 GEOGRAPHY MODULE MCQS

2. Pakistan is unilaterally mining oil and gas from the region and reaping commercial profits.
3. All the rivers of the Indus system meet the Arabian Sea at Sir Creek.

Select the correct answer using the codes below.


a) 1 and 2 only
b) 1 only
c) 3 only
d) 2 and 3 only

Solution: B
Justification: Sir Creek is a nearly hundred km strip of water disputed between India and Pakistan in the Rann
of Kutch marshlands. The Creek opens up in the Arabian Sea and roughly divides the Kutch region of Gujarat
from the Sindh Province of Pakistan.
Statement 1: Apart from its strategic location near Gujarat-Pakistan border, Sir Creek’s core importance is
fishing resources.
Statement 2: Another vital reason for two countries locking horns over this creek is the possible presence of
great oil and gas concentration under the sea, which are currently unexploited due to the impending
deadlock on the issue.

361. The fact that Puducherry is warmer than Kolkata can be attributed to
a) Latitude
b) Altitude
c) Distance from sea
d) Monsoon effects

Solution: A
Justification: Pudducherry being closer to the equator receives greater solar insolation than Calcutta; hence
the warmth. The difference in altitude is not considerable to cause noticeable variations in temperature.
Moreover, both are coastal cities. So, (b) and (c) are wrong.
It is correct that Pudducherry receives a lot of rainfall from the South-west Monsoon winds, and that does not
allow Chennai to cool in late summer. But, here (a) is a more appropriate option as monsoon lasts only for 2
months whereas the temperature effects can be noticed year round

362. Consider the following important religious places in India and their location.
1. Brindavana: Mount Govardhana
2. Thiruvannamalai: Arunachala Mountain
3. Ranakpur: Mount Abu

Select the correct answer using the codes below.


a) 1 and 2 only
b) 2 and 3 only
c) 1 and 3 only
d) 1, 2 and 3

Solution: A
Justification: Statement 1: Known as Govardhan or Giriraj it is the sacred center of Braj and is identified as a
natural form of the Krishna himself (Govardhana sila).
Statement 2: It is one of the five main Shaivite holy places in South India. The Annamalaiyar Temple, a temple
of Lord Shiva is located at the base of the hill. It is also an important place for devotees of Sri Ramana
Maharshi, with Sri Ramana Ashram situated at its foothills. It is believed that circumambulation of the hills
blesses one with the seed of spiritual liberation as it is the abode of Shiva.
Statement 3: It is about 160 km from Mount Abu. The renowned Jain temple at Ranakpur is dedicated to
Tirthankara Rishabhanatha.

Direct PDF - https://telegram.me/UPSC_PrelimsTest


https://telegram.me/UPSC_PDF

OGP 2019 GEOGRAPHY MODULE MCQS

363. A quadrilateral with the largest area can be made by joining which of the following regions of India?
a) Varansi, Bhopal, Nagpur (MP), Kolkata
b) Hyderabad, Chennai, Bengaluru and Vishakhapattnam
c) Panaji, Madurai, Vishakhapattnam and Port Blair
d) Udaipur, Kandla, Ahmedabad and Mumbai

Solution: C
Justification: The question was easy if you spotted the Port Blair in Option C. The quadrilateral formed thus
will be much larger in area than the others.

364. Arrange the following Gulfs of South Asia from North to South.
1. Gulf of Martaban
2. Gulf of Khambat
3. Gulf of Mannar
4. Gulf of Kutch

Select the correct answer using the codes below.


a) A. 1243
b) B. 4213
c) C. 2143
d) D. 1324

Solution: B
Justification: Gulf of Kutch lies northwards to Gulf of Khambat in Gujarat. So, 2 must come after 4. Gulf of
Martaban is near Myanmar. Gulf of Mannar is near Tamilnadu. So, 3 must come in the end and correct
answer must be B.

Direct PDF - https://telegram.me/UPSC_PrelimsTest


https://telegram.me/UPSC_PDF

OGP 2019 GEOGRAPHY MODULE MCQS

365. To a line joining New Delhi and Bengaluru, which of these places will be at the farthest distance taken
perpendicular to the line

a) Gwalior
b) Hyderabad
c) Bhopal
d) Nashik

Solution: D

Justification:

366. Straddling the international boundary line between India and Myanmar, this district is a major centre of
handicrafts in North-eastern India and was historically known for headhunting. It is?
a) Mon, Nagaland
b) Nalbari, Assam

Direct PDF - https://telegram.me/UPSC_PrelimsTest


https://telegram.me/UPSC_PDF

OGP 2019 GEOGRAPHY MODULE MCQS

c) Tura, Tripura
d) Pasighat, North Arunachal Pradesh

Solution: A
Justification: Option B, C and D: These regions (except the state of AP) do not border Myanmar and you could
have easily picked up the right option. Mon is the land of the captivating Konyak Nagas, whose culture and
traditions are an attraction by themselves for the visitors. The forefathers of the Konyak believed that they
were direct descendants of Noah, for they have biblical names like Mosa, Kaisa Aron and so on. It is also
believed that they crossed the historic gate known as Alemkaphan which is interpreted in Konyak as the gate
of the sun. The rulers of the villages still use the word Wang (Angh) for themselves, meaning ‘the beginning of
everything’. The Angh still enjoys considerable power over his people, acting as an autocrat and a democrat.
His house is a demonstration of tribal power and glory, flashing both human and animal skulls on the porch.
The Konyaks are known for their tattooed faces, blackened teeth and head hunting prowess, the last
thankfully being in the past.

367. Tropic of cancer does NOT pass through which of these Indian states?

a) Gujarat
b) Rajasthan
c) Madhya Pradesh
d) Bihar

Solution: D
Justification: Other states that it passes through are: Chhattisgarh, Jharkhand, West Bengal, Tripura, and
Mizoram.

368. With reference to the Palk Strait, consider the following statements.
1. It connects the state of Tamilnadu in India to Sri Lanka.
2. No river flows into the strait.
3. The popular Ram Setu is located near the strait.
4. The deep waters and reefs of the strait makes it a much-sought alternative shipping route for large cargo.

Select the correct answer using the codes below.


a) 1 and 3 only
b) 1, 3 and 4 only
c) 1 and 2 only
d) D. 1, 2, 3 and 4

Solution: A
Justification: Statement 1: It is a strait between Tamil Nadu and the Mannar district of the Northern Province
of Sri Lanka. It connects the Bay of Bengal in the northeast with the Palk Bay in the southwest.

Direct PDF - https://telegram.me/UPSC_PrelimsTest


https://telegram.me/UPSC_PDF

OGP 2019 GEOGRAPHY MODULE MCQS

Statement 2: Several rivers flow into it, including the Vaigai River of Tamil Nadu.
Statement 3: Palk Bay is studded at its southern end with a chain of low islands and reef shoals that are
collectively called Adam's Bridge, since ages it is popularly known in Hindu Mythology as "Ram Setu" i.e. The
Bridge of Rama.
Statement 4: The shallow waters and reefs of the strait make it difficult for large ships to pass through,
although fishing boats and small craft carrying coastal trade have navigated the strait for centuries. Large
ships must travel around Sri Lanka.
Sethusamudram Shipping Canal Project is a proposed project to create a shipping route in the shallow straits
between India and Sri Lanka. The channel would be dredged in the Sethusamudram Sea between Tamil Nadu
and Sri Lanka, passing through the limestone shoals of Adam's Bridge (Ram Setu). Hence, it has been opposed
by religious groups.

369. A straight line joining Dehradun to Vijayawada will pass through how many states, excluding the origin
and destination?

a) 3
b) 4
c) 5
d) 6

Solution: C
Justification: It will pass through UP, MP, Maharashtra, Chhattisgarh and Telangana apart from UK and AP.
We will keep covering India map based questions in upcoming tests.

370. The Petrapole-Benapole Integrated Check Post is a major route for

a) India-Bangladesh bilateral trade


b) India-Bhutan border trade
c) India-China transit trade
d) India-Pakistan Wagha border trade

Direct PDF - https://telegram.me/UPSC_PrelimsTest


https://telegram.me/UPSC_PDF

OGP 2019 GEOGRAPHY MODULE MCQS

Solution: A
Justification: In order to facilitate movement of cargo across the border, India and Bangladesh have recently
agreed to operate Petrapole-Benapole ICP 24x7. The 24x7 operationalization of Petrapole-Benapole ICP is
expected to be a significant milestone towards expeditious clearance of cargo and, hence boost the bilateral
trade between the two countries. Land Ports Authority of India (LPAI) and Central Board of Excise and
Customs (CBEC) are the implementing agencies for this agreement from the Indian side.

371. If China were to takeover Doklam plateau, which region in or near India would be most affected by this
move?

a) Leh and Ladakh


b) Siliguri Corridor
c) India-Uttarakhand Border
d) Sundarban delta

Solution: B
Justification: China is building a road project in the region which as per Bhutan would run from the town of
Dokola to the Bhutanese army camp at Zompelri. This is seen by Bhutan as a direct violation of the
agreements between two nations. India has said the Chinese road project threatens its access to the Siliguri
corridor that connects our mainland to north-eastern states.

372. The Mekedatu dispute recently seen in news is between

a) Karnataka and Tamil Nadu


b) Government of India and Government of Sri Lanka
c) Fishermen community from Kerala and Northern province of Sri Lanka
d) Andhra Pradesh, Telangana and Karnataka

Solution: A
Justification: Karnataka intends to build a reservoir across river Cauvery near Mekedatu in Kanakapura taluk.
It was first proposed along with Shivanasamudra hydro power project in 2003 with an intention to use the
water for a hydro power station and supply drinking water to Bengaluru city. However, Tamil Nadu objected
saying Karnataka had not sought prior permission for the project. Its argument was that the project would
affect the flow of Cauvery water to Tamil Nadu.

373. World's first sanctuary for White Tigers is located in which of these states of India?

a) Gujarat
b) Assam
c) Madhya Pradesh
d) Uttar Pradesh

Solution: C
Justification: In India, white tigers are predominantly found in Rewa, Madhya Pradesh. In 2016, world’s first
White Tiger Safari was inaugurated in Satna district of Madhya Pradesh. White tigers are found in other major
zoos across India as well, for e.g. Nandankanan Zoological Park, Odisha. According to the scientists, white
tigers lack pheomelanin, which is responsible for the red-yellow hue in the skin coat.

374. The Southernmost of these mountains ranges is

a) Karakoram range
b) Pir Panjal range
c) Kaimur range

Direct PDF - https://telegram.me/UPSC_PrelimsTest


https://telegram.me/UPSC_PDF

OGP 2019 GEOGRAPHY MODULE MCQS

d) Zanskar range

Solution: C
Justification: Kaimur Range is the eastern portion of the Vindhya Range extending from around Katangi in
Jabalpur district of Madhya Pradesh to around Sasaram in Rohtas district of Bihar.

375. What is the correct order of these major regions in J&K from east to west?

a) Srinagar, Kargil, Leh, Shyok


b) Kargil, Srinagar, Shyok, Leh
c) Shyok, Leh, Kargil, Srinagar
d) Leh, Shyok, Srinagar, Kargil

Solution: C
Justification:

SHYOK

376. Manali - Leh highway crosses which of these passes?


1. Rohtang la
2. Baralacha la
3. Lungalacha la
4. Tanglang la

Select the correct answer using the codes below.


a) 2 and 3 only
b) 1 and 4 only
c) 1, 2 and 3 only
d) 1, 2, 3 and 4

Direct PDF - https://telegram.me/UPSC_PrelimsTest


https://telegram.me/UPSC_PDF

OGP 2019 GEOGRAPHY MODULE MCQS

Solution: D
Justification: The highway opens only between July and September when snow is cleared from the road.
Statement 1: Rohtang is a high mountain pass on the eastern Pir Panjal Range of the Himalayas. It connects
the Kullu Valley with the Lahaul and Spiti Valleys of Himachal Pradesh.
Statement 2: It is a high mountain pass in Zanskar range.
Statement 4: It is situated in Ladakh region of the Indian state of Jammu and Kashmir.

377. Consider the following monasteries and their locations in India.


1. Hemis: Sikkim
2. Thiksey: J&K
3. Shey: Uttarakhand
4. Lamayuru: Himachal Pradesh

Select the correct answer using the codes below.


a) 1, 3 and 4 only
b) 2 only
c) 2 and 3 only
d) D. 1, 2, 3 and 4

Solution: B
Justification: Statement 2: Thiksay Monastery is a gompa affiliated with the Gelug sect of Tibetan Buddhism
located in east of Leh in Ladakh.
Statement 3: The Shey Monastery or Gompa and the Shey Palace complex are structures located south of Leh
in Ladakh on the Leh-Manali road. Shey was the summer capital of Ladakh in the past.
Statement 1 and 4: Both of them are located in Ladakh as well.

378. It serves as an international corridor for Asian elephant migration between India and Bhutan as it runs
along the international border with Bhutan. The Sinchula hill range lies all along the northern side of this
reserve and the eastern boundary touches that of the Assam state. It is?

a) Kaziranga Tiger Reserve


b) Valmiki Tiger Reserve
c) Achanakmar Tiger Reserve
d) Buxa Tiger reserve

Solution: D
Justification: Buxa consists of moist, deciduous and evergreen forests. The Phipsu Wildlife Sanctuary of
Bhutan is contiguous to the north of BTR. Manas National Park lies on east of BTR. BTR, thus, serves as
international corridor for Asian elephant migration between India and Bhutan. Over the past few years, the
number of tigers in the Buxa reserve has been hotly debated by wildlife enthusiasts.

WORLD BASED

379. Which of these nations lie both in Northern and Southern Hemisphere?
1. Brazil
2. Maldives
3. Thailand
4. Morocco

Select the correct answer using the codes below.


a) 1 and 2 only
b) 3 and 4 only

Direct PDF - https://telegram.me/UPSC_PrelimsTest


https://telegram.me/UPSC_PDF

OGP 2019 GEOGRAPHY MODULE MCQS

c) 1, 2 and 3 only
d) 2, 3 and 4 only

Solution: A
Justification:

380. The major tectonic plates that converge on Western Asia are
1. Indian Plate
2. African plate
3. Arabian Plate
4. Pacific plate

Select the correct answer using the codes below.


a) 2 and 3 only
b) 3 and 4 only
c) 1, 2 and 3 only
d) 2, 3 and 4 only

Solution: A
Justification: The outer shell of the earth, the lithosphere, is broken up into tectonic plates. The seven major
plates are the African plate, Antarctic plate, Eurasian plate, Indo-Australian plate, North American plate,
Pacific plate and South American plate. The West Asian convergence includes the African, Eurasian, and
Arabian plates. The boundaries between the tectonic plates make up the Azores-Gibraltar Ridge, extending
across North Africa, the Red Sea, and into Iran. The Arabian Plate is moving northward into the Anatolian
plate (Turkey) at the East Anatolian Fault, and the boundary between the Aegean and Anatolian plate in
eastern Turkey is also seismically active

381. A country is likely to have multiple time zones if


a) It has a large area.

Direct PDF - https://telegram.me/UPSC_PrelimsTest


https://telegram.me/UPSC_PDF

OGP 2019 GEOGRAPHY MODULE MCQS

b) It is closer to the equator.


c) If it is closer to the poles.
d) None of the above has any bearing on the time zones of a country.

Solution: A
Justification: If a nation is spread on a large geographical area latitudinally, the Sun will be at different
positions in the sky for different latitudes on the nation at the same time. So, there is a possibility of having
multiple time zones. For e.g. in Russia, there are about 11 time zones. The same cannot be said for longitude
as it does not concern with the movement of the Sun or daylight.

382. Nations bordering Russia include


1. Mongolia
2. Uzbekistan
3. China
4. Turkmenistan

Select the correct answer using the codes below.


a) 1 and 3 only
b) 1, 2 and 3 only
c) 2 and 4 only
d) D. 1, 2, 3 and 4

Solution: A
Learning

383. If Iceland, Finland, Turkey and Spain are the four vortexes of a Quadrilateral, assuming the vortexes to be
at the geometrical centre of each nation, the area of the quadrilateral would encompass which of these
regions?
1. Caspian Sea
2. Poland
3. Italy
4. North Sea

Select the correct answer using the codes below.

Direct PDF - https://telegram.me/UPSC_PrelimsTest


https://telegram.me/UPSC_PDF

OGP 2019 GEOGRAPHY MODULE MCQS

a) 1 and 2 only
b) 2 and 3 only
c) 1 and 4 only
d) 2 and 4 only

Solution: B
Justification:

384. Arrange these geographical regions from the West to east.


1. Gulf of Aden
2. Laccadive Sea
3. Gulf of Thailand
4. South China Sea

Select the correct answer using the codes below.


a) 1234
b) 2143
c) 2134
d) 1243

Solution: A

Direct PDF - https://telegram.me/UPSC_PrelimsTest


https://telegram.me/UPSC_PDF

OGP 2019 GEOGRAPHY MODULE MCQS

385. Syrian Desert encompasses which of these nations?


1. Iraq
2. Kuwait
3. Iran
4. Jordan

Select the correct answer using the codes below.


a) 1, 2 and 3 only
b) 1 and 4 only
c) 3 and 4 only
d) 1 and 3 only

Solution: B
Justification: The Syrian Desert was frequently in news due to the ongoing civil wars and conflicts. It is a
combination of steppe and true desert of the Middle East, including parts of south-eastern Syria, north-
eastern Jordan, northern Saudi Arabia, and western Iraq. To the south it borders and merges into the Arabian
Desert. Several parts of the Syrian Desert have been referred to separately such as the Palmyrene desert
around Palmyra.

Direct PDF - https://telegram.me/UPSC_PrelimsTest


https://telegram.me/UPSC_PDF

OGP 2019 GEOGRAPHY MODULE MCQS

386. Match the following regions in news with the country they are located in:
1. Al Qaim: Syria
2. Deir al-Zour: Iraq
3. Raqqa: Saudi Arabia

Select the correct answer using the codes below.


a) 1 and 2 only
b) 3 only
c) 1 only
d) None of the above

Solution: D
Justification: Syria and Iraq have taken control of Islamic State terror group’s two strongholds in their
countries. Syria’s army announced that it has taken Deir al-Zour. Iraqi Prime Minister (Haider al-Abadi) said,
al-Qaim, across the border, had been recaptured by the government forces. Last month, a US-backed Syrian
alliance took control of Raqqa in Syria, the former capital of Islamic State’s self-styled caliphate. In July 2017,
Mosul – Iraq’s second-largest city – was taken from IS after months of fighting.

387. With regard to total length of land borders, which country has the longest of them all?
a) China
b) Russia
c) India
d) United States

Solution: A

Direct PDF - https://telegram.me/UPSC_PrelimsTest


https://telegram.me/UPSC_PDF

OGP 2019 GEOGRAPHY MODULE MCQS

Justification: Borders along lakes, rivers, and other internal waters are considered land borders for the
purposes of this calculation. When it comes to the total length of borders – both land and coastal, Russia will
have the largest one.

388. Consider the following regions and the water body near which they are located:
1. Qatar: A. Mediterranean Sea
2. Cyprus: B. Red Sea
3. Mecca: C. Persian Gulf

Select the correct answer using the codes below.


a) 1A, 2B, 3C
b) 1C, 2A, 3B
c) 1B, 2C, 3A
d) 1C, 2B, 3A

Solution: B
Justification:

Direct PDF - https://telegram.me/UPSC_PrelimsTest


https://telegram.me/UPSC_PDF

OGP 2019 GEOGRAPHY MODULE MCQS

389. Nations that are sandwiched between Black Sea and Caspian Sea are?
1. Iran
2. Georgia
3. Syria
4. Jordan

Select the correct answer using the codes below.


a) 1, 2 and 4 only
b) 1 and 3 only
c) 2 only
d) 2 and 3 only

Solution: C
Justification:

Direct PDF - https://telegram.me/UPSC_PrelimsTest


https://telegram.me/UPSC_PDF

OGP 2019 GEOGRAPHY MODULE MCQS

390. Which of these European nations lie completely to the north of Turkey?
1. Spain
2. Greece
3. Germany
4. Ukraine
5. Italy

Select the correct answer using the codes below.


a) 1, 3, 4 and 5 only
b) 3 and 4 only
c) 2, 3 and 4 only
d) 1, 2 and 5 only

Solution: B
Justification:

Direct PDF - https://telegram.me/UPSC_PrelimsTest


https://telegram.me/UPSC_PDF

OGP 2019 GEOGRAPHY MODULE MCQS

391. On the map of South Asia, if a circle is drawn with Hambantota and Gwadar being at two diametrically
opposite ends, the circle would encompass which of these regions?
1. Some islands of Maldives
2. Sumatra
3. Seychelles

Select the correct answer using the codes below.


a) 1 and 2 only
b) 1 only
c) 1 and 3 only
d) 2 and 3 only

Solution: B
Justification:

Direct PDF - https://telegram.me/UPSC_PrelimsTest


https://telegram.me/UPSC_PDF

OGP 2019 GEOGRAPHY MODULE MCQS

392. Consider the following statements.


1. Dead Sea is the world’s largest lake.
2. Lake Tanganyika is World’s lowest situated lake.
3. Caspian Sea, which is a crucial water body in Eurasia, is a rift valley lake.

Select the correct answer using the codes below.


a) 1 and 2 only
b) 2 and 3 only
c) 3 only
d) None of the above

Solution: D
Justification: S3: Due to the warping, sagging, bending and fracturing of the earth’s crust, tectonic
depressions occur. Such depressions give rise to lakes of immense sizes and depths. They include Lake
Titicaca, occupying a huge depression in the intermont plateau of the Andes, 12,500 feet above sea level the
highest lake in the world; and the Caspian Sea, 143,550 square miles, the largest lake, almost 5 times larger
than its nearest rival, Lake Superior.
S1 and S2: Due to faulting, a rift valley is formed by the sinking of land between two parallel faults, deep,
narrow and elongated in character. Water collects in these troughs and their floors are often below sea level.
It includes such lakes as Lakes Tanganyika (4,700 feet deep, the world’s deepest lake), Malawi, Rudolf,
Edward. Albert, as well as the Dead Sea 1,286 feet below mean sea level, the world’s lowest lake. The best
known example is the East African Rift Valley which runs through Zambia, Malawi, Tanzania, Kenya and
Ethiopia, and extends along the Red Sea to Israel and Jordan over a total distance of 3,000 miles.

393. It holds 20% of the world's fresh surface water and is the world's largest freshwater lake by volume. It is
a) Caspian Sea
b) Lake Baikal
c) Lake Superior
d) Victoria Lake

Direct PDF - https://telegram.me/UPSC_PrelimsTest


https://telegram.me/UPSC_PDF

OGP 2019 GEOGRAPHY MODULE MCQS

Solution: B
Justification: This is a lateral explanation (covers peripheral subjects as well). Most of all inland salt water on
earth is held in the Caspian Sea. But, about 20 percent of global freshwater is held in Lake Baikal and the
Great Lakes of North America.
 Lake Baikal is an estimated 5,387 feet deep (1,642 meters), and its bottom is approximately 3,893 feet
(1,187 meters) below sea level.
 The salty Caspian Sea has the greatest surface area of any lake at 143,200 square miles (370,886 square
kilometers).
 Lake Superior, on the United States/Canada border, is the named freshwater lake with the greatest
surface area at 31,700 square miles (82,103 square kilometers).

394. The Strait of Gibraltar is one of the busiest shipping routes in the world because
a) The strait is free from generation of internal waves and high faring currents
b) Being near Mediterranean region, sea level is at the higher side aiding ship movement through higher
gradient
c) It has the most indented coastline in Europe providing ships a natural transit route.
d) The strait is the only connection between the Atlantic Ocean and the Mediterranean Sea

Solution: D
Justification: It lies between the southern coast of Spain and the northern coast of Morocco, is the only place
where water from the Atlantic Ocean mixes with water from the Mediterranean Sea. As water flows into and
out of the Mediterranean, two currents are formed in the strait. An upper layer of Atlantic water flows
eastward into the sea over a lower layer of saltier and heavier Mediterranean water flowing westward into
the ocean, known as the Mediterranean Outflow water.

395. The strategic Port of Duqm, to which India has recently secured access for military use and logistical
support, is nearest to
a) Strait of Hormuz
b) Straits of Malacca
c) Strait of Gibraltar
d) Panama Canal

Solution: A
Justification: Securing access to key Port of Duqm for military use and logistical support was one of the key
takeaways of Prime Minister’s visit to Oman. The Port of Duqm is strategically located situated on south-
eastern seaboard of Oman, overlooking Arabian Sea and Indian Ocean. It is straddled along critical sea lanes
in Arabian Sea and Gulf of Aden. It is located in close proximity to Chabahar port in Iran (being developed by
India open a trade route to Afghanistan and Central Asia, bypassing Pakistan) and Gwadar Port in Pakistan
which is developed by China as part of CPEC (China–Pakistan Economic Corridor). The port also has special
economic zone, where about $1.8 billion investments are being made by some Indian companies. The access
to Duqm port fits into India’s proactive maritime security roadmap along with development string of military
facilities at Assumption Island in Seychelles and Agalega in Mauritius to counter China’s increasing presence

Direct PDF - https://telegram.me/UPSC_PrelimsTest


https://telegram.me/UPSC_PDF

OGP 2019 GEOGRAPHY MODULE MCQS

396. Gulf of Guinea is bordered by


1. Kenya
2. Tanzania
3. Ethiopia
4. Ghana
5. Sierra Leone

Select the correct answer using the codes below.


a) 1, 4 and 5 only
b) 1, 2 and 3 only
c) 4 and 5 only
d) 1, 2, 4 and 5 only

Solution: C
Justification: Statement 1, 2 and 3: They are on the eastern side of Africa, near the Gulf of Aden. Gulf of
Guinea is on the western Side.

397. Micronesia is
a) The lower end of the Arakan Mountains in South-east Asia
b) An extension of the West Asian Peninsular plateau
c) A sub-region of Oceania in the western Pacific Ocean

Direct PDF - https://telegram.me/UPSC_PrelimsTest


https://telegram.me/UPSC_PDF

OGP 2019 GEOGRAPHY MODULE MCQS

d) The Northern region of Russia extending into the Arctic Ocean

Solution: C
Justification: It is composed of thousands of small islands in the western Pacific Ocean. It has a shared
cultural history with two other island regions, Polynesia to the east and Melanesia to the south. The region
has a tropical marine climate, and is part of the Oceania Eco zone. There are four main archipelagos along
with numerous outlying islands. The Micronesia region encompasses five sovereign, independent nations—
the Federated States of Micronesia, Palau, Kiribati, Marshall Islands, and Nauru—as well as three U.S.
territories in the northern part: Northern Mariana Islands, Guam, and Wake Island. Micronesia began to be
settled several millennia ago, although there are competing theories about the origin and arrival of the first
settlers.

398. The States that border the Black Sea are


1. Romania
2. Turkey
3. Poland
4. Serbia
5. Greece

Select the correct answer using the codes below.


a) 1, 2, 3 and 5 only
b) 2, 3 and 4 only
c) 1 and 2 only
d) 1, 2 and 5 only

Solution: C
Justification:

Direct PDF - https://telegram.me/UPSC_PrelimsTest


https://telegram.me/UPSC_PDF

OGP 2019 GEOGRAPHY MODULE MCQS

399. Which of these places, frequently seen in news, is closest to the Caspian Sea?
a) Ashgabat
b) Kabul
c) Kandahar
d) Farkhor

Solution: A
Justification:

Direct PDF - https://telegram.me/UPSC_PrelimsTest


https://telegram.me/UPSC_PDF

OGP 2019 GEOGRAPHY MODULE MCQS

TAPI gas pipeline project is proposed trans-country natural gas pipeline from Turkmenistan to India through
Afghanistan and Pakistan. The pipe line connects central Asia with south Asia covering 1,840 km. The pipeline
runs through Galkynysh field (Turkmenistan), Afghanistan’s Herat and Kandahar province, Pakistan’s Multan
via Quetta and ends at Fazilka in Punjab (India). For its security, an inter-government joint security task force
(JSTF) has been recommended to serve as nucleus of safety provided by security consultants.

400. India has contributed USD one million for the rehabilitation efforts in Tonga after the Tropical Cyclone
Gita caused massive destruction in the island. Where is Tonga located?
a) Southern Pacific Ocean
b) Northern Atlantic Ocean
c) Indian Ocean
d) South Atlantic Ocean

Solution: A
Justification: Tonga is a Polynesian sovereign state and archipelago comprising 169 islands, of which 36 are
inhabited. The total surface area is about 750 square kilometres scattered over 700,000 square kilometres of
the southern Pacific Ocean. India has allocated USD 500,000 in the India-UN Development Partnership Fund
for the rehabilitation efforts while USD 500,000 will be provided for immediate relief assistance.

Direct PDF - https://telegram.me/UPSC_PrelimsTest


https://telegram.me/UPSC_PDF

OGP 2019 GEOGRAPHY MODULE MCQS

401. Consider the following statements about a European Country.


1. This country is poorly endowed with natural resources compared to most other Western European
countries.
2. Mountainous terrain and an unfavourably dry climate make the country poorly suited for farming.
3. There are not significant deposits of coal or iron ore.
4. It is also divided between an agriculturally and industrially well-endowed north and a much poorer south.
5. Even with the abovementioned limitations, it rates as one of the world's most important industrial power.

The above refer to?


a) Belgium
b) Germany
c) Portugal
d) Italy

Solution: D
Justification: Option D: Despite all these limitations, one economic advantage for Italy, like all other Southern
European nations, is the influx of tourism into the country, which helps raise money in the poorer southern
economies. The center of industry is in Milan and the northern areas, while the southern areas of Italy are
engaged primarily in farming and tourism.
Option C: Portugal is the poorest of the major nations. It is also the nation with the economy most dependent
upon agriculture

402. Which of the following canal connects the Atlantic Ocean in the east to the Pacific Ocean in the west?
a) Suez Canal
b) Panama Canal
c) Hormuz Canal

Direct PDF - https://telegram.me/UPSC_PrelimsTest


https://telegram.me/UPSC_PDF

OGP 2019 GEOGRAPHY MODULE MCQS

d) Vistok Canal
Solution: B
Justification:

It shortens the distance between New York and San Francisco by 13,000 km by sea. Likewise the distance
between Western Europe and the West-coast of U.S.A.; and North-eastern and Central U.S.A. and East and
South-east Asia is shortened. The economic significance of this Canal is relatively less than that of the Suez.
However, it is vital to the economies of Latin America

403. Which of the following are landlocked states in Europe?


a) Poland, Switzerland, Austria, Hungary, Luxembourg
b) Czech Republic, Belgium, Austria, Hungary, Luxembourg
c) Switzerland, Austria, Hungary, Czech Republic, Luxembourg
d) Czech Republic, Switzerland, Austria, Hungary, Denmark

Solution: C
Justification: There are 17 landlocked countries in Europe: Andorra, Armenia, Austria, Belarus, Bosnia and
Herzegovina, Czech Republic, Hungary, Kazakhstan, Liechtenstein, Luxembourg, Macedonia, Moldova, San
Marino, Serbia, Slovakia, Switzerland and Vatican City.

404. Consider the following statements.


Assertion (A): No river drains in the Arctic Ocean.
Reason (R): Arctic Ocean is at a higher average elevation than the Northern Hinterland of Eurasia.
In the context of the above, which of these is correct?

Direct PDF - https://telegram.me/UPSC_PrelimsTest


https://telegram.me/UPSC_PDF

OGP 2019 GEOGRAPHY MODULE MCQS

a) A is correct, and R is an appropriate explanation of A.


b) A is correct, but R is not an appropriate explanation of A.
c) A is correct, but R is incorrect.
d) Both A and R are incorrect.

Solution: D
Justification: Unfortunately, over the greater part of Siberia, all the rivers drain polewards into the Arctic
Ocean which is frozen for the three-quarters of the years. With the use of Northern Sea route, which links
Murmansk and Vladivostok via the Arctic Ocean, development is increasing. Cheap hydro-electricity for
driving the saw-mills is harnessed in the mountainous uplands of North America and Europe and has greatly
assisted the lumbering industry.

405. The straits nearest to the International Date Line is


a) Straits of Malacca
b) Bering Strait
c) Strait of Hormuz
d) Strait of Papua New Guinea

Solution: B
Justification: It is an imaginary line of navigation on the surface of the Earth that runs from the North Pole to
the South Pole and demarcates the change of one calendar day to the next. It passes through the middle of
the Pacific Ocean. The IDL passes through the Bering Strait between the Diomede Islands.

406. What is Point Nemo, which was recently seen in news?


a) It is an Arctic space observatory.
b) It is a launchpad used by most global space agencies.
c) It is the location in the ocean that is farthest from land.
d) It is the orbital centre of the International Space Station (ISS).

Solution: C
Justification: You can't get farther away from land than 'Point Nemo.' Named after the famous submarine
sailor from Jules Verne's Twenty Thousand Leagues under the Sea, it is located in Pacific Ocean. This remote
oceanic location is 2,688 kilometers from the nearest land—Ducie Island, part of the Pitcairn Islands, to the
north; Motu Nui, one of the Easter Islands, to the northeast; and Maher Island, part of Antarctica, to the
south. It is also known as the spacecraft cemetery as between 1971 and mid-2016, space agencies all over the
world have dumped between 260 and 300 spacecraft into the region.

Direct PDF - https://telegram.me/UPSC_PrelimsTest


https://telegram.me/UPSC_PDF

OGP 2019 GEOGRAPHY MODULE MCQS

407. Which of the following regions come under the Arctic Circle?
1. Oslo
2. Greenland
3. Alaska
4. Sakhalin

Select the correct answer using the codes below.


a) 1 and 4 only
b) 2 and 3 only
c) 1, 2 and 3 only
d) 2, 3 and 4 only

Solution: B
Justification:

Direct PDF - https://telegram.me/UPSC_PrelimsTest


https://telegram.me/UPSC_PDF

OGP 2019 GEOGRAPHY MODULE MCQS

Direct PDF - https://telegram.me/UPSC_PrelimsTest


https://telegram.me/UPSC_PDF

OGP 2019 GEOGRAPHY MODULE MCQS

408. Consider the following statements. The Ross Sea is


1. A deep bay of the Southern Ocean in Antarctica
2. Free of ice for most of the year
3. Dominated by movements of cold counter-equatorial currents

Select the correct answer using the codes below.


a) 1 only
b) 2 and 3 only
c) 1 and 2 only
d) 1 and 3 only

Solution: A
Justification: Statement 1: To the west of the sea lies Ross Island and to the east Roosevelt Island, while the
southernmost part is covered by the Ross Ice Shelf, and is about 200 miles from the South Pole.
Statement 2 and 3: The circulation of the Ross Sea is dominated by a wind-driven ocean gyre and the flow is
strongly influenced by three submarine ridges that run from southwest to northeast. The circumpolar deep
water current is a relatively warm, salty and nutrient-rich water mass that flows onto the continental shelf at
certain locations. But, even then Ross Sea is covered with ice for most of the year.

409. The Bering Sea is a marginal sea of the


a) Arctic Ocean
b) North Sea
c) Pacific Ocean
d) Beaufort Sea

Solution: C

Direct PDF - https://telegram.me/UPSC_PrelimsTest


https://telegram.me/UPSC_PDF

OGP 2019 GEOGRAPHY MODULE MCQS

Justification: The Bering Sea is separated from the Gulf of Alaska by the Alaska Peninsula. It comprises a deep
water basin, which then rises through a narrow slope into the shallower water above the continental shelves.
The Bering Sea ecosystem includes resources within the jurisdiction of the United States and Russia, as well as
international waters in the middle of the sea (known as the "Donut Hole").

410. The interaction between currents, sea ice, and weather makes for a vigorous and productive ecosystem.
Which of these seas surround Arctic Ocean?
1. Beaufort Sea
2. East Siberian Sea
3. Kuril Sea
4. Greenland Sea

Select the correct answer using the codes below.


a) 1 and 3 only
b) 1, 2 and 4 only
c) 2 and 4 only
d) D. 1, 2, 3 and 4

Solution: B
Justification:

411. Which of the following characterize the famous Lake Vostok?


1. It lies below the sea level.
2. It is a brackish lake.

Direct PDF - https://telegram.me/UPSC_PrelimsTest


https://telegram.me/UPSC_PDF

OGP 2019 GEOGRAPHY MODULE MCQS

Which of the above is/are correct?


a) 1 only
b) 2 only
c) Both 1 and 2
d) None

Solution: A
Justification: Statement 1: There are a number of rivers and lakes in Antarctica, the longest river being the
Onyx. The largest lake, Vostok, is one of the largest sub-glacial lakes in the world.
Statement 2: The surface of this fresh water lake is approximately 4,000 m under the surface of the ice, which
places it at approximately 500 m below sea level.
The lake water is estimated to have been sealed off under the thick ice sheet about 15 million years ago.
Living micro-organisms have been found in Lake Vostok's deep ice core drillings; despite the complete
absence of sunlight received in it. This suggests the presence of a deep biosphere utilizing a geothermal
system of the bedrock encircling the sub glacial lake. Due to the disconnection of Lake Vostok with the rest of
the world, it is a very interesting puzzle for evolutionary biologists.

412. Parties to the Antarctica Treaty System include


1. India
2. Russia
3. China
4. USA
5. Australia
Select the correct answer using the codes below.
a) 2 and 4 only
b) 2, 4 and 5 only
c) 1, 3 and 5 only
d) 1, 2, 3, 4 and 5

Solution: B
Justification: The treaty regulates international relations with respect to Antarctica. We will cover more on
this important treaty later.

Direct PDF - https://telegram.me/UPSC_PrelimsTest


https://telegram.me/UPSC_PDF

OGP 2019 GEOGRAPHY MODULE MCQS

413. Barents Sea is closest to which of these nations?


a) Finland, Sweden and Norway
b) Poland, Belarus and Ukraine
c) France and Germany
d) Morocco, Italy and Greece

Solution: A

Direct PDF - https://telegram.me/UPSC_PrelimsTest


https://telegram.me/UPSC_PDF

OGP 2019 GEOGRAPHY MODULE MCQS

Justification:

414. The coastal city of Hambantota has gained strategic significance due to
1. Being in the centre of the shipping lane of the Straits of Malacca
2. Chinese sponsored infrastructure development supported by a South Asian neighbour of India.
3. Being the largest military base for US marine troops in Asia pacific

Select the correct answer using the codes below.


a) 1 only
b) 2 and 3 only
c) 2 only
d) 1 and 3 only

Solution: C
Justification: Statement 1: Hambantota is the main town in the Southern Province of Sri Lanka. It is important
because it is right in the middle of vital energy supply lines in the Indian Ocean, connecting the Middle East
and East Asia.
Statement 2: It gained strategic significance after President Rajapaksa built a massive port and an airport with
huge Chinese loans. This year the SL government sold a majority stake of the port to China to service an
outstanding $8-billion debt it owes China, fanning concerns of countries with competing strategic interests,
particularly India and the U.S. Local residents protested the selling of “national assets to foreign entities”

Direct PDF - https://telegram.me/UPSC_PrelimsTest


https://telegram.me/UPSC_PDF

OGP 2019 GEOGRAPHY MODULE MCQS

leading to violent clashes. India has been taking steps to protect itself in the Indian Ocean by allying itself with
the United States and Japan in a clear bid to counter growing Chinese influence.

415. If you were to draw a circle on a map with Warsaw at the Centre and Berlin at the periphery, this circle
would encompass which of these regions?
1. Paris
2. Dublin
3. Madrid
4. Brussels

Select the correct answer using the codes below.


a) 1 and 2 only
b) 3 and 4 only
c) 1, 3 and 4 only
d) None of the above

Solution: D
Justification:

416. The Nile is an "international" river as its drainage basin covers eleven countries, some of which include
1. Ethiopia
2. Egypt
3. Morocco
4. Nigeria

Select the correct answer using the codes below.

Direct PDF - https://telegram.me/UPSC_PrelimsTest


https://telegram.me/UPSC_PDF

OGP 2019 GEOGRAPHY MODULE MCQS

a) 2 only
b) 1 and 2 only
c) 2, 3 and 4 only
d) 1 and 4 only

Solution: B
Justification: These countries are Tanzania, Uganda, Rwanda, Burundi, the Democratic Republic of the Congo,
Kenya, Ethiopia, Eritrea, South Sudan, Sudan and Egypt. In particular, the Nile is the primary water source of
Egypt and Sudan.

417. The cities that fall between Yellow Sea and Sea of Japan are
1. Seoul
2. Shanghai
3. Tokyo
4. Taipei

Select the correct answer using the codes below.


a) 1 only
b) 2, 3 and 4 only

Direct PDF - https://telegram.me/UPSC_PrelimsTest


https://telegram.me/UPSC_PDF

OGP 2019 GEOGRAPHY MODULE MCQS

c) 1 and 3 only
d) 2 and 4 only

Solution: A
Justification: Taipei is in Taiwan, Tokyo in Japan and Shanghai in China. Both are on the western or Southern
margins.

418. Which of these West Asian countries border Iraq as well as Persian Gulf?
1. Kuwait
2. Syria
3. Qatar
4. Saudi Arabia
5. Iran

Select the correct answer using the codes below.


a) 4 only
b) 1, 4 and 5 only
c) 2 and 3 only
d) None of the above

Solution: B

Direct PDF - https://telegram.me/UPSC_PrelimsTest


https://telegram.me/UPSC_PDF

OGP 2019 GEOGRAPHY MODULE MCQS

Justification:

419. The closest South-East Asian destination from Kolkata is?


a) Bandung
b) Jakarta
c) Surabaya
d) Bangkok

Solution: D
Justification:

Direct PDF - https://telegram.me/UPSC_PrelimsTest


https://telegram.me/UPSC_PDF

OGP 2019 GEOGRAPHY MODULE MCQS

Most of these cities are famous for recurrent international conferences

420. The Bay of Fundy in Canada is known for


1. Integrating largest volume of freshwater
2. Most indented coastline
3. Largest frequency of cyclones

Select the correct answer using the codes below.


a) 1 only
b) 2 and 3 only
c) 3 only
d) None of the above

Solution: D
Justification: It has the highest tidal range in the world. In the Bay of Fundy they occur in Nova Scotia,
Canada. The tidal bulge is 15 - 16 m. Because there are two high tides and two low tides every day (roughly a
24 hour period); then a tide must come in within about a six hour period. As a rough estimate, the tide rises
about 240 cm an hour (1,440 cm divided by 6 hours). If you have walked down a beach with a steep cliff
alongside (which is common there), make sure you watch the tides. If you walk for about an hour and then
notice that the tide is coming in, the water will be over your head before you get back to where you started!

421. The shortest route from Chennai to Vladivostok, as proposed by the Government of India, will pass
through the High seas near which of these countries?
1. Egypt

Direct PDF - https://telegram.me/UPSC_PrelimsTest


https://telegram.me/UPSC_PDF

OGP 2019 GEOGRAPHY MODULE MCQS

2. Morocco
3. China
4. Indonesia
5. Iran

Select the correct answer using the codes below.


a) 1 and 2 only
b) 3 and 4 only
c) 2 and 5 only
d) 3, 4 and 5 only

Solution: B
Justification: Government is proposing a major connectivity initiative — direct shipping link between Chennai
and Vladivostok amid China’s ambitious Maritime Silk Route (MSR) connecting Asia with Africa.

This will be a key maritime route connecting India with Northeast Asia and Western Pacific region. This
shipping link would enable to transfer cargo between Chennai and Vladivostok in 24 days in comparison to
over 40 days currently taken to transport goods from India to Far East Russia via Europe, according to experts
on the subject.

422. Canary Islands, often seen in news and also known as a tax haven, is located
1. To the east of Sahara deserts
2. To the South of Tropic of Cancer
Which of the above is/are correct?
a) 1 only
b) 2 only
c) Both 1 and 2
d) None

Solution: D
Justification: They are an archipelago and autonomous community of Spain located on the Atlantic Ocean.
The EU allows the Canary Islands Government to offer special tax concessions for investors.

Direct PDF - https://telegram.me/UPSC_PrelimsTest


https://telegram.me/UPSC_PDF

OGP 2019 GEOGRAPHY MODULE MCQS

423. A line joining Gulf of Guinea to Mozambique channel is likely to pass through
1. Namibia
2. Botswana
3. Kenya
4. Zambia
Select the correct answer using the codes below.
a) 2 and 3 only
b) 4 only
c) 1 and 4 only
d) 1, 2, 3 and 4

Solution: B

Direct PDF - https://telegram.me/UPSC_PrelimsTest


https://telegram.me/UPSC_PDF

OGP 2019 GEOGRAPHY MODULE MCQS

Justification:

424. Which of these African states border the Mediterranean Sea?


1. Tunisia
2. Egypt
3. Niger
4. Chad

Select the correct answer using the codes below.


a) 1 and 2 only
b) 3 and 4 only
c) 1 only
d) 2 and 3 only

Solution: A
Justification:

Direct PDF - https://telegram.me/UPSC_PrelimsTest


https://telegram.me/UPSC_PDF

OGP 2019 GEOGRAPHY MODULE MCQS

425. Which of these geographical features lies in Southern Africa?


a) Djouf Basin
b) Kalahari Basin
c) Sudan Basin
d) Chad Basin

Solution: B
Justification:

Direct PDF - https://telegram.me/UPSC_PrelimsTest


https://telegram.me/UPSC_PDF

OGP 2019 GEOGRAPHY MODULE MCQS

426. Which of these lies the easternmost”?


a) Cape Agulhas
b) Cape Hangklip
c) Cape point
d) Port Elizabeth

Solution: D

Direct PDF - https://telegram.me/UPSC_PrelimsTest


https://telegram.me/UPSC_PDF

OGP 2019 GEOGRAPHY MODULE MCQS

Justification:

427. Which of these African nations is closest to Red Sea?


a) Eritrea
b) Cameroon
c) Uganda
d) Ghana

Solution: A
Justification: Red sea forms the coast of Eritrea.

Direct PDF - https://telegram.me/UPSC_PrelimsTest


https://telegram.me/UPSC_PDF

OGP 2019 GEOGRAPHY MODULE MCQS

428. The largest African nation in terms of geographical area is


a) Algeria
b) South Sudan
c) Libya
d) South Africa

Solution: A
Justification: Sudan was, with an area of 2,505,813km², formerly the largest country in Africa, until South
Sudan formally split from it in 2011.

429. Cape of Good Hope borders which of the following two oceans/seas?
a) Indian and Atlantic Ocean
b) Arabian and Pacific Ocean
c) Mediterranean Sea and Caspian Sea
d) Red Sea and Arabian Sea

Solution: A
Justification: It is a rocky headland on the Atlantic coast of the Cape Peninsula, South Africa. When following
the western side of the African coastline from the equator, however, the Cape of Good Hope marks the point
where a ship begins to travel more eastward than southward. Thus, the first modern rounding of the cape in
1488 by Portuguese explorer Bartolomeu Dias was a milestone in the attempts by the Portuguese to establish
direct trade relations with the Far East.

Direct PDF - https://telegram.me/UPSC_PrelimsTest


https://telegram.me/UPSC_PDF

OGP 2019 GEOGRAPHY MODULE MCQS

430. Arrange the following African geographical features from West to East:
1. Atlas Mountains
2. Congo River basin
3. Great Rift valley
4. Horn of Africa

Select the correct answer using the codes below.


a) 1234
b) 2314
c) 3142
d) 1324

Solution: A
Justification:

Direct PDF - https://telegram.me/UPSC_PrelimsTest


https://telegram.me/UPSC_PDF

OGP 2019 GEOGRAPHY MODULE MCQS

431. There are several islands in the Mediterranean Sea, but only two are sovereign nations. These are
1. Cyprus
2. Sicily
3. Samoa
4. Malta

Select the correct answer using the codes below.


a) 1 and 2 only
b) 2 and 3 only
c) 1 and 4 only
d) 1 and 3 only

Solution: C
Justification: The Mediterranean Sea is home to over 3,300 islands, of which only two are independent
nations: Cyprus and Malta. These countries are often referred to as the Mediterranean States.

Direct PDF - https://telegram.me/UPSC_PrelimsTest


https://telegram.me/UPSC_PDF

OGP 2019 GEOGRAPHY MODULE MCQS

Statement 1: Cyprus is a country situated between Turkey and Egypt, bordered by Greece to the northwest
and Syria and Lebanon to the east. The country covers an area of 3,572 square miles, making it the third
largest island in the Mediterranean.
Statement 2: Sicily is the largest island in the Mediterranean Sea. It is an autonomous region of Italy. Sicily is
located in the central Mediterranean Sea, south of the Italian Peninsula.
Statement 4: Malta is a country located just 50 miles south of Italy, bordered by Tunisia to the west and Libya
to the south.

432. The water body that touches Scandinavian countries, Russia and Europe as well is
a) North Sea
b) Black Sea
c) Baltic Sea
d) Arctic Sea

Solution: C
Justification:

Direct PDF - https://telegram.me/UPSC_PrelimsTest


https://telegram.me/UPSC_PDF

OGP 2019 GEOGRAPHY MODULE MCQS

433. A direct flight from Paris to Copenhagen will pass through which of these countries?
1. Czech Republic
2. Germany
3. Austria
4. Poland

Select the correct answer using the codes below.


a) 1 only
b) 2 and 3 only
c) 2 only
d) 1 and 4 only

Solution: C

Direct PDF - https://telegram.me/UPSC_PrelimsTest


https://telegram.me/UPSC_PDF

OGP 2019 GEOGRAPHY MODULE MCQS

Justification:

434. The countries that will fall to the South of the line joining Kolkata and Kuala Lumpur are
1. Thailand
2. Vietnam
3. Indonesia

Select the correct answer using the codes below.


a) 1 only
b) 3 only
c) 1, 2 and 3
d) 1 and 3 only

Solution: B
Justification:

Direct PDF - https://telegram.me/UPSC_PrelimsTest


https://telegram.me/UPSC_PDF

OGP 2019 GEOGRAPHY MODULE MCQS

435. Suez Canal allows ships to travel between


a) North and South America
b) Europe and Russia
c) Europe and South Asia
d) East Africa to South-East Asia

Solution: C
Justification: It is an artificial sea-level waterway in Egypt, connecting the Mediterranean Sea to the Red Sea.
It was opened in 1869. It allows ships to travel between Europe and South Asia without navigating around
Africa thereby reducing the sea voyage distance between Europe and India by about 7,000 kilometres. The
canal is owned and maintained by the Suez Canal Authority (SCA) of Egypt. Under the Convention of
Constantinople, it may be used "in time of war as in time of peace, by every vessel of commerce or of war,
without distinction of flag”.

436. Keeping the political geography of West Asia in mind, the shortest aerial route to cross Red Sea will be via
a) Djibouti and Amman
b) Mecca and Port Sudan
c) Cairo and Baghdad
d) Abu Dhabi and Doha

Solution: B
Justification: The options mentioned are the capital/major cities of West Asia. The easy answer to this
question will be an aerial route between Saudi Arabia (Mecca) and Sudan (Port Sudan). It would have been
easier to eliminate this way.

Direct PDF - https://telegram.me/UPSC_PrelimsTest


https://telegram.me/UPSC_PDF

OGP 2019 GEOGRAPHY MODULE MCQS

437. Which of the following clearly bring(s) out the importance of South China Sea?
1. Strategically located it is a major international shipping route as a large number of ships pass through it
2. The sea is rich in energy, mineral and fishing resources

Which of the above is/are correct?


a) 1 only
b) 2 only
c) Both 1 and 2
d) None

Solution: C
Justification: Statement 1: Almost 50% of world’s merchant ships pass through it.
Statement 2: Reserves of natural resources around the sea including petroleum has been clearly found in
South China Sea. India is already exploring a few blocks in association with Vietnam.
The Permanent Court of Arbitration (PCA) in Hague has recently rejected China’s claims to economic rights
across large swathes of South China Sea. It says, there was no legal basis for China to claim historic rights to
resources within the South China Sea areas falling within the ‘nine-dash line.

Direct PDF - https://telegram.me/UPSC_PrelimsTest


https://telegram.me/UPSC_PDF

OGP 2019 GEOGRAPHY MODULE MCQS

438. Sinai Peninsula lies between


a) Gulf of Suez and Gulf of Aqaba
b) Gulf of Aden and Gulf of Aqaba
c) Gulf of Suez and Gulf of Aden
d) Gulf of Masirah and Gulf of Aqaba

Solution: A
Justification: It is situated between the Mediterranean Sea to the north and the Red Sea to the south, serving
as a land bridge between Asia and Africa. It is the only part of Egyptian territory located in Asia. Israel invaded
and occupied Sinai during the Suez Crisis. As a result of the Israel-Egypt Peace Treaty of 1979 and subsequent
efforts, Israel withdrew from all of the Sinai Peninsula.

Direct PDF - https://telegram.me/UPSC_PrelimsTest


https://telegram.me/UPSC_PDF

OGP 2019 GEOGRAPHY MODULE MCQS

439. Arrange the following regions/locations in West Asia from West to East.
1. Azerbaijan
2. Ashgabat
3. Tel Aviv
4. Damascus

Select the correct answer using the codes below.


a) 3421
b) 4312
c) 4321
d) 3412

Solution: D

Justification:

Direct PDF - https://telegram.me/UPSC_PrelimsTest


https://telegram.me/UPSC_PDF

OGP 2019 GEOGRAPHY MODULE MCQS

Tel Aviv is in Israel, whereas Damascus is in Syria. Damascus lies to the east of Tel Aviv. So, 4 must come after 3.
Rest should be clear from the map. All these places have been in news for some or the other reason and
important, for e.g. Ashgabat transport agreement.

440. Mecca, a major religious pilgrimage centre, is closest to


a) Caspian Sea
b) Black Sea
c) Red Sea
d) Persian Gulf

Solution: C
Justification:

Direct PDF - https://telegram.me/UPSC_PrelimsTest


https://telegram.me/UPSC_PDF

OGP 2019 GEOGRAPHY MODULE MCQS

441. Which of the following nations does NOT open to Mediterranean Sea?

a) Jordan
b) Syria
c) Turkey
d) Lebanon

Solution: A

Direct PDF - https://telegram.me/UPSC_PrelimsTest


https://telegram.me/UPSC_PDF

OGP 2019 GEOGRAPHY MODULE MCQS

442. Arrange the following Asian geographical features from West to East.
1. Gobi Desert
2. Plateau of Tibet
3. Yellow Sea
4. Western Himalayas

Select the correct answer using the codes below.


a) 4213
b) 4231
c) 4123
d) 2143

Solution: A
Justification:

Direct PDF - https://telegram.me/UPSC_PrelimsTest


https://telegram.me/UPSC_PDF

OGP 2019 GEOGRAPHY MODULE MCQS

443. Consider the following sites in news and the country they are located in.
1. Djibouti: Indonesia
2. Hebor: Iran
3. Gaza Strip: Egypt

Select the correct answer using the codes below.


a) 1 and 2 only
b) 2 and 3 only
c) 2 only
d) None of the above

Solution: D
Justification: Statement 1: Djibouti has acquired major significance as result of geopolitical developments and
presence of military bases of US and China in Djibouti located in the Horn of Africa. It is bordered by Eritrea in
the north, Ethiopia in the west and south, and Somalia in the southeast. The remainder of the border is
formed by the Red Sea and the Gulf of Aden at the east.
Statement 2: The UNESCO World Heritage Committee has put the West Bank city of Hebron on endangered
heritage list. This made Israel angry and triggered a new Israeli-Palestinian spat at the international body.
Hebron is part of the West Bank, a territory captured by Israel in the 1967 Mideast war. The international
community considers it to be occupied. The Old City of Hebron has sites that are holy to both Jews and
Muslims, known as either the Tomb of the Patriarchs or the alIbrahimi mosque.
Statement 3: Israel was in control of the West Bank and Gaza Strip, two territories home to large Palestinian
populations. Today, the West Bank is nominally controlled by the Palestinian Authority and is under Israeli
occupation.

Direct PDF - https://telegram.me/UPSC_PrelimsTest


https://telegram.me/UPSC_PDF

OGP 2019 GEOGRAPHY MODULE MCQS

444. Dead Sea is sandwiched between


a) Turkey and Lebanon
b) Israel and Syria
c) Jordan and Israel
d) Jordan and Turkey

Solution: C
Justification: The Dead Sea also called the Salt Sea, is a salt lake bordered by Jordan to the east and Israel and
Palestine to the west. The Dead Sea is the deepest hypersaline lake in the world. It is 9.6 times as salty as the
ocean, and one of the world's saltiest bodies of water. This salinity makes for a harsh environment in which
plants and animals cannot flourish, hence its name. It lies in the Jordan Rift Valley and its main tributary is the
Jordan River. The Dead Sea has attracted visitors from around the Mediterranean basin for thousands of
years. It has been the supplier of a wide variety of products, from asphalt for Egyptian mummification to
potash for fertilizers. People also use the salt and the minerals from the Dead Sea to create cosmetics and
herbal sachets. The Dead Sea water has a density of 1.24 kg/litre, which makes swimming similar to floating.

445. It is one of Asia's two main opium-producing areas. It overlaps the mountains of three countries of
Southeast Asia: Myanmar, Laos and Thailand. It is?

a) Arakanyoma
b) Golden triangle
c) Tien Shan mountain system
d) Extension of Tibetan Plateau

Direct PDF - https://telegram.me/UPSC_PrelimsTest


https://telegram.me/UPSC_PDF

OGP 2019 GEOGRAPHY MODULE MCQS

Solution: B
Justification: Along with Afghanistan in the Golden Crescent, it has been one of the most extensive opium-
producing areas of Asia and of the world since the 1950s. The Golden Triangle designates the confluence of
the Ruak River and the Mekong River, since the term has been appropriated by the Thai tourist industry to
describe the nearby border tripoint of Thailand, Laos and Myanmar.

446. North America is linked to South America by

a) A very narrow Isthmus of Panama


b) A sea overbridge constructed by the USA government
c) Only air and ship travel through coastal regions
d) An international highway constructed jointly by Mexico and Brazil

Solution: A
Justification: Isthmus is narrow strip of land connecting two large land bodies. A strait is a narrow
passageway of water, usually between continents or islands, or between two larger bodies of water, e.g. the
Strait of Gibraltar. A marginal sea is a sea partially enclosed by islands, archipelagos, or peninsulas. For e.g.
Arabian Sea, Baltic Sea, Bay of Bengal etc.

447. What is the highest point on Earth as measured from ‘Earth's centre’?

a) Top of Mount Chimborazo


b) Karakoram
c) Mauna Kea
d) Summit of Mount Everest

Solution: A
Justification: The answer is contrary to common perception.

Direct PDF - https://telegram.me/UPSC_PrelimsTest


https://telegram.me/UPSC_PDF

OGP 2019 GEOGRAPHY MODULE MCQS

Option D: Mount Everest is usually said to be the highest mountain on Earth. Everest is indeed the highest
point above global mean sea level—the average level for the ocean surface from which elevations are
measured. But the summit of Mt. Everest is not the farthest point from Earth’s centre.
Earth is not a perfect sphere, but is a bit thicker at the Equator due to the centrifugal force created by the
planet’s constant rotation. Because of this, the highest point above Earth’s centre is the peak of Ecuador’s
Mount Chimborazo, located just one degree south of the Equator where Earth’s bulge is greatest. The summit
of Chimborazo is 20,564 feet above sea level. However, due to the Earth’s bulge, the summit of Chimborazo is
over 6,800 feet farther from the centre of the Earth than Everest’s peak. That makes Chimborazo the closest
point on Earth to the stars.
Option C: You may be surprised to learn that Everest is not the tallest mountain on Earth, either. That honour
belongs to Mauna Kea, a volcano on the Big Island of Hawaii. Mauna Kea originates deep beneath the Pacific
Ocean, and rises more than 33,500 feet from base to peak.

448. The only continent through which the Tropic of Cancer, the Equator and the Tropic of Capricorn pass is

a) Asia
b) South America
c) Australia
d) Africa

Solution: D
Justification: refer the map

Direct PDF - https://telegram.me/UPSC_PrelimsTest


https://telegram.me/UPSC_PDF

OGP 2019 GEOGRAPHY MODULE MCQS

South America misses Tropic of Cancer, Australia misses both equator and tropic of cancer
449. Which of these places share closest latitudes?

a) Aswan, Riyadh and Abu Dhabi


b) Cairo, Istanbul and Damascus
c) Tehran, Ashgabat and Aleppo
d) Mosul, Muscat and Tel-Aviv

Solution: A
Justification: Closest latitudes mean that they are located in a narrow band of latitude. They can be located at
different longitudes.

450. Which of these nations has nationalized the Suez Canal?

a) Egypt
b) Syria
c) Israel
d) Saudi Arabia

Solution: A
Justification: The Suez Canal is considered to be the shortest link between the east and the west due to its
unique geographic location. It is an important international navigation canal linking between the
Mediterranean Sea at Port Said and the red sea at Suez. Egypt nationalized the canal in 1956.

451. Which of these nations border both Caspian Sea and Black Sea?

a) Iran
b) Turkey
c) Russia

Direct PDF - https://telegram.me/UPSC_PrelimsTest


https://telegram.me/UPSC_PDF

OGP 2019 GEOGRAPHY MODULE MCQS

d) Georgia

Solution: C
Justification:

452. Nations that border the Persian Gulf are


1. Saudi Arabia
2. United Arab Emirates
3. Iran
4. Kuwait
5. Jordan

Select the correct answer using the codes below.


a) 2 and 4 only
b) 1, 2, 3 and 4 only
c) 3 and 5 only
d) 1 and 5 only

Solution: B
Justification:

Direct PDF - https://telegram.me/UPSC_PrelimsTest


https://telegram.me/UPSC_PDF

OGP 2019 GEOGRAPHY MODULE MCQS

453. Consider the following statements.


1. Dasht-e Kavir is a great salt desert located in Arabian Peninsula.
2. Dasht-e Lut is a large desert located in Iran that has been inscribed on UNESCO's World Heritage List.

Which of the above is/are correct?


a) 1 only
b) 2 only
c) Both 1 and 2
d) None

Solution: B
Justification: Statement 1: Also known as Kavir-e Namak (literally 'salty lowlands') and the Great Salt Desert,
it is a large desert lying in the middle of the Iranian plateau.
Statement 2: The Lut Desert, widely referred to as Dasht-e Lut is a large salt desert located in the provinces of
Kerman and Sistan and Baluchestan, Iran. It is the world's 27th-largest desert, and was inscribed on UNESCO's
World Heritage List in 2016. The surface of its sand has been measured at temperatures as high as 70 °C
making it one of the world's driest and hottest places.

454. Consider the following about several major maritime choke points.
1. Strait of Mandeb connects the Red Sea to the Gulf of Aden.
2. Lombok Strait connects the Java Sea to the Indian Ocean.
3. Strait of Malacca connects North Atlantic Ocean to the South Pacific Ocean.

Select the correct answer using the codes below.


a) 1 and 2 only
b) 2 and 3 only
c) 3 only
d) 1, 2 and 3

Solution: A
Justification: Statement 1: The Bab-el-Mandeb or Mandeb Strait is a strait located between Yemen on the
Arabian Peninsula, and Djibouti and Eritrea in the Horn of Africa. It connects the Red Sea to the Gulf of Aden.

Direct PDF - https://telegram.me/UPSC_PrelimsTest


https://telegram.me/UPSC_PDF

OGP 2019 GEOGRAPHY MODULE MCQS

Statement 2: The Lombok Strait, connects the Java Sea to the Indian Ocean, and is located between the
islands of Bali and Lombok in Indonesia. The Gili Islands are on the Lombok side.
Statement 3: Strait of mandeb connects Indian Ocean to Pacific Ocean.

455. The Mekong is a trans-boundary river in Southeast Asia. It does NOT pass through which of these
countries?

a) India
b) China
c) Vietnam
d) Myanmar

Solution: A
Justification: From the Tibetan Plateau the river runs through China's Yunnan Province, Myanmar, Laos,
Thailand, Cambodia, and Vietnam. The MGC is an initiative by India and five ASEAN countries, Cambodia,
Laos, Myanmar, Thailand and Vietnam for cooperation in tourism, culture, education, as well as transport and
communications. It was launched in 2000 at Vientiane, Laos.

456. Among the following, the South-east Asian region that is closest to equator is?

a) Bangkok
b) Singapore
c) Manila
d) Yangon

Solution

Justification:

Direct PDF - https://telegram.me/UPSC_PrelimsTest


https://telegram.me/UPSC_PDF

457. Amazon basin in South America spreads over which of these countries?
1. Venezuela
2. Brazil
3. Chile
4. Argentina
5. Bolivia

Select the correct answer using the codes below.


a) 1, 2 and 5 only
b) 2, 3 and 4 only
c) 1, 3 and 4 only
d) 1, 2, 3, 4 and 5

Solution: A
Justification: It spans over Venezuela, Columbia, Ecuador, Peru, Brazil and other nations
as shown in the image below.

458. The Prairies are known as the “Granaries of the world”. They are bound by which of
these geographical features from the West and East respectively?

a) Rocky Mountains and Great Lakes


b) Lake Victoria and Andes mountain
c) Alps mountains and Zambia falls
d) Appalachian Mountains and Queensland

Solution: A
Justification: They are known as granaries due to the huge surplus of wheat production.
The temperate grasslands of North America are known as the Prairies. It is a region of
flat, gently sloping or hilly land. For the most part, prairies are treeless but, near the low

Direct PDF - https://telegram.me/UPSC_PrelimsTest


https://telegram.me/UPSC_PDF

lying plains, flanking river valleys, woodlands can be found. The prairies are bound by
the Rocky Mountains in the West and the Great Lakes in the East.

459. The Indian National Centre for Ocean Information Services (INCOIS) of the Ministry
of Earth Sciences inaugurated the Ocean Forecasting System for Indian Ocean countries
Comoros, Madagascar, and Mozambique. Consider the following about the location of
these countries?
1. They lie to the North of Somalia.
2. Madagascar lies to the North of Comoros.
3. Seychelles lies to the South of Madagascar.

Select the correct answer using the codes below.


a) 1 only
b) 2 and 3 only
c) 1 and 3 only
d) None of the above

Solution: D
Justification: The Ocean Forecast System will offer, high wave alerts, port warnings,
forecast, oil spill advisory services along the ship routes in addition to tsunami and
storm surge warnings and help in search and rescue operations. These ocean services
are aimed towards safety at the sea.

460. Which of these European nations has the most indented and longest coastline,
among the following?

a) Norway
b) Finland
c) Moldova
d) Cyprus

Solution: A
Justification: Indented coastline makes for good natural harbours and ports, and gives
the nation a strategic maritime trade advantage.

Direct PDF - https://telegram.me/UPSC_PrelimsTest


https://telegram.me/UPSC_PDF

461. If you are to give the correct North-south order of these European nations, it would
be?
1. Poland
2. Romania
3. Bulgaria
4. Czech Republic

Select the correct answer using the codes below.


a) 1234
b) 1423
c) 2341
d) 3214

Solution: B
Justification:

Direct PDF - https://telegram.me/UPSC_PrelimsTest


https://telegram.me/UPSC_PDF

462. Which of these European nations are open to seas/oceans or have coasts?
1. Serbia
2. Belgium
3. Macedonia
4. Austria

Select the correct answer using the codes below.


a) 1 only
b) 2 and 3 only
c) 2 only
d) 1 and 4 only

Solution: C
Justification:

Direct PDF - https://telegram.me/UPSC_PrelimsTest


https://telegram.me/UPSC_PDF

463. Which of these countries touches Celtic sea, English Channel and Bay of Biscay all?

a) France
b) Switzerland
c) Italy
d) Austria

Solution: A
Justification:

464. Which of these nations border Baltic Sea?


1. Russia
2. Germany
3. Poland
4. Belarus

Direct PDF - https://telegram.me/UPSC_PrelimsTest


https://telegram.me/UPSC_PDF

Select the correct answer using the codes below.


a) 1, 2 and 3 only
b) 2 and 4 only
c) 1 and 3 only
d) 1, 3 and 4 only

Solution: A
Justification:

Direct PDF - https://telegram.me/UPSC_PrelimsTest

You might also like